Download as docx, pdf, or txt
Download as docx, pdf, or txt
You are on page 1of 100

1 A 12yearold glrl wlLh chronlc renal fallure has had perslsLenL eplsLaxls for 13 days LaboraLory

sLudles show

Pemoglobln 72 g/dL
laLeleL counL 173000/mm3
8leedlng Llme 12 mln
roLhrombln Llme 12 sec
arLlal LhromboplasLln Llme (acLlvaLed) 30 sec
Serum urea nlLrogen (8un) 123 mg/dL

Whlch of Lhe followlng ls Lhe mosL llkely cause of Lhe eplsLaxls?
C A) Acqulred plaLeleL dysfuncLlon
C 8) ClrculaLlng lmmune complexes
C C) LryLhropoleLln deflclency
C u) lacLor lll deflclency
C L) Pypocalcemla




Answer ls A


2 A 37yearold woman comes Lo Lhe physlclan because of depressed mood and dally crylng
eplsodes slnce Lhe deaLh of her faLher 6 weeks ago She has had dlfflculLy concenLraLlng decreased
llbldo and a 32kg (7lb) welghL loss durlng Lhls perlod She ls less lnLeresLed ln her usual acLlvlLles
buL conLlnues Lo work producLlvely She sleeps for abouL 8 Lo 10 hours every nlghL Cver Lhe pasL 2
weeks she has been smoklng and drlnklng more frequenLly she now smokes onehalf pack of
clgareLLes dally and drlnks Lwo Lo Lhree glasses of wlne every oLher day She has no hlsLory of serlous
lllness and Lakes no medlcaLlons She ls 163 cm (3 fL 4 ln) Lall and welghs 80 kg (176 lb) 8Ml ls 30
kg/m2 Per LemperaLure ls 37C (986l) pulse ls 641mln and blood pressure ls 108160 mm Pg 1he
remalnder of Lhe examlnaLlon shows no abnormallLles MenLal sLaLus examlnaLlon shows a sad
mood and resLrlcLed affecL Per speech ls normal ln raLe and rhyLhm 1here ls no evldence of sulcldal
ldeaLlon or halluclnaLlons She can recall Lhree of Lhree ob[ecLs afLer 3 mlnuLes LongLerm memory
ls lnLacL Whlch of Lhe followlng ls Lhe mosL llkely dlagnosls?

C A) Alcohol dependence
C 8) 8ereavemenL
C C) 8lpolar dlsorder
C u) CycloLhymlc dlsorder
C L) uysLhymlc dlsorder
C l) Ma[or depresslve dlsorder
C C) Mallngerlng


Answer ls 8

3 An 80yearold man ls broughL Lo Lhe emergency deparLmenL on an 80degree day 2 hours afLer a
syncopal eplsode whlle playlng golf Pe plays golf Lwlce weekly An LCC obLalned aL hls lasL
examlnaLlon 6 monLhs ago showed occaslonal premaLure venLrlcular conLracLlons (vCs) Cn arrlval
hls LemperaLure ls 38C (1004l) blood pressure ls 130/90 mm Pg pulse ls 80/mln and regular and
resplraLlons are 22/mln Cardlac examlnaLlon shows a grade 3/6 laLepeaklng sysLollc e[ecLlon
murmur no brulLs are heard over Lhe caroLld arLerles buL Lhe upsLrokes are delayed Whlch of Lhe
followlng ls Lhe mosL llkely cause of Lhe syncope?

C A) AorLlc sLenosls
C 8) PeaL sLroke
C C) PyperLenslve cardlomyopaLhy
C u) laLeleL emboll
0 L) vCs




AnSWL8 AAAA

4 llve days afLer susLalnlng a 6cm laceraLlon Lhrough Lhe skln and subcuLaneous Llssue of Lhe lefL
upper exLremlLy wlLh a clean knlfe a 32yearold man has lncreaslng Lenderness ln Lhe area of Lhe
laceraLlon 1reaLmenL aL Lhe Llme of ln[ury lncluded cleanslng and dresslng of Lhe wound 1he wound
ls now eryLhemaLous and yellow pus ls expressed when pressure ls applled Whlch of Lhe followlng
ls Lhe mosL llkely mechanlsm for Lhe accumulaLlon of pus?
C A) ChemoLaxls
C 8) uysplasla
C C) Pyperoxla
C u) MeLaplasla
0 L) vasoconsLrlcLlon


AnSWL8 AAAA

3 1wenLyfour hours afLer splenecLomy for blunL Lrauma susLalned ln a moLor vehlcle colllslon a
prevlously healLhy 23yearold man has ollgurla and paln aL Lhe lnclslon slLe lnLraoperaLlve
compllcaLlons lncluded a 30mlnuLe perlod of hypoLenslon and a LoLal blood loss of 23 L requlrlng 4
unlLs of packed red blood cells CurrenLly hls LemperaLure ls 38C (1004l) blood pressure ls
120180 mm Pg pulse ls 1001mln and resplraLlons are 14/mln CenLral venous pressure ls 8 cm P 2C
(n38) 1he lungs are clear Lo ausculLaLlon and breaLh sounds are heard bllaLerally Abdomlnal
examlnaLlon shows no dlsLenLlon bowel sounds are absenL A loley caLheLer ls ln place and over
Lhe pasL 3 hours hls urlne ouLpuL has been 20 mL/h LaboraLory sLudles show a hemaLocrlL of 28 a
serum urea nlLrogen (8un) level of 30 mg1dL and a serum creaLlnlne level of 23 mgfdL serum
elecLrolyLe levels are wlLhln normal llmlLs Whlch of Lhe followlng ls Lhe mosL llkely explanaLlon for
Lhese flndlngs?
C A) AcuLe Lubular necrosls
C 8) loley caLheLer malfuncLlon
C C) Pypervolemla
C u) 1ransfuslon reacLlon
0 L) ureLeral ln[ury



AnSWL8 AAAA

6 A 32yearold man comes Lo Lhe physlclan for a second oplnlon regardlng an enlarged cervlcal
lymph node LhaL he noLed 6 weeks ago Pe has noL had paln or Lenderness buL has been unable Lo
reLurn Lo work desplLe normal flndlngs on a blopsy of Lhe node 4 weeks ago Pe had a mlld upper
resplraLory LracL lnfecLlon 1 monLh ago Pls maLernal uncle recenLly dled of lymphoma Pe sLaLes
LhaL cancer runs ln hls famlly and LhaL for Lhe pasL several years he has LhoughL LhaL he wlll have
some klnd of cancer evenLually Whlch of Lhe followlng ls Lhe mosL llkely dlagnosls?
C A) AsLhma
C 8) Converslon dlsorder
C C) lacLlLlous dlsorder
C u) Pypochondrlasls
C L) Mallngerlng
C l) ulmonary embolus
0 C) SomaLlzaLlon dlsorder


AnSWL8 uuuu

7 A 32yearold woman gravlda 3 para 2 aL 40 weeks gesLaLlon ls admlLLed Lo Lhe hosplLal ln
labor vaglnal and anal culLures aL 36 weeks gesLaLlon were poslLlve for group 8 sLrepLococcus She
has no known drug allergles Whlch of Lhe followlng ls Lhe mosL approprlaLe pharmacoLherapy for
Lhls paLlenL before dellvery?
C A) lnLravenous azlLhromycln
C 8) lnLravenous cllndamycln
C C) lnLravenous penlclllln
C u) Cral amoxlclllln and clavulanaLe
C L) Cral eryLhromycln
C l) no LreaLmenL unLll afLer dellvery



Ccccccc


8 A 38yearold woman comes Lo Lhe physlclan because of a lowgrade fever and generallzed rash
for 4 days She ls currenLly recelvlng cefazolln Lherapy for chronlc osLeomyellLls Per LemperaLure ls
382C (1008l) blood pressure ls 1301108 mm Pg and pulse ls 1001mln 1here ls a falnL dlffuse
maculopapular rash LxamlnaLlon of Lhe back shows no cosLoverLebral angle Lenderness Cardlac and
pulmonary examlnaLlons show no abnormallLles LaboraLory sLudles show

LeukocyLe counL 108001mm3
SegmenLed neuLrophlls 60
8ands 8
Loslnophlls 4
LymphocyLes 20
MonocyLes 8

Serum
urea nlLrogen (8un) 20 mg/dL
CreaLlnlne 16 mg/dL
urlne
W8C 121hpf
88C 81hpf
88C casLs none
W8C casLs rare

Loslnophlls are found ln Lhe urlne sedlmenL Whlch of Lhe followlng ls Lhe mosL llkely explanaLlon for
Lhese flndlngs?

C A) AcuLe Lubular necrosls
C 8) llbromuscular dysplasla
C C) lnLersLlLlal nephropaLhy
C u) olyarLerlLls nodosa
C L) yelonephrlLls
C l) Wegeners granulomaLosls


Ccccccccccccccccc

9 A healLhy 60yearold woman comes Lo Lhe physlclan for a rouLlne examlnaLlon She has no
hlsLory of lllness over Lhe pasL year and has never had an operaLlon She Lakes no medlcaLlons
Menopause occurred 6 years ago She welghs 37 kg (123 lb) and ls 160 cm (63 ln) Lall elvlc
examlnaLlon shows aLrophlc exLernal genlLalla and a small mldposlLloned uLerus 1he lefL ovary ls 3
x 3 cm Lhe rlghL ovary ls noL palpable Whlch of Lhe followlng ls Lhe mosL approprlaLe nexL sLep ln
managemenL?

C A) 8eexamlnaLlon ln 1 monLh
C 8) 8eexamlnaLlon ln 1 year
C C) CbLaln paLlenLs medlcal records
C u) MeasuremenL of serum progesLerone level
C L) elvlc ulLrasonography


Leeeeeeeeeeeeeeeeeeeee


10 Cn Lhe flfLh day of a 7day crulse ln Lhe wesLern Carlbbean a 37yearold woman develops
headaches fever chllls abdomlnal dlscomforL and waLery dlarrhea over a 12hour perlod Many
oLher passengers and crew members have had slmllar sympLoms Per LemperaLure ls 37C (986l)
Abdomlnal examlnaLlon shows dlffuse Lenderness wlLhouL rebound bowel sounds are hyperacLlve
Per leukocyLe counL ls 11000/mm3 Crams sLaln of a sLool speclmen shows a small number of
neuLrophlls A sLool culLure grows Salmonella enLerlLldls 1wo days afLer LreaLmenL wlLh blsmuLh
subsallcylaLe and oral rehydraLlon her sympLoms subslde Whlch of Lhe followlng ls Lhe mosL
approprlaLe lmmedlaLe measure Lo prevenL furLher spread of Lhls paLhogen?

C A) CancellaLlon of shore leave for all crew members
C 8) LllmlnaLlon of seafood and shellflsh from Lhe shlps menu
C C) Lxcluslve use of pasLeurlzed eggs
C u) PyperchlorlnaLlon of Lhe shlps drlnklng waLer
C L) 8easslgnmenL of all foodhandllng personnel Lo oLher duLles
C l) 1reaLmenL of all affecLed persons wlLh doxycycllne


CCCCCCCCCCCCCCCCCC

11 A 37yearold man wlLh a 10year hlsLory of Lype 2 dlabeLes melllLus comes Lo Lhe physlclan fora
rouLlne examlnaLlon Pls lasL offlce vlslL was 6 monLhs ago Pe feels well CurrenL medlcaLlons
lnclude enalaprll and glyburlde Pe ls 168 cm (3 fL 6 ln) Lall and welghs 84 kg (183 lb) 8Ml ls 30
kg/mZ Pls pulse ls 60/mln and blood pressure ls 100/70 mm Pg lunduscoplc examlnaLlon shows
sofL and hard exudaLes LaboraLory sLudles show
Pemoglobln Al 12
Serum
urea nlLrogen (8un) 23 mg/dL
CreaLlnlne 14 mg/dL
urlne proLeln 1+
Whlch of Lhe followlng ls Lhe mosL approprlaLe addlLlonal pharmacoLherapy?

C A) ALenolol
C 8) CapLoprll
C C) PydrochloroLhlazlde
C u) MeLformln
0 L) verapamll



uuuuuuuuuuuuuuuuuuuuu

12 An 82yearold man ls broughL Lo Lhe physlclan by hls nelghbors because he looks lll 1hey say
LhaL he subslsLs prlmarlly on hoL dogs and canned meaLs and LhaL hls slovenly hablLs have aLLracLed
raLs Lo Lhe nelghborhood Pe does noL drlnk alcohol buL ls forgeLful LxamlnaLlon shows a pleasanL
dlrLy confused man who appears Lhln Pe has dlffuse purpura on hls legs and perlfolllcular
lnflammaLlon Whlch of Lhe followlng ls Lhe mosL llkely cause of Lhese flndlngs?
C A) AplasLlc anemla
C 8) Leukemla
C C) 8aLblLe fever
C u) Scurvy
C L) Warfarln lngesLlon



uuuuuuuuuuuuuuuuuuu

13 A 6monLhold glrl ls broughL Lo Lhe physlclan because of fever cough and coryza for 1 day Per
pulse ls 1001mln and resplraLlons are 301mln Per cough ls harsh and sounds llke a dogs bark
1here ls lnsplraLory sLrldor and lnLercosLal reLracLlons Whlch of Lhe followlng ls Lhe mosL llkely
dlagnosls?
C A) AsLhma
C 8) 8acLerlal LrachelLls
C C) 8ronchlollLls
C u) lorelgn body ln Lhe small alrways
C L) lorelgn body ln Lhe Lrachea
C l) LaryngoLracheobronchlLls
C C) neumonla
C P) neumoLhorax
0 l) ulmonary edema


LaryngoLracheobronchlLlsC8Cu

llllllllllllllllllll


14 An 80yearold man has had poor balance for 6 monLhs Pe has a hlsLory of hyperLenslon LreaLed
wlLh hydrochloroLhlazlde Pls blood pressure ls 136/86 mm Pg neurologlc examlnaLlon shows mlld
Lremor of Lhe hands when hls arms are ouLsLreLched and decreased
vlbraLory sensaLlon aL Lhe knees ueep Lendon reflexes of Lhe quadrlceps and gasLrocnemlussoleus
muscles are hyperacLlve 8ablnskls slgn ls presenL bllaLerally Pe ls unable Lo sLand wlLh hls eyes
closed Whlch of Lhe followlng ls mosL conslsLenL wlLh normal
agerelaLed changes?

C A) 8ablnskls slgn
C 8) PyperacLlve deep Lendon reflexes of Lhe gasLrocnemlussoleus muscles
C C) lnablllLy Lo sLand wlLh Lhe eyes closed
C u) 8educed vlbraLory sensaLlon aL Lhe knees
0 L) 1remor of Lhe ouLsLreLched hands
Should be ddddddd


15. A previously healthy 6-month-old boy is brought to the physician because oI a 12-hour
history oI vomiting and diarrhea. He vomits aIter all Ieedings, the vomitus does not contain
blood or bile. His mother says that he has had Iewer wet diapers than usual during this period.
He appears dehydrated and is crying without tears. He is at the 50th percentile Ior length and
30th percentile Ior weight. He appears lethargic. His temperature is 38C (100.4F), pulse is
180/min, and blood pressure is 60/40 mm Hg. Examination shows sunken eyes, dry mucous
membranes, and a sunken anterior Iontanel. Arterial blood gas analysis on room air shows:
pH 7.2
PCO2 38 mm Hg
PO2 90 mm Hg

Which oI the Iollowing is the most likely explanation Ior this patient's arterial blood gas
Iindings?
O A) Excessive metabolic acid Iormation
O B) Impaired ventilation
O C) Increased chloride loss (aincrease HCO3 which is not the case here)
O D) Increased CO2 concentration in the extracellular Iluid
0 E) Increased metabolic acid produced by the gastrointestinal tract



AA,?? hypovolumic septic shock-a hypoperIusion to organs-alactiic acidodis



16. A 42-year-old man is brought to the emergency department because oI a 2-day history oI
muscle spasms and jaw stiIIness. He is sexually active with three partners and uses condoms
inconsistently. He has a 5-year history oI intravenous heroin use. He appears irritable. His
temperature is 38.8C (101.8F), pulse is 120/min, and blood pressure is 152/96 mm Hg.
Physical examination shows Iacial and paraspinal muscle spasms and rigidity; stimulation oI
the muscles results in paroxysmal spasms. Mobility oI the jaw is decreased. Neurologic
examination shows hyperreIlexia. Which oI the Iollowing is most likely to have prevented
this condition?
O A) Consistent condom use
O B) Botulism antitoxin therapy
O C) Prednisone therapy
O D) RiIampin therapy
O E) Tetanus toxoid vaccination
0 F) Vitamin B1 (thiamine) supplementation


I chose EEE

17. A 72-year-old woman comes to the physician because oI increasing episodes oI urinary
incontinence over the past 6 months. Her incontinence usually occurs at night, she Ieels no
sensation to urinate prior to the episodes. The episodes are less Irequent iI she schedules her
trips to the bathroom and restricts liquid intake several hours prior to bedtime. She has a 20-
year history oI type 2 diabetes mellitus. Current medications include metIormin and
glyburide. Funduscopic examination shows mild retinopathy. Pelvic examination shows
normal vaginal mucosa. Sensation to pinprick is decreased in a stocking-glove distribution.
Urinalysis shows 1 protein and no leukocyte esterase or nitrites. Which oI the Iollowing is
the most likely cause oI this patient's urinary symptoms?

O A) Functional incontinence
O B) Hypersensitivity oI the detrusor muscle
O C) OverIlow oI urine Irom large residual volumes
O D) Urethral atrophywith loss oI urethrovesical angle
O E) Normal aging


I chose CCC

18. An aIebrile 2-year-old boy has had right ear pain Ior 3 days. He has been swimming
everyday Ior the past week. Purulent Iluid is draining Irom the right external ear canal, and
manipulation oI the pinna is painIul. No abnormalities are noted on visualization oI tympanic
membranes. Which oI the Iollowing is the most likely causal organism?
O A) Haemophilus inIluenzae type b
O B) Moraxella catarrhalis
O C) Pasteurella multocida
O D) Pseudomonas aeruginosa
O E) Streptococcus pneumoniae


I chose DDD

19. Over the past 2 weeks, a 60-year-old man has had shortness oI breath on exertion. He also
has paroxysmal nocturnal dyspnea with two-pillow orthopnea. He has taken aspirin daily
since a myocardial inIarction 3 years ago. He has a history oI atrial Iibrillation well controlled
with digoxin and type 2 diabetes mellitus treated with diet. His blood pressure is 136188 mm
Hg, pulse is 98/min and irregular, and respirations are 20/min. Jugular-venous pressure is
increased. Breath sounds are decreased over the right lung base.. there is dullness to
percussion. Cardiac examination shows an S. gallop. There is 2 edema oI the lower
extremities. Pulse oximetry shows an oxygen saturation oI 90. Which oI the Iollowing is
the most appropriate next step in diagnosis?
O A) X-ray Iilm oI the chest
O B) Ambulatory ECG monitoring
O C) Thallium stress test
O D) Echocardiography
O E) Ventilation-perIusion lung scans


Ddddddd


20. A 47-year-old woman comes to the emergency department because oI severe abdominal
pain Ior 3 hours. The pain began aIter a 2-week drinking binge. She has a 15-year history oI
alcoholism. She has no history oI jaundice or hepatitis. Current medications include
multivitamins and iron. Her temperature is 38C (100.4F), blood pressure is 110/80 mm Hg,
and pulse is 110/min. Examination shows mild jaundice and diIIuse spider angiomata over
the trunk and abdomen. The liver is tender to palpation. A complete blood count shows mild
anemia with normal red cell indices. Ultrasonography oI the abdomen shows normal-sized
hepatic ducts. Which oI the Iollowing is the most likely set oI laboratory Iindings?

Total Indirect Alkaline
Bilirubin Bilirubin Phosphatase Reticulocyte
(mg/dL) (mg/dL) (U/L) (ALT, GPT) (U/L) Count ()
O A) 2 0.9 80 30 1.2
O B) 3 2.8 70 30 1.0
O C) 3 2.8 80 20 3.0
O D) 4 2.0 800 200 1.5
O E) 4 1.0 150 400 1.0


AAAAAAAAAAAAAAAAAAAAAAAAA
(Mixed hyperbilirubinemia due to alcohol damage to the liver)

21. A 3-month-old boy is brought to the physician in January because oI diIIiculty breathing,
clear nasal discharge, and cough Ior 24 hours. His temperature is 37.6C (99.6F), blood
pressure is 88/54 mm Hg, pulse is 168/min, and respirations are 60/min. Bilateral wheezing,
prolonged expiration, and a grade 2/6 systolic murmur along the leIt sternal border are heard.
The liver is palpated 3 cm below the right costal margin. An x-ray Iilm oI the chest shows
bilateral hyperinIlation and no cardiomegaly. Which oI the Iollowing is the most likely
diagnosis?
O A) Adenovirus pneumonia
O B) Congestive heart Iailure
O C) InIluenza A virus pneumonia
O D) Respiratory syncytial viral bronchiolitis
O E) Staphylococcal pneumonia
0 F) Status asthmaticus


DDDDDDDDDDDDDDDDD

22. A 22-year-old woman comes to the physician because oI diIIuse constant headaches and
vision problems Ior 3 months, the headaches are worse in the morning when she awakens
Irom sleep. She also has had brieI episodes oI loss oI vision in both eyes. She has had an 18-
kg (40-Ib) weight gain over the past year. She now weighs 100 kg (220 lb) and is 163 cm (64
in) tall. Visual acuity is 20/30 bilaterally. Visual Iields are Iull, but the blind spots are
enlarged bilaterally. Funduscopic examination shows marked blurring oI the optic disc
margins bilaterally. The remainder oI the neurologic examination shows no abnormalities.
Which oI the Iollowing is the most likely diagnosis?
O A) Amaurosis Iugax
O B) Central retinal vein occlusion
O C) Glaucoma
O D) Macular degeneration
O E) Migraine
O F) Nutritional optic neuropathy
O G) Optic neuritis
O H) Pituitary adenoma
O I) Pseudotumor cerebri
0 J) Temporal arteritis


Answer should be IIIII



23

AAAAAAAAAAAAAAAAAA




24. A 49-year-old man has had progressive shortness oI breath over the past year. He now has
dyspnea aIter walking up one Ilight oI stairs. He has Iine crackles bilaterally on auscultation
oI the lungs. Spirometry shows:

Vital capacity (VC) decreased
FEV1 decreased
Ratio oI FEV, to VC increased

Arterial blood gas analysis on room air:
pH 7.42
PCO2 60 mm Hg
PO2 34 mm Hg

Which oI the Iollowing is the most likely diagnosis?
0 A) Asthma
p B) Bronchiectasis
o C) Chronic bronchitis
p D) Emphysema
p E) Pulmonary Iibrosis



EEEEEEEEEEEEEEEEEEEEEEEE


25. A previously healthy 24-year-old woman, gravida 3, para 3, is brought to the emergency
department because oI deep, sharp, intermittently severe pain in the leIt lower quadrant oI the
abdomen Ior 2 hours. Her last menstrual period was 3 weeks ago. She takes no medications.
She is in obvious distress and is lying on her leIt side with her lower extremities drawn up
against her abdomen. Her temperature is 37C (98.6F), blood pressure is 140/70 mm Hg,
and pulse is 125/min. Abdominal examination shows rigidity and tenderness. Pelvic
examination shows a 12-cm mass in the leIt lower quadrant oI the abdomen. Which oI the
Iollowing is the most appropriate next step in management?

O A) Ultrasound-guided aspiration
O B) Dilatation and curettage
O C) Exploratory laparotomy
O D) Hysterectomy



CCCCCCCCCCCCCCCCCCCCCCCCCCC




26. Fourteen hours aIter admission to the hospital Ior treatment oI severe hypertension, a 32-
year-old woman has stridor. On admission, she was given captopril. She appears anxious. Her
blood pressure is 140/85 mm Hg, pulse is 140/min, and respirations are 32/min. Examination
shows swelling oI the lips and tongue. DiIIuse stridorous wheezes are heard on auscultation.
There is diminished air movement. Which oI the Iollowing is the most appropriate next step
in management?

p A) Observation only
p B) Measurement oI serum captopril level
O C) Measurement oI serum IgE level
o D) X-ray Iilm oI the chest
0 E) Tracheal intubation


EEEEEEEEEEEEEEEEEEEEEEEE

27. A 65-year-old woman has had progressive irritability, palpitations, and heat intolerance
Ior 6 months. She has had a 7-kg (15-Ib) weight loss during this period. She has had a neck
mass Ior more than 10 years. 1311 scan shows an enlarged thyroid gland with multiple areas
oI increased and decreased uptake. Which oI the Iollowing is the most likely diagnosis?
O A) Graves' disease
O B) Multinodular goiter
O C) Thyroiditis
O D) Toxic adenoma
0 E) Triiodothyronine (T3) thyrotoxicosis



BBBBBBBBBBBBBBBBBBBBBBB

For each patient with chronic knee pain, select the most appropriate next step in management.

C A) Anterior cruciate ligament reconstruction
C B) Arthroscopic partial meniscectomy
C C) Quadriceps strengthening exercises
C D) Sympathetic blockade
C E) Total knee replacement
C F) Upper tibial osteotomy


CCCCCCCCCCCCCCCCCCCCCCCCCCCCCC


30. A 70-year-old man comes to the physician because oI knee pain that has been present Ior
10 years. He is unable to comIortably walk Iurther than one block and has diIIiculty sleeping
because oI the pain. He has pain on both medial and lateral sides oI the knee. Range oI
motion is Irom 15 to 100 degrees, there is a bowleg deIormity when he stands.

EEEEEEEEEEEEEEE

31. A 55-year-old man with alcoholic cirrhosis is hospitalized Ior 2 weeks in April Ior
treatment oI gastrointestinal bleeding. His last immunization Ior diphtheria-tetanus (Td) was
6 years ago. Which oI the Iollowing is the most appropriate management prior to discharge?

O A) Administration oI immune globulin
O B) Inactivated poliovirus vaccine
O C) InIluenza virus vaccine
O D) Pneumococcal vaccine
0 E) Td toxoids

DDD


32. One week aIter cholecystectomy Ior acute cholecystitis, a 57-year-old woman comes to
the physician because oI a 1-day history oI abdominal cramps and watery, green, Ioul-
smelling diarrhea. She Iinished a 7-day course oI ceIoxitin 3 days ago. She appears
dehydrated. Her temperature is 39.8C (103.6F), pulse is 115/min, and blood pressure is
105/70 mm Hg. Abdominal examination shows moderate distention and diIIuse tenderness,
there is no guarding. Test oI the stool Ior occult blood is positive, the stool contains
leukocytes. Her hemoglobin level is 10.8 g/dL, and leukocyte count is 39,000/mm3. The
patient is admitted to the hospital, and administration oI intravenous Iluids is begun. Which
oI the Iollowing is the most appropriate next step in pharmacotherapy?
O A) Intravenous amphotericin
O B) Intravenous ampicillin, clindamycin, and gentamicin
O C) Intravenous corticosteroids
O D) Oral diphenoxylate and atropine
0 E) Oral metronidazole



EEEEEEEEEEEEEEEEEEE



A picture here


33. A 32-year-old comes to the physician because oI painless growths around her anal region
that she Iirst noticed 4 months ago. Over the past 10 years, she has had sexual intercourse
with multiple partners. She has a 4-year history oI Crohn's disease. Medications include
sulIasalazine and metronidazole. Her Iather has psoriasis. A photograph oI the lesions is
shown. Examination shows no other abnormalities. Which oI the Iollowing is the greatest risk
Iactor Ior this condition?
O A) Crohn's disease
O B) Family history oI psoriasis
O C) Multiple sexual partners
O D) Treatment with metronidazole
0 E) Treatment with sulIasalazine


CCCCCCCCCCCCCCCCCCCCCC



The response options Ior the next two items are the same. You will be required to select one
answer Ior each item in the set.

For each patient with dysphagia, select the most likely diagnosis.

O A)Achalasia
O B)Adenocarcinoma oI the esophagus
O C)Candidal esophagitis
O D) DiIIuse esophageal spasm
O E) Polymyositis
0 F)ReIluxesophagitis
0 G)Scleroderma (systemic sclerosis)
0 H)Squamous cell carcinoma oI the esophagus
0 I)Viral esophagitis


34. A 42-year-old man with an 8-year history oI intermittent diIIiculty swallowing solids and
liquids has had increasingly severe exacerbations because oI stress over the past 6 months.
He also has nocturnal regurgitation and cough. Esophageal motility studies show Iailure oI
relaxation oI the lower esophageal sphincter.


AAAAAAAAAAAAAAA


For each patient with dysphagia, select the most likely diagnosis.

W A) Achalasia 0 F)ReIluxesophagitis
Q B) Adenocarcinoma oI the esophagus 0 G)Scleroderma (systemic sclerosis)
p C) Candidal esophagitis 0 H)Squamous cell carcinoma oI the esophagus
p D) DiIIuse esophageal spasm 0 I)Viral esophagitis
0 E) Polymyositis


35. A 45-year-old woman has had generalized weakness Ior 1 year, increasing diIIiculty
walking up stairs over the past 6 months, and diIIiculty swallowing Ior 1 month. Examination
shows weakness oI the proximal muscles oI the extremities. Ear, nose, and throat
examinations and manometry show decreased contractions oI the pharynx with decreased
upper esophageal tone.

EEEEEEEEEEEEEEEE


36. A 42-year-old man with hypertension is brought to the emergency department 1 hour
aIter the onset oI chest discomIort, severe headache, irritability, and conIusion. His blood
pressure is 2201148 mm Hg. Examination shows papilledema. Urinalysis shows blood. An
ECG shows ischemic changes with leIt ventricular hypertrophy. A CT scan oI the head shows
no abnormalities. Which oI the Iollowing is the most appropriate pharmacotherapy?

O A) Atenolol
O B) Captopril
O C) Diltiazem
O D) Furosemide
O E) Guanethidine
O F) Hydralazine
O G) Losartan
O H) Methyldopa
O I) Nitroprusside
O J) Prazosin
0 K) Thiazide diuretic


IIIIIIIIIIIIIIIIIIII


37. A 2-year-old girl is Iound Iloating Iacedown in a swimming pool. On rescue, she
immediately coughs and breathes spontaneously. She is conscious and oriented. Without
treatment, which oI the Iollowing is the most likely outcome Ior this child?
A) Complete recovery
B) Development oI acute respiratory distress syndrome
C) Hemolysis
D) Pulmonary hypertension
E) Severe neurologic deIicit


AAAAAAAAAAAAAAAAAAAAAAAAAAA


38. A 25-year-old woman has had daily cough and wheezing Ior 4 months. She has had
diIIiculty sleeping because oI her symptoms. Inspiratory and expiratory wheezes are heard
with a slightly prolonged exhalation phase. Rz Adrenergic agonist therapy by inhalation is
begun. She is still mildly symptomatic during the day with bronchospastic episodes occurring
three times weekly at night. Which oI the Iollowing medications should be added to the
regimen?
O A) Inhaled glucocorticoids
O B) Inhaled ipratropium
O C) Oral erythromycin
O D) Oral Iurosemide
0 E) Oral theophylline

AAAAAAAAAAAAAAAAAAAAAAAAA

39. Five years aIter being shot in the right thigh, a 21-year-old man comes to the emergency
department because oI a busing sensation adjacent to the scar. At the time oI the initial
wound, he was discharged aIter 6 hours oI observation with no Iractures or soIt-tissue
swelling. A loud murmur is heard on auscultation, there is a thrill. He has dilated varicose
veins with incompetent valves in the lower leg. Which oI the Iollowing is the most likely
diagnosis?
O A) Arterial spasm
O B) Arteriovenous Iistula
O C) Deep venous thrombosis
O D) Occlusion oI the superIicial Iemoral artery
O E) Pseudoaneurysm

bbbbbbbbbbbbbbbbbbbbbbbbbbbbbbbbbb


40. A study is conducted to investigate the prevalence oI past inIection with genital herpes
among students attending a large coeducational Midwestern university. AIter obtaining
inIormed consent, the study participants undergo antibody testing Ior herpes simplex virus 2
(HSV). Which oI the Iollowing is the most important Iactor in determining the validity oI a
positive test?

A) The number oI sexual partners oI the student
B) The proportion oI students inIected with HSV
C) The proportion oI students with negative tests
D) The proportion oI students with positive tests
E) The total number oI students screened


BBBBBBBBBBBBBBBBB MY ANSWER

41. During a routine examination, a 20-year-old man is Iound to have a Iirm, bilaterally
enlarged thyroid gland and lymphadenopathy. His Iather died oI thyroid cancer at the age oI
40 years. Which oI the Iollowing hormones is most likely associated with this patient's
Iindings?
O A) ACTH
O B) ADH (vasopressin)
O C) Aldosterone
O D) Calcitonin
O E) Growth hormone
O F) G3-hCG
O G) Insulin
O H) Parathyroid hormone
O I) Thyroid-stimulating hormone
0 J) Thyroxine (T4)


DDDDDDDDDDDDDDDDDDDDDDDDD

42. Twelve days aIter sustaining a cerebral inIarction, a 72-year-old man has Iever and
cough. Initial symptoms included inability to move his right arm and leg, swallow, speak, or
respond to questions. He has been receiving a diet oI pureed Ioods since he recovered his
ability to swallow 3 days ago. He is wearing Ialse teeth. His temperature now is 38.8C
(101.8F), blood pressure is 135/85 mm Hg, pulse is 94/min, and respirations are 28/min.
Examination shows moderate weakness oI the Iacial muscles and right extremities. Gag
reIlex is absent. Breath sounds are decreased, and there is dullness to percussion over the
right lung base posteriorly. An x-ray Iilm oI the chest shows an inIiltrate in the posterior
basal segment oI the right lung. Which oI the Iollowing is most likely to prevent recurrence
oI this patient's lung condition?
O A) Removal oI Ialse teeth
O B) Suppression oI gastric acid production
O C) Chronic antibiotic prophylaxis
O D) Administration oI metoclopramide to increase gastrointestinal motility
O E) Insertion oI a Ieeding jejunostomy tube

Should be AAAAAAA


43. A previously healthy 72-year-old man is brought to the physician because oI two episodes
oI transient paresis in the leIt arm during the past 2 days; each episode lasted Ior 3 to 4
minutes. No bruits are heard over the carotid arteries, and there are no cardiac murmurs.
Carotid duplex ultrasonography and conIirmatory arteriography show a 30 stenosis oI the
internal carotid arteries bilaterally. Which oI the Iollowing is the most appropriate next step
in management?
A) Aspirin therapy
B) Heparin therapy
C) NiIedipine therapy
D) Placement oI a carotid endovascular stent
E) Carotid endarterectomy


AAAAAAAAAAAAAAAAAA


44. A 35-year-old man with a 10-year history oI persistent hallucinations and delusions
comes Ior a Iollow-up examination. He has been treated with haloperidol, chlorpromazine,
and Iluphenazine with minimal relieI oI symptoms. He is currently taking haloperidol. On
examination, he walks slowly with no arm swing and has no Iacial movements. Which oI the
Iollowing is the most likely cause oI his motor behavior?
O A) Excess limbic y-aminobutyric acid activity
O B) Increased serum prolactin level
O C) Nigrostriatal dopamine blockade
O D) Noradrenergic depletion in the Irontal lobe
O E) Ventral tegmental dopamine blockade

CCCCCCCCCCCCCCCCCCCCCC


45. An 82-year-old woman comes to the physician because oI a 2-month history oI
progressive shortness oI breath with exertion and a 3-week history oI right upper quadrant
abdominal discomIort. She has chronic obstructive pulmonary disease, hypertension, and
rheumatoid arthritis. Medications include methotrexate, hydrochlorothiazide, naproxen,
albuterol metered-dose inhaler, and aspirin. Examination shows pallor. Breath sounds are
decreased. The liver is enlarged, Iirm, and mildly tender. Test oI the stool Ior occult blood is
positive. Her hematocrit is 27, and mean corpuscular volume is 76 pm3. A CT scan oI the
abdomen shows multiple hypodense lesions in the liver. Which oI the Iollowing is the most
likely diagnosis?
O A) Cholangiocarcinoma
O B) Hepatocellular carcinoma
O C) Lymphoma
O D) Metastatic colon cancer
O E) Metastatic lung cancer
O F) Metastatic pancreatic cancer

DDDDDDDDDDDDDDDDDDDDDDDDD
Share Twitter
Replies to This Discussion
Permalink Reply by usmle99 on March 13, 2009 at 7:26am
46. On routine health maintenance examination, a 23-year-old man has a systolic
murmur. There is no Iamily history oI heart disease. His blood pressure is 110170 mm
Hg, and pulse is 601min. Cardiac examination shows a point oI maximal impulse in
the IiIth intercostal
space at the midclavicular line, there is a normal S1 and a split, Iixed S2. A grade 216,
systolic ejection murmur is heard at the leIt sternal border, there is no change with
inspiration, Valsalva's maneuver, or hand grip. An exercise stress test shows no
abnormalities. Which oI the Iollowing is the most likely cause oI this patient's cardiac
murmur?
O A) Aortic stenosis
O B) Atrial septal deIect
O C) Hypertrophic cardiomyopathy
O D) Mitral valve prolapse
O E) Ventricular septal deIect


BBBBBBBBBBBBBBBBBBBBBBBBBB


nbme3 sec2
#223301
quansar - 09/25/07 00:43

here you go the section 2, I will work on the sec3/4 in next Iew days




1. For the past 10 years, a 28-year-old woman has Iollowed a strict diet that prohibits
dairy products, meat, Iish, and Iowl. On routine examination, her hematocrit is 30,
and mean corpuscular volume is 122 pm3. Which oI the Iollowing is the most
appropriate next step in diagnosis?

O A) Erythrocyte Iolate determination
O B) Serum iron and total iron-binding capacity determinations
O C) Serum thyroid hormone determination
O D) Serum urea nitrogen (BUN) determination
O E) Serum vitamin B12 (cyanocobalamin) determination

EEEEEEEEEEEEEEE


2. A 47-year-old man comes to the physician because oI a persistent nonproductive
cough Ior 3 months. He has not had Iever, rhinorrhea, or nasal congestion. He was
seen by a physician 3 months ago Ior the cough and treated with amoxicillin and an
over-the-counter cough medication. He has a 3-month history oI hypertension well
controlled with lisinopril. He is in no distress. He weighs 79 kg (175 lb) and is 178 cm
(70 in) tall. His temperature is 37.1C (98.8F), blood pressure is 130178 mm Hg,
pulse is 721min and regular, and respirations are 161min. Examination shows no
abnormalities. A complete blood count and x-ray Iilms oI the chest show no
abnormalities. Which oI the Iollowing is the most appropriate next step in
management?

O A) X-ray Iilms oI the sinuses
O B) Discontinue lisinopril therapy
O Q Add a beclomethasone inhaler to the medication regimen
O D) Add ciproIloxacin to the medication regimen
O E) Add hydrocodone to the medication regimen


BBBBBBBBBBBBBBBBBBBBB

3. A 12-year-old girl is brought to the physician by her mother because she is
concerned about her daughter's diIIiculty making Iriends and socializing. Her
development and adjustment had been normal until 6 months ago when she began to
reIuse to use restrooms at school or eat in the caIeteria. Her mother describes her as a
quiet, serious child who does not readily interact with other people. Her teachers
report that her concentration varies. at times, she appears to be daydreaming. On
examination, she is reserved but pleasant and appears to be oI normal intelligence.
Her speech is normal in rate and rhythm. She says that she is concerned that her voice
will Iail her iI she has to read aloud in class. Which oI the Iollowing is the most likely
diagnosis?

O A) Attention-deIicitthyperactivity disorder, inattentive type
O B) Autistic disorder
O C) Expressive language disorder
O D) Oppositional deIiant disorder
O E) Selective mutism
O F) Social phobia
O G) Age-appropriate behavior

FFFFFFFFFFFFFFFFFF



A picture here Ior q4

4. A 25-year-old woman is brought to the emergency department 45 minutes aIter
being hit in the right eye with a tennis ball. She has severe pain and decreased vision
in the injured eye. Visual acuity is 201400 on the right. A photograph oI the eye is
shown. Which oI the Iollowing is the most likely diagnosis?
O A) Acute angle-closure glaucoma
O B) Anterior iritis
O C) Cataract
O D) Hyphema
O E) Hypopyon

DDDDDDDDDDDDDDD

5. A 44-year-old woman is brought to the emergency department 40 minutes aIter
being stabbed in the leIt groin. Her blood pressure is 128/84 mm Hg, pulse is
100/min, and respirations are 16/min. Examination oI the wound shows a small
hematoma and no external bleeding. Popliteal and pedal pulses are palpable in the leIt
lower extremity below the injury. Lacerations oI the Iemoral artery and vein are Iound
and repaired. Two days postoperatively, she has progressively severe pain oI the leIt
lower extremity and swelling oI the leg Irom the knee down. Which oI the Iollowing
is the most likely cause oI her new symptoms?

O A) Arterial embolism
O B) Compartment syndrome
O C) Lymphatic disruption Irom the injury
O D) Thrombosis oI the Iemoral vein
0 E) Wound hemorrhage

BBBBBBBBBBBBBB


6. The crude mortality rate Ior coronary artery disease in Community A is twice the
crude mortality rate Ior coronary artery disease in Community B. The age-
standardized mortality rates Ior coronary artery disease in the two communities are
the same. These Iindings are most consistent with which oI the Iollowing inIerences?

A) Coronary artery disease mortality in the elderly is higher in Community A than
Community B
B) Coronary artery disease mortality in young adults is higher in Community A than
Community B
C) The population oI Community B is younger than the population oI Community A
D) The proportionate mortality Irom coronary artery disease is higher in Community
A than Community B
E) The two communities have similar age distributions

CCCCCCCCCCCCCC

7. An otherwise healthy 28-year-old man comes to the physician because oI a dry
hacking cough Ior 3 months. A routine examination 6 months ago showed no
abnormalities. His paternal grandIather had colon cancer. Examination today shows
normal Iindings. An x-ray Iilm oI the chest shows a large anterior mediastinal mass.
Which oI the Iollowing is the most likely origin oI this neoplasm?
O A) Colon
O B) Kidney
O C) Prostate
O D) Stomach
0 E) Testicle

EEEEEEEEEEEEEEEEEE

A testis tumor may become metastatic and maniIest with large retroperitoneal and/or
chest lesions, while the primary tumor is nonpalpable. Scrotal ultrasonography may
locate the primary tumor. Histopathology oI the primary testis oIten shows a Iocus oI
tumor surrounded by Iibrous scar, termed burned-out testis cancer.

8. A 50-year-old man has a cardiac arrest during sexual intercourse. He is promptly
resuscitated and recovers uneventIully Irom an acute anterior wall myocardial
inIarction. At discharge Irom the hospital, he and his wiIe inquire about the saIety oI
Iuture sexual intercourse. Which oI the Iollowing is the most appropriate advice?
O A) Psychiatric consultation is necessary
O B) Sexual intercourse should be perIormed while wearing an ambulatory ECG
monitor
O C) Sexual intercourse will be saIe when exercise tolerance improves
O D) They should wait 3 months beIore having sexual intercourse
O E) They should have sexual intercourse as soon as possible to overcome the Iear oI
another event



CCCCCCCCCCCCCCCCCC



9. A 68-year-old woman with terminal metastatic breast cancer is living at home with
her son. She has a living will requesting that she be allowed to "die in peace." She is
unresponsive to voice and has not had any Iood or liquids Ior 3 days. Her son
disagrees with her decision not to accept Iurther therapy, including chemotherapy,
antibiotics, hospitalization, and enteral or parenteral nutrition. Which oI the Iollowing
is the most appropriate next step in management?
p A) Abide by the decisions oI the next oI kin
p B) Obtain a court order to allow additional therapy to be given
O C) Start enteral Ieedings
p D) Admit her to the hospital
0 E) No Iurther intervention


EEEEEEEEEEEEEEEEEEE


10. A 57-year-old woman comes to the physician Ior a Iollow-up examination. She
has a 5-year history oI hypercalcemia, which was diagnosed with routine laboratory
studies, and her serum calcium levels have ranged Irom 10.8 mg/dL to 11.5 mg/dL.
She declined Iurther evaluation in the past because she "Ielt well." She takes no
medications. Her last menstrual period was 7 years ago. She maintains a weight oI 67
kg (148 lb) and is 170 cm (67 in) tall, BMI is 23 kg/m2. Her blood pressure is 126/80
mm Hg, and pulse is 66/min. Examination shows no abnormalities. Which oI the
Iollowing is the most appropriate next step to assess her risk Ior Iracture?

O A) 24-Hour urine collection Ior measurement oI collagen cross-links excretion
O B) Bone densitometry
O C) Bone-speciIic measurement oI serum alkaline phosphatase activity
O D) Posteroanterior and lateral x-ray Iilms oI the thoracic spine and hips
O E) Biopsy oI the iliac crest


BBBBBBBBBBBBBBBBBBBBBBB



11. A 30-year-old woman, gravida 4, para 3, is admitted to the hospital in labor at 38
weeks' gestation. The cervix is 4 cm dilated. Contractions occur every4 minutes. The
Iundal height is 40 cm. The membranes are intact. The Ietal lie cannot be determined
by abdominal examination, and no presenting part is palpable in the pelvis. Which oI
the Iollowing is the most appropriate next step in management?

O A) Ultrasonography
O B) Administration oI oxytocin
O C) Administration oI tocolytic drugs
O D) Amniotomy
O E) Cesarean delivery


AAAAAAAAAAAAAAAAA


12. Six days aIter undergoing a laparoscopic cholecystectomy Ior acute cholecystitis,
a 35-year-old woman comes to the physician because oI Iever and abdominal pain Ior
3 days. She is jaundiced. Her temperature is 38C (100.4F). Abdominal examination
shows distention and incisions that are healing normally. Leukocyte count is
12,000/mm3, and total serum bilirubin level is 7.9 mg/dL. Which oI the Iollowing is
the most likely cause oI the jaundice?

O A) Anesthetic-related hepatitis
O B) Common bile duct injury
O C) Fulminant hepatic Iailure
O D) Reaction to perioperative antibiotics
O E) Subhepatic abscess

BBBBBBBBBBBBB


13. A 9-year-old boy is brought to the physician because oI progressive weakness and
a purple-red discoloration over his cheeks and upper eyelids over the past 8 weeks.
His symptoms began shortly aIter a camping trip, and he now is unable to climb
stairs, walk long distances, comb his hair, or dress himselI. His mother says that she
was careIul to apply his sunscreen on the trip and can recall no tick bites or exposure
to poisonous plants. His only medication is a topical corticosteroid Ior several dry,
scaly patches oI the skin. He appears weak and lethargic. He is at the 75th percentile
Ior height and 25th percentile Ior weight, he has had no change in his weight since his
last examination 9 months ago. His temperature is 37.7C (99.8F), blood pressure is
110/68 mm Hg, pulse is 105/min, and respirations are 28/min. Examination oI the
skin shows a purple-red discoloration over the cheeks and eyelids, periorbital edema,
erythematous plaques and scales over the elbows and knees, and Ilat-topped red
papules over all knuckles. There is generalized weakness and atrophy oI the proximal
muscles. Which oI the Iollowing is the most likely diagnosis2

O A) Dermatomyositis
O B) Duchenne's muscular dystrophy
O C) Eczema
O D) Lyme disease
O E) Psoriasis
O F) Rocky Mountain spotted Iever
O G) Seborrhea
0 H) Systemic lupus erythematosus


AAAAAAAAAAAAAAAA

14. An 82-year-old man is brought to the emergency department because oI the
sudden onset oI conIusion 48 hours ago. His Iamily says that he previously had been
Iunctional and independent. His temperature is 37.5C (99.5F), blood pressure is
110/70 mm Hg, pulse is 90/min, and respirations are 12/min. The skin is warm and
clammy. Bowel sounds are hypoactive, and there is guarding and tenderness in the
right lower quadrant oI the abdomen. Examination oI the heart, lungs, and extremities
shows no abnormalities. He is drowsy but arousable and oriented to person but not
place or time. There are no Iocal deIicits. Which oI the Iollowing is the most likely
diagnosis?
O A) Appendicitis
O B) Colon cancer
O C) Meningitis
O D) Pneumonia
0 E) Urinary tract inIection

AAAAAAAAAAAAAAAA

15. Two hours aIter emergency cholecystectomy, a 48-year-old woman has an oxygen
saturation oI 84 and a PO2 oI 56 mm Hg on 2 LImin oI oxygen via nasal cannula.
Her blood pressure is 120180 mm Hg, and respirations are 16/min. Decreased breath
sounds are heard on the right, and there is decreased excursion on inspiration
bilaterally. There is minimal dullness over the right base. Examination shows no
jugular venous distention or pedal edema. An x-ray Iilm oI the chest shows increased
density over the right lower lung Iield, the tracheal, mediastinal, and cardiac
silhouettes are shiIted to the right. The right lung Iield appears considerably smaller
than the leIt lung Iield. Which oI the Iollowing is the most likely cause oI these
Iindings?
O A) Atelectasis
O B) Pleural eIIusion
O C) Pneumonia
O D) Pneumothorax
0 E) Pulmonary embolism

AAAAAAAAAAAA



16. A 47-year-old man comes to the physician because oI low-grade Iever and malaise
Ior 3 months. He was inIected with hepatitis B 25 years ago. He appears well
nourished but ill. His temperature is 37.7C (99.8F), blood pressure is 110170 mm
Hg, pulse is 801min, and respirations are 121min. Examination shows slight jaundice
and hepatomegaly. The spleen is not palpable, and there is no ascites or dependent
edema. Serum studies show:

Total bilirubin 3 mg1dL
Alkaline phosphatase 160 U!L
Aspartate aminotransIerase (AST, GOT) 230 U!L
Alanine aminotransIerase (ALT, GPT) 275 U!L

Serum a-Ietoprotein level is markedly increased. Ultrasonography oI the right upper
quadrant oI the abdomen shows a 5-cm, solid lesion in the right lobe oI the liver.
Biopsy is most likely to show which oI the Iollowing?
O A) Focal nodular hyperplasia
O B) Hepatic adenoma
O C) Hepatocellular carcinoma
O D) Metastatic adenocarcinoma
0 E) Regenerating liver nodule


CCCCCCCCCCCCCCCCCC



17. A previously healthy 30-year-old woman has had a painless lump in her neck Ior 2
days. Her mother was treated Ior a thyroid tumor at the age oI 35 years. Her 28-year-
old sister has an increased serum calcitonin level but no thyroid mass. Examination
shows a palpable thyroid nodule. Her serum calcitonin level is increased. Which oI
the Iollowing diagnoses should be excluded prior to surgical treatment oI the thyroid
gland?
O A) Adrenocortical carcinoma
O B) Lung carcinoma
O C) Ovarian carcinoma
O D) Parathyroid carcinoma
O E) Pheochromocytoma

EEEEEEEEEEEEEEEEEEEEE



18. A 67-year-old man comes to the physician because oI a 1-month history oI
shortness oI breath with exertion, easy Iatigability, and mild ankle swelling. Five
years ago, he underwent successIul chemotherapy with doxorubicin Ior lymphoma.
Moist crackles are heard throughout both lungs. An S3 is heard. The liver edge is
palpated 4 cm below the right costal margin. There is 2 pitting edema oI the pretibial
region, ankles, and Ieet. Which oI the Iollowing is the most likely underlying cause oI
this patient's symptoms?
O A) Aortic sclerosis
O B) Cardiotoxicity
O C) Cirrhosis
O D) Mediastinal obstruction
0 E) Pneumonitis

BBBBBBBBBBBBBBBBBBBB

The response options Ior the next two items are the same. You will be required to
select one answer Ior each item in the set.

For each patient with vaginal bleeding during pregnancy, select the most likely
diagnosis.

O A)Abortion oI a blighted twin
O B)Cervicitis
O C)Ectopic pregnancy
O D)Focal decidual necrosis
O E) Gestational trophoblastic disease
0 F) Incomplete abortion
0 G) Ovarian torsion
0 H) Ruptured ovarian cyst
0 I) Threatened abortion


19. A 23-year-old woman, gravida 2, para 0, has had vaginal spotting and abdominal
cramps Ior 2 days. Her last menstrual period was 8 weeks ago. A home pregnancy test
was positive 2 weeks ago. She underwent a salpingectomy4 years ago Iollowing an
ectopic pregnancy. Examination shows a closed cervix, an enlarged uterus, and no
adnexal masses. Transvaginal ultrasonography shows an empty uterus. Serum 3-hCG
level is 8000 mIU1mL.

CCCCCCCCCCCCCCCC

For each patient with vaginal bleeding during pregnancy, select the most likely
diagnosis.

C A) Abortion oI a blighted twin 0 F) Incomplete abortion
C B) Cervicitis ) Ovarian torsion
0 C) Ectopic pregnancy H) Ruptured ovarian cyst
0 D) Focal decidual necrosis I) Threatened abortion
0 E) Gestational trophoblastic disease


20. A 25-year-old woman has had painless vaginal postcoital bleeding Ior 2 hours.
Her last menstrual period was 10 weeks ago. A pregnancy test is positive. There is a
small amount oI dark blood in the vaginal canal. She has a reddened, Iriable cervical
os.


IIIIIIIIIIIIIIIIIIIIIIII



21. A previously healthy 62-year-old man comes to the physician because oI a 1-year
history oI numbness and weakness oI the right hand. He is a carpenter and has no
history oI injury to his hand. Examination shows wasting oI muscle mass in the Iirst
web space. Sensation to touch is decreased along the ring and little Iingers. There is
weakness oI abduction and adduction oI the Iingers. Which oI the Iollowing is the
most likely site oI nerve injury?
O A) Axillary nerve
O B) Long thoracic nerve
O C) Median nerve above the elbow
O D) Median nerve at the elbow
O E) Median nerve at the wrist
O F) Musculocutaneous nerve above the elbow O G) Musculocutaneous nerve at the
elbow
O H) Musculocutaneous nerve at the wrist
O I) Radial nerve above the elbow
O J) Radial nerve at the elbow
O K) Radial nerve at the wrist
O L) Suprascapular nerve
O M) Thoracodorsal nerve
0 N) Ulnar nerve at the elbow

NNNNNNNNNNNNNNNNNNNNN

22. A 45-year-old man comes to the physician Ior a Iollow-up visit. He has had
recurrent ulcer disease Ior 8 years, his symptoms are currently relieved with antacid
therapy. He had one episode oI bleeding 1 year ago that required a blood transIusion.
He appears pale. His hemoglobin level is 10.6 g/dL, mean corpuscular volume is 78
pm3, and reticulocyte count is 0.8. Which oI the Iollowing additional laboratory
Iindings is most likely?

Serum Iron Iron-binding Capacity (TIBC) Saturation oI Serum Ferritin
(pg/dL) (pg/dL) (N250-400) TIBC () (ng/mL)
O A) 30 120 25 50
O B) 40 360 11 10
O C) 60 180 33 80
O D) 80 200 40 120
0 E) 100 150 67 600

BBBBBBBBBBBBBBB




23. A 6-year-old girl is brought to the physician because oI a 1-day history oI
vomiting, headache, and weakness. One month ago, she underwent resection oI an
astrocytoma and placement oI a ventriculoatrial shunt Ior residual hydrocephalus, her
postoperative course had been uncomplicated. Currently, she is drowsy and irritable.
Her temperature is 37.5C (99.5F), blood pressure is 126/54 mm Hg, pulse is 82/min,
and respirations are 24/min. Funduscopic examination shows papilledema. ReIlexes
are brisk with hypertonia in the lower extremities.

Which oI the Iollowing is the most likely underlying mechanism Ior these symptoms?

O A) Carcinomatous inIiltration oI cerebrospinal Iluid
O B) MalIunction oI ventriculoatrial shunt
O C) Overproduction oI cerebrospinal Iluid Irom a choroid plexus papilloma
O D) Recurrence and extension oI the astrocytoma
O E) Thrombosis oI the superior sagittal sinus



BBBBBBBBBBBB

The response options Ior the next three items are the same. You will be required to
select one answer Ior each item in the set.

For each patient with a respiratory problem, select the most appropriate next step in
management.

O A)Administration oI BCG vaccine
O B)Administration oI an inhaled bronchodilator
O C)Administration oI oxygen
O D)Arterial blood gas analysis
O E)Bronchoscopy
O F)Culture oI the pharynx Ior bacteria
O G)Direct laryngoscopy
O H) Gastric washings Ior acid-Iast bacteria
O I) Intravenous inIusion oI saline
O J) Isoniazid and riIampin therapy O K) Lumbar puncture
O L) Measurement oI serum aspirin level
O M) Subcutaneous administration oI epinephrine.
N) Viral culture oI respiratory secretions
00) X-ray Iilms oI the chest
O P) X-ray Iilms oI the neck


24. A previously healthy 8-year-old boy is brought to the emergency department
because oI swelling oI the lips and diIIiculty breathing Ior 20 minutes. The symptoms
began when he was helping his Iather clean the gutters on their house. He appears
anxious. His temperature is 37.2C (99F), pulse is 1201min, and respirations are
50/min. Pulse oximetry shows an oxygen saturation oI 96. His lips and eyes appear
puIIy. He has subcostal and intercostal retractions. Auscultation oI the chest shows
diIIuse bilateral wheezing.


MMMMMMMMMMMMMMMMMM



For each patient with a respiratory problem, select the most appropriate next step in
management.

O A)Administration oI BCG vaccine 0 I) Intravenous inIusion oI saline
O B)Administration oI an inhaled bronchodilator O J) Isoniazid and riIampin therapy
Oc)Administration oI oxygen O K) Lumbar puncture
O D)Arterial blood gas analysis O L) Measurement oI serum aspirin level
O E)Bronchoscopy O M) Subcutaneous administration oI epinephrine
O F)Culture oI the pharynx Ior bacteria O N) Viral culture oI respiratory secretions
O G)Direct laryngoscopy 00) X-ray Iilms oI the chest
O H)Gastric washings Ior acid-Iast bacteria O P) X-ray Iilms oI the neck


25. A 16-year-old girl is brought to the emergency department because oI heavy
breathing Ior 8 hours, and vomiting and ringing in the ears Ior 1 hour. She recently
broke up with her boyIriend and has been threatening to hurt herselI. She appears
somnolent but is arousable and answers questions appropriately. Her temperature is
37.5C (99.5F), pulse is 88/min, and respirations are 50/min. Pulse oximetry shows
an oxygen saturation oI 96. She has no retractions, nasal Ilaring, or cough. The
lungs are clear to auscultation.


L) Measurement oI serum aspirin level



For each patient with a respiratory problem, select the most appropriate next step in
management.

0 A)Administration oI BCG vaccine 0 I) Intravenous inIusion oI saline
0 B)Administration oI an inhaled bronchodilator 0 J) Isoniazid and riIampin therapy
0 C)Administration oI oxygen 0 K) Lumbar puncture
0 D)Arterial blood gas analysis O L) Measurement oI serum aspirin level
0 E)Bronchoscopy 0 M) Subcutaneous administration oI epinephrine
0 F)Culture oI the pharynx Ior bacteria 0 N) Viral culture oI respiratory secretions
O G)Direct laryngoscopy O) X-ray Iilms oI the chest
O H)Gastric washings Ior acid-Iast bacteria 0 P) X-ray Iilms oI the neck


26. A previously healthy 6-year-old boy is brought to the physician Ior a Iollow-up
examination aIter a PPD skin test produced an induration greater than 15 mm. He has
had no cough or Iever. He has a good appetite and is at the 50th percentile Ior height
and weight. He appears well. His temperature is 37.2C (99F), pulse is 88/min, and
respirations are 18/min. The lungs are clear to
auscultation.

OOOOOOOOOOOOOOOOOOOO



27. A 25-year-old nulligravid woman comes to the emergency department because oI
severe pain in the right lower quadrant oI the abdomen Ior 4 hours. She has had no
nausea, vomiting, Iever, or chills. Three weeks ago, a right adnexal mass was Iound
on routine examination. Her last menstrual period was 3 days ago. Her temperature is
37.2C (99F), blood pressure is 110/70 mm Hg, and pulse is 92/min. Pelvic
examination shows right adnexal tenderness. Hemoglobin level is 13 g/dL, and
leukocyte count is 9000/mm3. Pelvic ultrasonography shows a small amount oI Iree
Iluid in the cul-de-sac. Which oI the Iollowing is the most likely cause oI the pain?
O A) Appendicitis
O B) Endometriosis
O C) Ovarian hemorrhage
O D) Ruptured ovarian cyst
O E) Torsion oI the adnexa
0 F) Tubal obstruction

DDDDDDDDDDDDDDDDDDDDD

28. A 15-year-old boy comes to the physician because oI acne over his Iace Ior 1 year.
He preIers not to use any oral medications. Examination shows 10 to 15 pustules.
Which oI the Iollowing is the most eIIective treatment Ior this patient's symptoms?

p A) Avoidance oI chocolate, Iatty Ioods, and caIIeine
p B) Use oI benzoyl peroxide soap
O C) Vigorous scrubbing oI aIIected areas
p D) Application oI vitamin A and vitamin E to aIIected areas
p E) Ultraviolet light therapy

BBBBBBBBBBBBBBBBBBBBBBBBBB


29. A 3-year-old boy with acute lymphoblastic leukemia has had Iever Ior 3 days. He
completed his last course oI chemotherapy 6 days ago. He has had no malaise, rash, or
anorexia and has had no known contact with sick children in preschool. He appears
well. His temperature is 38.6C (101.5F), blood pressure is 75160 mm Hg, pulse is
100/min, and respirations are 22/min. Examination shows normal Iindings.
Laboratory studies show:
Hemoglobin 10.1 g/dL (N11.5-15.5)
Leukocyte count 2200/mm3
Segmented neutrophils 5
Bands 1
Lymphocytes 65
Atypical lymphocytes 11
Monocytes 18
Platelet count 35,000/mm3

Which oI the Iollowing is the most appropriate next step in management?

O A) Schedule a Iollow-up visit and discharge without medication
O B) Observe him in the oIIice Ior 3 hours, discharge without medication iI
examination remains unchanged
O C) Discharge him with oral broad-spectrum antibiotic therapy
O D) Admit him to the hospital Ior observation
O E) Admit him to the hospital Ior intravenous broad-spectrum antibiotic therapy

EEEEEEEEEEEEEEEEEEEEEE



30. A 72-year-old woman is brought to the emergency department 1 hour aIter the
sudden onset oI right Iacial droop and weakness oI the right arm and leg. She takes
captopril Ior hypertension and daily aspirin. Her blood pressure is 150/90 mm Hg,
pulse is 80/min, and respirations are 161min. Examination shows a leIt carotid bruit
and right central Iacial paralysis. There is moderate expressive aphasia. A CT scan oI
the head shows no abnormalities. Which oI the Iollowing is the most appropriate
initial pharmacotherapy?
O A) Oral ticlopidine
O B) Oral warIarin
O C) Sublingual niIedipine
O D) Intravenous nitroprusside
O E) Intravenous tissue plasminogen activator




EEEEEEEEEEEEEEEEEEEEEE/




31. A previously healthy 37-year-old woman comes to the physician because oI a 3-
month history oI episodes oI severe anxiety, shortness oI breath, palpitations, and
numbness in her hands and Ieet. Her vital signs are within normal limits. Physical
examination shows no abnormalities. Thyroid Iunction studies and an ECG show no
abnormalities. Which oI the Iollowing is the most appropriate pharmacotherapy?
A) Lithium carbonate
B) Methylphenidate
O C) Olanzapine
p D) Paroxetine
0 E) Valproic acid

DDDDDDDDDDDDDD



32. A 37-year-old nulligravid woman comes to the physician because she has not been
able to conceive Ior 2 years. Her 40-year-old husband has a child by a previous
marriage. Her last menstrual period was 6 weeks ago, menses have occurred at
increasingly inIrequent intervals over the past year. FiIteen years ago, she was treated
Ior one episode oI Chlamydia trachomatis inIection. She weighs 59 kg (130 lb) and is
170 cm (67 in) tall. Examination shows no abnormalities. Serum studies show:

R-hCG 5 mlUImL
Follicle-stimulating hormone 50 mlU1mL
Luteinizing hormone 45 mlUImL
Prolactin 13 ngImL
Thyroid-stimulating hormone 3 pU/mL

Which oI the Iollowing is the most likely cause oI this patient's inIertility?

O A) Hypothalamic amenorrhea
O B) Hypothyroidism
O C) Pituitary adenoma
O D) Polycystic ovarian disease
O E) Premature ovarian Iailure
0 F) Tubal Iactor


EEEEEEEEEEEEEEEEE

33. A 47-year-old woman comes to the physician because oI a 2-day history oI Iever
and joint pain. Six days ago, she completed a 10-day course oI dicloxacillin Ior
Iolliculitis. She appears comIortable. Her temperature is 38.6C (101.5E), pulse is
721min and regular, respirations are 161min, and blood pressure is 120176 mm Hg.
Examination shows an urticarial rash over the trunk and extremities. There is
moderate generalized lymphadenopathy and diIIuse joint tenderness. The remainder
oI the examination shows no abnormalities. Which oI the Iollowing is the most likely
cause oI these symptoms?
O A) Epstein-Barr virus inIection
O B) IgE-mediated allergic reaction
O C) Mycobacterium haemophilum inIection
O D) Serum sickness
0 E) Staphylococcal sepsis

DDDDDDDDDDDDDDDDDDDD


34. A 34-year-old man is brought to the emergency department by his Iamily because
oI a decreased need Ior sleep, increased goal-directed activity, increasingly irritable
and unstable mood, and uncharacteristic sexual promiscuity over the past week.
Yesterday, he quit his long-standing job at a hardware store and plans to move to
Washington, DC, to become the "Senator-in-ChieI." He says that this position is being
created especially Ior him by the president. He believes that he has developed
supersensitive hearing and is now able to hear conversations oI people hundreds oI
miles away. He has no history oI drug use, and there is no personal or Iamily history
oI psychiatric illness. On examination, he is extremely irritable and agitated,
threatening to "punch out anyone who tries to mess with me." Which oI the Iollowing
is the most likely diagnosis?
O A) Bipolar disorder
O B) Cyclothymic disorder
O C) Delusional disorder
O D) Schizophrenia
O E) SchizophreniIorm disorder

AAAAAAAAAAAAAAAAAAAAAAAA

.


35. A 17-year-old girl has had Ilu-like symptoms, low-grade Iever, and malaise Ior 3
days and mild jaundice Ior 2 days. Serum studies show an aspartate aminotransIerase
(AST, GOT) activity oI 670 UIL and an alanine aminotransIerase (ALT. GPT)
activity oI 860 U/L. Serum IgM antibody to hepatitis A is positive. Which oI the
Iollowing is most likely to minimize the risk Ior this disease in Iamily members?
O A) Acyclovir therapy Ior Iamily members
O B) Hepatitis B vaccination Ior Iamily members
O C) Immune globulin therapy Ior Iamily members
OD) InterIeron alIa-2b therapy Ior Iamily members
0 E) Use oI separate toilet Iacilities by the patient

CCCCCCCCCCCCCCCCC


36. One day aIter missing her dialysis treatment, a 27-year-old woman comes to the
physician because oI generalized weakness. Her blood pressure is 95172 mm Hg, and
pulse is 45/min and regular. Examination shows a well-Iunctioning arteriovenous
Iistula in the leIt upper extremity. Muscle strength is 315 in all extremities. An ECG
shows third-degree atrioventricular block. Laboratory studies are ordered. Which oI
the Iollowing is the most appropriate next step in management?
O A) Kayexalate-sorbitol enema
O B) Intravenous administration oI calcium gluconate
O C) Oral administration oI sodium polystyrene sulIonate (Kayexalate)
O D) Immediate placement oI a pacemaker
O E) No treatment pending laboratory results


DDDDDDDDDDDDDDDDDD

37. A previously healthy 2-year-old boy is brought to the physician because oI Iever
and abdominal pain Ior 24 hours. His developmental milestones are appropriate Ior
age. He is Iully alert and responsive. His temperature is 38.6C (101.5F), blood
pressure is 85/60 mm Hg, pulse is 1001min, and respirations are 20/min. Examination
shows suprapubic tenderness to deep palpation but no guarding or rebound. There are
no palpable abdominal masses or costovertebral angle tenderness. The penis is
circumcised, there is no urethral discharge. Urinalysis shows 20-30 leukocytes/hpI, 5-
6 erythrocytes/hpI, and nitrites. A urine culture grows 100,000 colonies/mL oI
Escherichia coli sensitive to all tested antibiotics. Amoxicillin therapy is initiated.
Five days later, he is aIebrile and asymptomatic. Ultrasonography oI the kidneys
shows no abnormalities. Which oI the Iollowing is the most appropriate next step in
management?

O A) Discontinue the antibiotic in 2 days and reexamine only iI symptoms recur
O B) Voiding cystourethrography
O C) Intravenous pyelography
O D) Cystoscopy
O E) No Iurther testing


BBBBBBBBBBBBBBBBBBBBBBB

38. A 62-year-old woman comes to the physician because oI increasingly severe low
back pain over the past month. Prolonged periods oI rest have not relieved her
symptoms. Sensorimotor examination oI the lower extremities shows no
abnormalities. Her hematocrit is 32, and serum calcium level is 12 mgIdL. An x-ray
Iilm oI the chest shows no abnormalities. An x-ray Iilm oI the spine is shown. Which
oI the Iollowing is the most likely diagnosis?
O A) Herniated nucleus pulposus
O B) Mechanical low back pain
O C) Metastatic carcinoma
O D) Osteopetrosis
0 E) Spinal stenosis


ccccccccccccccccc
O
Permalink Reply by usmle99 on March 13, 2009 at 7:26am
39. A 43-year-old man comes to the physician Ior evaluation and management oI
cardiac risk Iactors 8 weeks aIter sustaining a myocardial inIarction. He takes aspirin
and metoprolol daily, and he does not smoke cigarettes. His Iather and brother both
had myocardial inIarctions beIore the age oI 50 years, their serum cholesterol levels
are unknown. There is no Iamily history oI diabetes mellitus. He weighs 86 kg (190
lb) and is 180 cm (71 in) tall. His blood pressure is 130170 mm Hg, pulse is 68/min,
and respirations are 14/min. Two years ago, his serum cholesterol level was 245
mg/dL. Fasting serum glucose level is 88 mg/dL. Which oI the Iollowing is the most
appropriate next step to evaluate his cardiac risk Iactors?


O A) Random measurements oI serum cholesterol level
O B) Measurement oI Iasting serum cholesterol level only
O C) Fasting serum lipid studies only
O D) Oral glucose tolerance test and Iasting serum lipid studies
O E) Oral glucose tolerance test and measurement oI Iasting serum cholesterol level

CCCCCCCCCCCCCCCCCCCC



40. A 47-year-old man comes to the physician because oI a 3-week history oI
increasing Iacial swelling and a 1-week history oI morning headaches and mild
shortness oI breath. He had previously Ielt well. He completed a course oI
chemotherapy 4 months ago Ior small cell carcinoma oI the lung. His temperature is
37.2C (98.9F), blood pressure is 142180 mm Hg with an 8-mm Hg paradoxical
pulse, pulse is 84/min, and respirations are 18/min. Examination shows signiIicant
diIIuse Iacial and periorbital edema. The optic discs are sharp, and ocular movements
are intact. The lungs are hyperresonant bilaterally with a moderately prolonged
expiratory phase. Mild rhonchi are heard on inspiration and expiration. An x-ray Iilm
oI the chest shows a 10-cm mass in the right upper lobe and apex. Which
oI the Iollowing is the most likely explanation Ior these Iindings?
O A) Chemotherapy-induced bone marrow toxicity
O B) Chemotherapy-related cardiac toxicity
O C) Hypercoagulable state secondary to malignancy
O D) Interstitial metastatic pulmonary disease
O E) Intracranial metastases
O F) Lymphatic obstruction
O G) Malignant pericarditis
O H) Paraneoplastic syndrome
O I) Pleural metastases
O J) SoIt tissue metastases
0 K) Vascular obstruction


KKKKKKKKKKKKKKKKK


41. A 13-month-old girl is brought Ior a well-child examination. There is no history oI
prenatal or perinatal problems. Her diet consists oI breast milk, juice, legumes,
cooked vegetables, and Iruit. She has had three ear inIections treated successIully with
amoxicillin and two episodes oI diarrhea since birth. She will pull to a stand and stand
alone Ior 1 minute but has not begun to walk. She is able to pick up raisins with her
thumb and IoreIinger but is unable to Ieed herselI with a spoon. Which oI the
Iollowing is the most appropriate
assessment oI Iine and gross motor development?

`` Fine Motor Gross Motor
Development Development
O A) Normal normal
O B) Normal delayed
O C) Delayed normal
O D) Delayed delayed

AAAAAAAAAAAAAAAAAAA

42. An 18-month-old girl is brought to the physician because oI intermittent shortness
oI breath over the past 6 months. Her mother notes that her daughter oIten squats to
relieve her symptoms. Examination shows cyanosis and clubbing oI the Iingers. A
prominent right ventricular impulse with a grade 3/6, systolic ejection murmur is
heard best in the third leIt intercostal space. An ECG shows right ventricular
hypertrophy. An x-ray Iilm oI the chest shows a small-sized heart and decreased
pulmonary blood Ilow. Which oI the Iollowing is the most likely diagnosis?
O A) Atrial septal deIect (ostium primum type)
O B) Atrial septal deIect (ostium secundum type)
O C) Atrioventricular canal
O D) Coarctation oI the aorta
O E) Hypoplastic leIt heart syndrome
O F) Patent ductus arteriosus
O G) Tetralogy oI Fallot
O H) Transposition oI the great arteries
O I) Tricuspid atresia
0 J) Ventricular septal deIect

GGGGGGGGGGGG


43. A 5-day-old girl has been Ieeding poorly since birth. She weighed 2900 g (6 lb 7
oz) at birth and now weighs 2600 g (5 lb 12 oz). Physical examination shows an
enlarged clitoris and labial Iusion. A sibling born 5 years ago died at 1 week oI age. A
liIe-threatening complication oI this syndrome results Irom Iailure to produce which
oI the Iollowing?
O A) Aldosterone
O B) Cholesterol
O C) Estrogen
O D) Insulin
0 E) Testosterone


AAAAAAAAAAAAAAAAAAA

44. A 57-year-old hospitalized man undergoes right subclavian venous catheterization
Ior hyperalimentation. He is currently being treated Ior a small bowel Iistula. While
the results oI an x-ray Iilm oI the chest to check the catheter position are pending, the
patient suddenly becomes agitated. His blood pressure is 70150 mm Hg, and pulse is
1101min. Examination shows jugular venous distention. The lungs are clear to
auscultation. Breath sounds are equal bilaterally. The trachea is midline. An x-ray
Iilm oI the chest shows a catheter in the superior vena cava, transversing the right
ventricle and crossing to the leIt oI the midline. Which oI the Iollowing is the most
likely cause oI this patient's hypotension?
O A) Air embolism
O B) Pericardial tamponade
O C) Pulmonary artery perIoration
O D) Staphylococcal bacteremia
O E) Tension pneumothorax

BBBBBBBBBBBBBBBBBBBBBB

45. Twenty-Iour hours aIter hospitalization Ior Iixation oI a Iemoral Iracture, a 35-
year-old construction worker has nausea, agitation, and insomnia. His blood pressure
is 1501100 mm Hg. He is oriented but anxious. Which oI the Iollowing is the most
appropriate next step in diagnosis?
O A) Inquire about recent alcohol intake
O B) Examination oI a urine specimen Ior Iat bodies
O C) Leukocyte count with diIIerential
O D) ECG
0 E) Pulmonary arteriography

AAAAAAAAAAAAAAAAAAA

46. A previously healthy 27-year-old woman comes to the physician because oI a 3-
month history oI moderate abdominal pain that improves Ior a short time aIter she
eats. She has not had any rectal bleeding. Her temperature is 36.4C (97.5F), blood
pressure is 110170 mm Hg, and pulse is 80/min. Abdominal examination shows
midepigastric tenderness. Her hemoglobin level is 12 gIdL, and leukocyte count is
80001mm3. Serum Helicobacterpylori antibody assay is positive. Which oI the
Iollowing is the most appropriate next step in management?
O A) HIDA scan
O B) Amoxicillin, clarithromycin, and omeprazole therapy
O C) CiproIloxacin therapy
O D) Omeprazole, magnesium hydroxide, and metoclopramide therapy O E)
Ranitidine therapy


BBBBBBBBBBBBBBBBBBBB

nbme3 sec2
#223301
quansar - 09/25/07 00:43

here you go the section 2, I will work on the sec3/4 in next Iew days




1. For the past 10 years, a 28-year-old woman has Iollowed a strict diet that prohibits
dairy products, meat, Iish, and Iowl. On routine examination, her hematocrit is 30,
and mean corpuscular volume is 122 pm3. Which oI the Iollowing is the most
appropriate next step in diagnosis?

O A) Erythrocyte Iolate determination
O B) Serum iron and total iron-binding capacity determinations
O C) Serum thyroid hormone determination
O D) Serum urea nitrogen (BUN) determination
O E) Serum vitamin B12 (cyanocobalamin) determination

EEEEEEEEEEEEEEE


2. A 47-year-old man comes to the physician because oI a persistent nonproductive
cough Ior 3 months. He has not had Iever, rhinorrhea, or nasal congestion. He was
seen by a physician 3 months ago Ior the cough and treated with amoxicillin and an
over-the-counter cough medication. He has a 3-month history oI hypertension well
controlled with lisinopril. He is in no distress. He weighs 79 kg (175 lb) and is 178 cm
(70 in) tall. His temperature is 37.1C (98.8F), blood pressure is 130178 mm Hg,
pulse is 721min and regular, and respirations are 161min. Examination shows no
abnormalities. A complete blood count and x-ray Iilms oI the chest show no
abnormalities. Which oI the Iollowing is the most appropriate next step in
management?

O A) X-ray Iilms oI the sinuses
O B) Discontinue lisinopril therapy
O Q Add a beclomethasone inhaler to the medication regimen
O D) Add ciproIloxacin to the medication regimen
O E) Add hydrocodone to the medication regimen


BBBBBBBBBBBBBBBBBBBBB

3. A 12-year-old girl is brought to the physician by her mother because she is
concerned about her daughter's diIIiculty making Iriends and socializing. Her
development and adjustment had been normal until 6 months ago when she began to
reIuse to use restrooms at school or eat in the caIeteria. Her mother describes her as a
quiet, serious child who does not readily interact with other people. Her teachers
report that her concentration varies. at times, she appears to be daydreaming. On
examination, she is reserved but pleasant and appears to be oI normal intelligence.
Her speech is normal in rate and rhythm. She says that she is concerned that her voice
will Iail her iI she has to read aloud in class. Which oI the Iollowing is the most likely
diagnosis?

O A) Attention-deIicitthyperactivity disorder, inattentive type
O B) Autistic disorder
O C) Expressive language disorder
O D) Oppositional deIiant disorder
O E) Selective mutism
O F) Social phobia
O G) Age-appropriate behavior

FFFFFFFFFFFFFFFFFF



A picture here Ior q4

4. A 25-year-old woman is brought to the emergency department 45 minutes aIter
being hit in the right eye with a tennis ball. She has severe pain and decreased vision
in the injured eye. Visual acuity is 201400 on the right. A photograph oI the eye is
shown. Which oI the Iollowing is the most likely diagnosis?
O A) Acute angle-closure glaucoma
O B) Anterior iritis
O C) Cataract
O D) Hyphema
O E) Hypopyon

DDDDDDDDDDDDDDD

5. A 44-year-old woman is brought to the emergency department 40 minutes aIter
being stabbed in the leIt groin. Her blood pressure is 128/84 mm Hg, pulse is
100/min, and respirations are 16/min. Examination oI the wound shows a small
hematoma and no external bleeding. Popliteal and pedal pulses are palpable in the leIt
lower extremity below the injury. Lacerations oI the Iemoral artery and vein are Iound
and repaired. Two days postoperatively, she has progressively severe pain oI the leIt
lower extremity and swelling oI the leg Irom the knee down. Which oI the Iollowing
is the most likely cause oI her new symptoms?

O A) Arterial embolism
O B) Compartment syndrome
O C) Lymphatic disruption Irom the injury
O D) Thrombosis oI the Iemoral vein
0 E) Wound hemorrhage

BBBBBBBBBBBBBB


6. The crude mortality rate Ior coronary artery disease in Community A is twice the
crude mortality rate Ior coronary artery disease in Community B. The age-
standardized mortality rates Ior coronary artery disease in the two communities are
the same. These Iindings are most consistent with which oI the Iollowing inIerences?

A) Coronary artery disease mortality in the elderly is higher in Community A than
Community B
B) Coronary artery disease mortality in young adults is higher in Community A than
Community B
C) The population oI Community B is younger than the population oI Community A
D) The proportionate mortality Irom coronary artery disease is higher in Community
A than Community B
E) The two communities have similar age distributions

CCCCCCCCCCCCCC

7. An otherwise healthy 28-year-old man comes to the physician because oI a dry
hacking cough Ior 3 months. A routine examination 6 months ago showed no
abnormalities. His paternal grandIather had colon cancer. Examination today shows
normal Iindings. An x-ray Iilm oI the chest shows a large anterior mediastinal mass.
Which oI the Iollowing is the most likely origin oI this neoplasm?
O A) Colon
O B) Kidney
O C) Prostate
O D) Stomach
0 E) Testicle

EEEEEEEEEEEEEEEEEE

A testis tumor may become metastatic and maniIest with large retroperitoneal and/or
chest lesions, while the primary tumor is nonpalpable. Scrotal ultrasonography may
locate the primary tumor. Histopathology oI the primary testis oIten shows a Iocus oI
tumor surrounded by Iibrous scar, termed burned-out testis cancer.

8. A 50-year-old man has a cardiac arrest during sexual intercourse. He is promptly
resuscitated and recovers uneventIully Irom an acute anterior wall myocardial
inIarction. At discharge Irom the hospital, he and his wiIe inquire about the saIety oI
Iuture sexual intercourse. Which oI the Iollowing is the most appropriate advice?
O A) Psychiatric consultation is necessary
O B) Sexual intercourse should be perIormed while wearing an ambulatory ECG
monitor
O C) Sexual intercourse will be saIe when exercise tolerance improves
O D) They should wait 3 months beIore having sexual intercourse
O E) They should have sexual intercourse as soon as possible to overcome the Iear oI
another event



CCCCCCCCCCCCCCCCCC



9. A 68-year-old woman with terminal metastatic breast cancer is living at home with
her son. She has a living will requesting that she be allowed to "die in peace." She is
unresponsive to voice and has not had any Iood or liquids Ior 3 days. Her son
disagrees with her decision not to accept Iurther therapy, including chemotherapy,
antibiotics, hospitalization, and enteral or parenteral nutrition. Which oI the Iollowing
is the most appropriate next step in management?
p A) Abide by the decisions oI the next oI kin
p B) Obtain a court order to allow additional therapy to be given
O C) Start enteral Ieedings
p D) Admit her to the hospital
0 E) No Iurther intervention


EEEEEEEEEEEEEEEEEEE


10. A 57-year-old woman comes to the physician Ior a Iollow-up examination. She
has a 5-year history oI hypercalcemia, which was diagnosed with routine laboratory
studies, and her serum calcium levels have ranged Irom 10.8 mg/dL to 11.5 mg/dL.
She declined Iurther evaluation in the past because she "Ielt well." She takes no
medications. Her last menstrual period was 7 years ago. She maintains a weight oI 67
kg (148 lb) and is 170 cm (67 in) tall, BMI is 23 kg/m2. Her blood pressure is 126/80
mm Hg, and pulse is 66/min. Examination shows no abnormalities. Which oI the
Iollowing is the most appropriate next step to assess her risk Ior Iracture?

O A) 24-Hour urine collection Ior measurement oI collagen cross-links excretion
O B) Bone densitometry
O C) Bone-speciIic measurement oI serum alkaline phosphatase activity
O D) Posteroanterior and lateral x-ray Iilms oI the thoracic spine and hips
O E) Biopsy oI the iliac crest


BBBBBBBBBBBBBBBBBBBBBBB



11. A 30-year-old woman, gravida 4, para 3, is admitted to the hospital in labor at 38
weeks' gestation. The cervix is 4 cm dilated. Contractions occur every4 minutes. The
Iundal height is 40 cm. The membranes are intact. The Ietal lie cannot be determined
by abdominal examination, and no presenting part is palpable in the pelvis. Which oI
the Iollowing is the most appropriate next step in management?

O A) Ultrasonography
O B) Administration oI oxytocin
O C) Administration oI tocolytic drugs
O D) Amniotomy
O E) Cesarean delivery


AAAAAAAAAAAAAAAAA


12. Six days aIter undergoing a laparoscopic cholecystectomy Ior acute cholecystitis,
a 35-year-old woman comes to the physician because oI Iever and abdominal pain Ior
3 days. She is jaundiced. Her temperature is 38C (100.4F). Abdominal examination
shows distention and incisions that are healing normally. Leukocyte count is
12,000/mm3, and total serum bilirubin level is 7.9 mg/dL. Which oI the Iollowing is
the most likely cause oI the jaundice?

O A) Anesthetic-related hepatitis
O B) Common bile duct injury
O C) Fulminant hepatic Iailure
O D) Reaction to perioperative antibiotics
O E) Subhepatic abscess

BBBBBBBBBBBBB


13. A 9-year-old boy is brought to the physician because oI progressive weakness and
a purple-red discoloration over his cheeks and upper eyelids over the past 8 weeks.
His symptoms began shortly aIter a camping trip, and he now is unable to climb
stairs, walk long distances, comb his hair, or dress himselI. His mother says that she
was careIul to apply his sunscreen on the trip and can recall no tick bites or exposure
to poisonous plants. His only medication is a topical corticosteroid Ior several dry,
scaly patches oI the skin. He appears weak and lethargic. He is at the 75th percentile
Ior height and 25th percentile Ior weight, he has had no change in his weight since his
last examination 9 months ago. His temperature is 37.7C (99.8F), blood pressure is
110/68 mm Hg, pulse is 105/min, and respirations are 28/min. Examination oI the
skin shows a purple-red discoloration over the cheeks and eyelids, periorbital edema,
erythematous plaques and scales over the elbows and knees, and Ilat-topped red
papules over all knuckles. There is generalized weakness and atrophy oI the proximal
muscles. Which oI the Iollowing is the most likely diagnosis2

O A) Dermatomyositis
O B) Duchenne's muscular dystrophy
O C) Eczema
O D) Lyme disease
O E) Psoriasis
O F) Rocky Mountain spotted Iever
O G) Seborrhea
0 H) Systemic lupus erythematosus


AAAAAAAAAAAAAAAA

14. An 82-year-old man is brought to the emergency department because oI the
sudden onset oI conIusion 48 hours ago. His Iamily says that he previously had been
Iunctional and independent. His temperature is 37.5C (99.5F), blood pressure is
110/70 mm Hg, pulse is 90/min, and respirations are 12/min. The skin is warm and
clammy. Bowel sounds are hypoactive, and there is guarding and tenderness in the
right lower quadrant oI the abdomen. Examination oI the heart, lungs, and extremities
shows no abnormalities. He is drowsy but arousable and oriented to person but not
place or time. There are no Iocal deIicits. Which oI the Iollowing is the most likely
diagnosis?
O A) Appendicitis
O B) Colon cancer
O C) Meningitis
O D) Pneumonia
0 E) Urinary tract inIection

AAAAAAAAAAAAAAAA

15. Two hours aIter emergency cholecystectomy, a 48-year-old woman has an oxygen
saturation oI 84 and a PO2 oI 56 mm Hg on 2 LImin oI oxygen via nasal cannula.
Her blood pressure is 120180 mm Hg, and respirations are 16/min. Decreased breath
sounds are heard on the right, and there is decreased excursion on inspiration
bilaterally. There is minimal dullness over the right base. Examination shows no
jugular venous distention or pedal edema. An x-ray Iilm oI the chest shows increased
density over the right lower lung Iield, the tracheal, mediastinal, and cardiac
silhouettes are shiIted to the right. The right lung Iield appears considerably smaller
than the leIt lung Iield. Which oI the Iollowing is the most likely cause oI these
Iindings?
O A) Atelectasis
O B) Pleural eIIusion
O C) Pneumonia
O D) Pneumothorax
0 E) Pulmonary embolism

AAAAAAAAAAAA



16. A 47-year-old man comes to the physician because oI low-grade Iever and malaise
Ior 3 months. He was inIected with hepatitis B 25 years ago. He appears well
nourished but ill. His temperature is 37.7C (99.8F), blood pressure is 110170 mm
Hg, pulse is 801min, and respirations are 121min. Examination shows slight jaundice
and hepatomegaly. The spleen is not palpable, and there is no ascites or dependent
edema. Serum studies show:

Total bilirubin 3 mg1dL
Alkaline phosphatase 160 U!L
Aspartate aminotransIerase (AST, GOT) 230 U!L
Alanine aminotransIerase (ALT, GPT) 275 U!L

Serum a-Ietoprotein level is markedly increased. Ultrasonography oI the right upper
quadrant oI the abdomen shows a 5-cm, solid lesion in the right lobe oI the liver.
Biopsy is most likely to show which oI the Iollowing?
O A) Focal nodular hyperplasia
O B) Hepatic adenoma
O C) Hepatocellular carcinoma
O D) Metastatic adenocarcinoma
0 E) Regenerating liver nodule


CCCCCCCCCCCCCCCCCC



17. A previously healthy 30-year-old woman has had a painless lump in her neck Ior 2
days. Her mother was treated Ior a thyroid tumor at the age oI 35 years. Her 28-year-
old sister has an increased serum calcitonin level but no thyroid mass. Examination
shows a palpable thyroid nodule. Her serum calcitonin level is increased. Which oI
the Iollowing diagnoses should be excluded prior to surgical treatment oI the thyroid
gland?
O A) Adrenocortical carcinoma
O B) Lung carcinoma
O C) Ovarian carcinoma
O D) Parathyroid carcinoma
O E) Pheochromocytoma

EEEEEEEEEEEEEEEEEEEEE



18. A 67-year-old man comes to the physician because oI a 1-month history oI
shortness oI breath with exertion, easy Iatigability, and mild ankle swelling. Five
years ago, he underwent successIul chemotherapy with doxorubicin Ior lymphoma.
Moist crackles are heard throughout both lungs. An S3 is heard. The liver edge is
palpated 4 cm below the right costal margin. There is 2 pitting edema oI the pretibial
region, ankles, and Ieet. Which oI the Iollowing is the most likely underlying cause oI
this patient's symptoms?
O A) Aortic sclerosis
O B) Cardiotoxicity
O C) Cirrhosis
O D) Mediastinal obstruction
0 E) Pneumonitis

BBBBBBBBBBBBBBBBBBBB

The response options Ior the next two items are the same. You will be required to
select one answer Ior each item in the set.

For each patient with vaginal bleeding during pregnancy, select the most likely
diagnosis.

O A)Abortion oI a blighted twin
O B)Cervicitis
O C)Ectopic pregnancy
O D)Focal decidual necrosis
O E) Gestational trophoblastic disease
0 F) Incomplete abortion
0 G) Ovarian torsion
0 H) Ruptured ovarian cyst
0 I) Threatened abortion


19. A 23-year-old woman, gravida 2, para 0, has had vaginal spotting and abdominal
cramps Ior 2 days. Her last menstrual period was 8 weeks ago. A home pregnancy test
was positive 2 weeks ago. She underwent a salpingectomy4 years ago Iollowing an
ectopic pregnancy. Examination shows a closed cervix, an enlarged uterus, and no
adnexal masses. Transvaginal ultrasonography shows an empty uterus. Serum 3-hCG
level is 8000 mIU1mL.

CCCCCCCCCCCCCCCC

For each patient with vaginal bleeding during pregnancy, select the most likely
diagnosis.

C A) Abortion oI a blighted twin 0 F) Incomplete abortion
C B) Cervicitis ) Ovarian torsion
0 C) Ectopic pregnancy H) Ruptured ovarian cyst
0 D) Focal decidual necrosis I) Threatened abortion
0 E) Gestational trophoblastic disease


20. A 25-year-old woman has had painless vaginal postcoital bleeding Ior 2 hours.
Her last menstrual period was 10 weeks ago. A pregnancy test is positive. There is a
small amount oI dark blood in the vaginal canal. She has a reddened, Iriable cervical
os.


IIIIIIIIIIIIIIIIIIIIIIII



21. A previously healthy 62-year-old man comes to the physician because oI a 1-year
history oI numbness and weakness oI the right hand. He is a carpenter and has no
history oI injury to his hand. Examination shows wasting oI muscle mass in the Iirst
web space. Sensation to touch is decreased along the ring and little Iingers. There is
weakness oI abduction and adduction oI the Iingers. Which oI the Iollowing is the
most likely site oI nerve injury?
O A) Axillary nerve
O B) Long thoracic nerve
O C) Median nerve above the elbow
O D) Median nerve at the elbow
O E) Median nerve at the wrist
O F) Musculocutaneous nerve above the elbow O G) Musculocutaneous nerve at the
elbow
O H) Musculocutaneous nerve at the wrist
O I) Radial nerve above the elbow
O J) Radial nerve at the elbow
O K) Radial nerve at the wrist
O L) Suprascapular nerve
O M) Thoracodorsal nerve
0 N) Ulnar nerve at the elbow

NNNNNNNNNNNNNNNNNNNNN

22. A 45-year-old man comes to the physician Ior a Iollow-up visit. He has had
recurrent ulcer disease Ior 8 years, his symptoms are currently relieved with antacid
therapy. He had one episode oI bleeding 1 year ago that required a blood transIusion.
He appears pale. His hemoglobin level is 10.6 g/dL, mean corpuscular volume is 78
pm3, and reticulocyte count is 0.8. Which oI the Iollowing additional laboratory
Iindings is most likely?

Serum Iron Iron-binding Capacity (TIBC) Saturation oI Serum Ferritin
(pg/dL) (pg/dL) (N250-400) TIBC () (ng/mL)
O A) 30 120 25 50
O B) 40 360 11 10
O C) 60 180 33 80
O D) 80 200 40 120
0 E) 100 150 67 600

BBBBBBBBBBBBBBB




23. A 6-year-old girl is brought to the physician because oI a 1-day history oI
vomiting, headache, and weakness. One month ago, she underwent resection oI an
astrocytoma and placement oI a ventriculoatrial shunt Ior residual hydrocephalus, her
postoperative course had been uncomplicated. Currently, she is drowsy and irritable.
Her temperature is 37.5C (99.5F), blood pressure is 126/54 mm Hg, pulse is 82/min,
and respirations are 24/min. Funduscopic examination shows papilledema. ReIlexes
are brisk with hypertonia in the lower extremities.

Which oI the Iollowing is the most likely underlying mechanism Ior these symptoms?

O A) Carcinomatous inIiltration oI cerebrospinal Iluid
O B) MalIunction oI ventriculoatrial shunt
O C) Overproduction oI cerebrospinal Iluid Irom a choroid plexus papilloma
O D) Recurrence and extension oI the astrocytoma
O E) Thrombosis oI the superior sagittal sinus



BBBBBBBBBBBB

The response options Ior the next three items are the same. You will be required to
select one answer Ior each item in the set.

For each patient with a respiratory problem, select the most appropriate next step in
management.

O A)Administration oI BCG vaccine
O B)Administration oI an inhaled bronchodilator
O C)Administration oI oxygen
O D)Arterial blood gas analysis
O E)Bronchoscopy
O F)Culture oI the pharynx Ior bacteria
O G)Direct laryngoscopy
O H) Gastric washings Ior acid-Iast bacteria
O I) Intravenous inIusion oI saline
O J) Isoniazid and riIampin therapy O K) Lumbar puncture
O L) Measurement oI serum aspirin level
O M) Subcutaneous administration oI epinephrine.
N) Viral culture oI respiratory secretions
00) X-ray Iilms oI the chest
O P) X-ray Iilms oI the neck


24. A previously healthy 8-year-old boy is brought to the emergency department
because oI swelling oI the lips and diIIiculty breathing Ior 20 minutes. The symptoms
began when he was helping his Iather clean the gutters on their house. He appears
anxious. His temperature is 37.2C (99F), pulse is 1201min, and respirations are
50/min. Pulse oximetry shows an oxygen saturation oI 96. His lips and eyes appear
puIIy. He has subcostal and intercostal retractions. Auscultation oI the chest shows
diIIuse bilateral wheezing.


MMMMMMMMMMMMMMMMMM



For each patient with a respiratory problem, select the most appropriate next step in
management.

O A)Administration oI BCG vaccine 0 I) Intravenous inIusion oI saline
O B)Administration oI an inhaled bronchodilator O J) Isoniazid and riIampin therapy
Oc)Administration oI oxygen O K) Lumbar puncture
O D)Arterial blood gas analysis O L) Measurement oI serum aspirin level
O E)Bronchoscopy O M) Subcutaneous administration oI epinephrine
O F)Culture oI the pharynx Ior bacteria O N) Viral culture oI respiratory secretions
O G)Direct laryngoscopy 00) X-ray Iilms oI the chest
O H)Gastric washings Ior acid-Iast bacteria O P) X-ray Iilms oI the neck


25. A 16-year-old girl is brought to the emergency department because oI heavy
breathing Ior 8 hours, and vomiting and ringing in the ears Ior 1 hour. She recently
broke up with her boyIriend and has been threatening to hurt herselI. She appears
somnolent but is arousable and answers questions appropriately. Her temperature is
37.5C (99.5F), pulse is 88/min, and respirations are 50/min. Pulse oximetry shows
an oxygen saturation oI 96. She has no retractions, nasal Ilaring, or cough. The
lungs are clear to auscultation.


L) Measurement oI serum aspirin level



For each patient with a respiratory problem, select the most appropriate next step in
management.

0 A)Administration oI BCG vaccine 0 I) Intravenous inIusion oI saline
0 B)Administration oI an inhaled bronchodilator 0 J) Isoniazid and riIampin therapy
0 C)Administration oI oxygen 0 K) Lumbar puncture
0 D)Arterial blood gas analysis O L) Measurement oI serum aspirin level
0 E)Bronchoscopy 0 M) Subcutaneous administration oI epinephrine
0 F)Culture oI the pharynx Ior bacteria 0 N) Viral culture oI respiratory secretions
O G)Direct laryngoscopy O) X-ray Iilms oI the chest
O H)Gastric washings Ior acid-Iast bacteria 0 P) X-ray Iilms oI the neck


26. A previously healthy 6-year-old boy is brought to the physician Ior a Iollow-up
examination aIter a PPD skin test produced an induration greater than 15 mm. He has
had no cough or Iever. He has a good appetite and is at the 50th percentile Ior height
and weight. He appears well. His temperature is 37.2C (99F), pulse is 88/min, and
respirations are 18/min. The lungs are clear to
auscultation.

OOOOOOOOOOOOOOOOOOOO



27. A 25-year-old nulligravid woman comes to the emergency department because oI
severe pain in the right lower quadrant oI the abdomen Ior 4 hours. She has had no
nausea, vomiting, Iever, or chills. Three weeks ago, a right adnexal mass was Iound
on routine examination. Her last menstrual period was 3 days ago. Her temperature is
37.2C (99F), blood pressure is 110/70 mm Hg, and pulse is 92/min. Pelvic
examination shows right adnexal tenderness. Hemoglobin level is 13 g/dL, and
leukocyte count is 9000/mm3. Pelvic ultrasonography shows a small amount oI Iree
Iluid in the cul-de-sac. Which oI the Iollowing is the most likely cause oI the pain?
O A) Appendicitis
O B) Endometriosis
O C) Ovarian hemorrhage
O D) Ruptured ovarian cyst
O E) Torsion oI the adnexa
0 F) Tubal obstruction

DDDDDDDDDDDDDDDDDDDDD

28. A 15-year-old boy comes to the physician because oI acne over his Iace Ior 1 year.
He preIers not to use any oral medications. Examination shows 10 to 15 pustules.
Which oI the Iollowing is the most eIIective treatment Ior this patient's symptoms?

p A) Avoidance oI chocolate, Iatty Ioods, and caIIeine
p B) Use oI benzoyl peroxide soap
O C) Vigorous scrubbing oI aIIected areas
p D) Application oI vitamin A and vitamin E to aIIected areas
p E) Ultraviolet light therapy

BBBBBBBBBBBBBBBBBBBBBBBBBB


29. A 3-year-old boy with acute lymphoblastic leukemia has had Iever Ior 3 days. He
completed his last course oI chemotherapy 6 days ago. He has had no malaise, rash, or
anorexia and has had no known contact with sick children in preschool. He appears
well. His temperature is 38.6C (101.5F), blood pressure is 75160 mm Hg, pulse is
100/min, and respirations are 22/min. Examination shows normal Iindings.
Laboratory studies show:
Hemoglobin 10.1 g/dL (N11.5-15.5)
Leukocyte count 2200/mm3
Segmented neutrophils 5
Bands 1
Lymphocytes 65
Atypical lymphocytes 11
Monocytes 18
Platelet count 35,000/mm3

Which oI the Iollowing is the most appropriate next step in management?

O A) Schedule a Iollow-up visit and discharge without medication
O B) Observe him in the oIIice Ior 3 hours, discharge without medication iI
examination remains unchanged
O C) Discharge him with oral broad-spectrum antibiotic therapy
O D) Admit him to the hospital Ior observation
O E) Admit him to the hospital Ior intravenous broad-spectrum antibiotic therapy

EEEEEEEEEEEEEEEEEEEEEE



30. A 72-year-old woman is brought to the emergency department 1 hour aIter the
sudden onset oI right Iacial droop and weakness oI the right arm and leg. She takes
captopril Ior hypertension and daily aspirin. Her blood pressure is 150/90 mm Hg,
pulse is 80/min, and respirations are 161min. Examination shows a leIt carotid bruit
and right central Iacial paralysis. There is moderate expressive aphasia. A CT scan oI
the head shows no abnormalities. Which oI the Iollowing is the most appropriate
initial pharmacotherapy?
O A) Oral ticlopidine
O B) Oral warIarin
O C) Sublingual niIedipine
O D) Intravenous nitroprusside
O E) Intravenous tissue plasminogen activator




EEEEEEEEEEEEEEEEEEEEEE/




31. A previously healthy 37-year-old woman comes to the physician because oI a 3-
month history oI episodes oI severe anxiety, shortness oI breath, palpitations, and
numbness in her hands and Ieet. Her vital signs are within normal limits. Physical
examination shows no abnormalities. Thyroid Iunction studies and an ECG show no
abnormalities. Which oI the Iollowing is the most appropriate pharmacotherapy?
A) Lithium carbonate
B) Methylphenidate
O C) Olanzapine
p D) Paroxetine
0 E) Valproic acid

DDDDDDDDDDDDDD



32. A 37-year-old nulligravid woman comes to the physician because she has not been
able to conceive Ior 2 years. Her 40-year-old husband has a child by a previous
marriage. Her last menstrual period was 6 weeks ago, menses have occurred at
increasingly inIrequent intervals over the past year. FiIteen years ago, she was treated
Ior one episode oI Chlamydia trachomatis inIection. She weighs 59 kg (130 lb) and is
170 cm (67 in) tall. Examination shows no abnormalities. Serum studies show:

R-hCG 5 mlUImL
Follicle-stimulating hormone 50 mlU1mL
Luteinizing hormone 45 mlUImL
Prolactin 13 ngImL
Thyroid-stimulating hormone 3 pU/mL

Which oI the Iollowing is the most likely cause oI this patient's inIertility?

O A) Hypothalamic amenorrhea
O B) Hypothyroidism
O C) Pituitary adenoma
O D) Polycystic ovarian disease
O E) Premature ovarian Iailure
0 F) Tubal Iactor


EEEEEEEEEEEEEEEEE

33. A 47-year-old woman comes to the physician because oI a 2-day history oI Iever
and joint pain. Six days ago, she completed a 10-day course oI dicloxacillin Ior
Iolliculitis. She appears comIortable. Her temperature is 38.6C (101.5E), pulse is
721min and regular, respirations are 161min, and blood pressure is 120176 mm Hg.
Examination shows an urticarial rash over the trunk and extremities. There is
moderate generalized lymphadenopathy and diIIuse joint tenderness. The remainder
oI the examination shows no abnormalities. Which oI the Iollowing is the most likely
cause oI these symptoms?
O A) Epstein-Barr virus inIection
O B) IgE-mediated allergic reaction
O C) Mycobacterium haemophilum inIection
O D) Serum sickness
0 E) Staphylococcal sepsis

DDDDDDDDDDDDDDDDDDDD


34. A 34-year-old man is brought to the emergency department by his Iamily because
oI a decreased need Ior sleep, increased goal-directed activity, increasingly irritable
and unstable mood, and uncharacteristic sexual promiscuity over the past week.
Yesterday, he quit his long-standing job at a hardware store and plans to move to
Washington, DC, to become the "Senator-in-ChieI." He says that this position is being
created especially Ior him by the president. He believes that he has developed
supersensitive hearing and is now able to hear conversations oI people hundreds oI
miles away. He has no history oI drug use, and there is no personal or Iamily history
oI psychiatric illness. On examination, he is extremely irritable and agitated,
threatening to "punch out anyone who tries to mess with me." Which oI the Iollowing
is the most likely diagnosis?
O A) Bipolar disorder
O B) Cyclothymic disorder
O C) Delusional disorder
O D) Schizophrenia
O E) SchizophreniIorm disorder

AAAAAAAAAAAAAAAAAAAAAAAA

.


35. A 17-year-old girl has had Ilu-like symptoms, low-grade Iever, and malaise Ior 3
days and mild jaundice Ior 2 days. Serum studies show an aspartate aminotransIerase
(AST, GOT) activity oI 670 UIL and an alanine aminotransIerase (ALT. GPT)
activity oI 860 U/L. Serum IgM antibody to hepatitis A is positive. Which oI the
Iollowing is most likely to minimize the risk Ior this disease in Iamily members?
O A) Acyclovir therapy Ior Iamily members
O B) Hepatitis B vaccination Ior Iamily members
O C) Immune globulin therapy Ior Iamily members
OD) InterIeron alIa-2b therapy Ior Iamily members
0 E) Use oI separate toilet Iacilities by the patient

CCCCCCCCCCCCCCCCC
O
Permalink Reply by usmle99 on March 13, 2009 at 7:28am
36. One day aIter missing her dialysis treatment, a 27-year-old woman comes to the
physician because oI generalized weakness. Her blood pressure is 95172 mm Hg, and
pulse is 45/min and regular. Examination shows a well-Iunctioning arteriovenous
Iistula in the leIt upper extremity. Muscle strength is 315 in all extremities. An ECG
shows third-degree atrioventricular block. Laboratory studies are ordered. Which oI
the Iollowing is the most appropriate next step in management?
O A) Kayexalate-sorbitol enema
O B) Intravenous administration oI calcium gluconate
O C) Oral administration oI sodium polystyrene sulIonate (Kayexalate)
O D) Immediate placement oI a pacemaker
O E) No treatment pending laboratory results


DDDDDDDDDDDDDDDDDD

37. A previously healthy 2-year-old boy is brought to the physician because oI Iever
and abdominal pain Ior 24 hours. His developmental milestones are appropriate Ior
age. He is Iully alert and responsive. His temperature is 38.6C (101.5F), blood
pressure is 85/60 mm Hg, pulse is 1001min, and respirations are 20/min. Examination
shows suprapubic tenderness to deep palpation but no guarding or rebound. There are
no palpable abdominal masses or costovertebral angle tenderness. The penis is
circumcised, there is no urethral discharge. Urinalysis shows 20-30 leukocytes/hpI, 5-
6 erythrocytes/hpI, and nitrites. A urine culture grows 100,000 colonies/mL oI
Escherichia coli sensitive to all tested antibiotics. Amoxicillin therapy is initiated.
Five days later, he is aIebrile and asymptomatic. Ultrasonography oI the kidneys
shows no abnormalities. Which oI the Iollowing is the most appropriate next step in
management?

O A) Discontinue the antibiotic in 2 days and reexamine only iI symptoms recur
O B) Voiding cystourethrography
O C) Intravenous pyelography
O D) Cystoscopy
O E) No Iurther testing


BBBBBBBBBBBBBBBBBBBBBBB

38. A 62-year-old woman comes to the physician because oI increasingly severe low
back pain over the past month. Prolonged periods oI rest have not relieved her
symptoms. Sensorimotor examination oI the lower extremities shows no
abnormalities. Her hematocrit is 32, and serum calcium level is 12 mgIdL. An x-ray
Iilm oI the chest shows no abnormalities. An x-ray Iilm oI the spine is shown. Which
oI the Iollowing is the most likely diagnosis?
O A) Herniated nucleus pulposus
O B) Mechanical low back pain
O C) Metastatic carcinoma
O D) Osteopetrosis
0 E) Spinal stenosis


ccccccccccccccccc


39. A 43-year-old man comes to the physician Ior evaluation and management oI
cardiac risk Iactors 8 weeks aIter sustaining a myocardial inIarction. He takes aspirin
and metoprolol daily, and he does not smoke cigarettes. His Iather and brother both
had myocardial inIarctions beIore the age oI 50 years, their serum cholesterol levels
are unknown. There is no Iamily history oI diabetes mellitus. He weighs 86 kg (190
lb) and is 180 cm (71 in) tall. His blood pressure is 130170 mm Hg, pulse is 68/min,
and respirations are 14/min. Two years ago, his serum cholesterol level was 245
mg/dL. Fasting serum glucose level is 88 mg/dL. Which oI the Iollowing is the most
appropriate next step to evaluate his cardiac risk Iactors?


O A) Random measurements oI serum cholesterol level
O B) Measurement oI Iasting serum cholesterol level only
O C) Fasting serum lipid studies only
O D) Oral glucose tolerance test and Iasting serum lipid studies
O E) Oral glucose tolerance test and measurement oI Iasting serum cholesterol level

CCCCCCCCCCCCCCCCCCCC



40. A 47-year-old man comes to the physician because oI a 3-week history oI
increasing Iacial swelling and a 1-week history oI morning headaches and mild
shortness oI breath. He had previously Ielt well. He completed a course oI
chemotherapy 4 months ago Ior small cell carcinoma oI the lung. His temperature is
37.2C (98.9F), blood pressure is 142180 mm Hg with an 8-mm Hg paradoxical
pulse, pulse is 84/min, and respirations are 18/min. Examination shows signiIicant
diIIuse Iacial and periorbital edema. The optic discs are sharp, and ocular movements
are intact. The lungs are hyperresonant bilaterally with a moderately prolonged
expiratory phase. Mild rhonchi are heard on inspiration and expiration. An x-ray Iilm
oI the chest shows a 10-cm mass in the right upper lobe and apex. Which
oI the Iollowing is the most likely explanation Ior these Iindings?
O A) Chemotherapy-induced bone marrow toxicity
O B) Chemotherapy-related cardiac toxicity
O C) Hypercoagulable state secondary to malignancy
O D) Interstitial metastatic pulmonary disease
O E) Intracranial metastases
O F) Lymphatic obstruction
O G) Malignant pericarditis
O H) Paraneoplastic syndrome
O I) Pleural metastases
O J) SoIt tissue metastases
0 K) Vascular obstruction


KKKKKKKKKKKKKKKKK


41. A 13-month-old girl is brought Ior a well-child examination. There is no history oI
prenatal or perinatal problems. Her diet consists oI breast milk, juice, legumes,
cooked vegetables, and Iruit. She has had three ear inIections treated successIully with
amoxicillin and two episodes oI diarrhea since birth. She will pull to a stand and stand
alone Ior 1 minute but has not begun to walk. She is able to pick up raisins with her
thumb and IoreIinger but is unable to Ieed herselI with a spoon. Which oI the
Iollowing is the most appropriate
assessment oI Iine and gross motor development?

`` Fine Motor Gross Motor
Development Development
O A) Normal normal
O B) Normal delayed
O C) Delayed normal
O D) Delayed delayed

AAAAAAAAAAAAAAAAAAA

42. An 18-month-old girl is brought to the physician because oI intermittent shortness
oI breath over the past 6 months. Her mother notes that her daughter oIten squats to
relieve her symptoms. Examination shows cyanosis and clubbing oI the Iingers. A
prominent right ventricular impulse with a grade 3/6, systolic ejection murmur is
heard best in the third leIt intercostal space. An ECG shows right ventricular
hypertrophy. An x-ray Iilm oI the chest shows a small-sized heart and decreased
pulmonary blood Ilow. Which oI the Iollowing is the most likely diagnosis?
O A) Atrial septal deIect (ostium primum type)
O B) Atrial septal deIect (ostium secundum type)
O C) Atrioventricular canal
O D) Coarctation oI the aorta
O E) Hypoplastic leIt heart syndrome
O F) Patent ductus arteriosus
O G) Tetralogy oI Fallot
O H) Transposition oI the great arteries
O I) Tricuspid atresia
0 J) Ventricular septal deIect

GGGGGGGGGGGG


43. A 5-day-old girl has been Ieeding poorly since birth. She weighed 2900 g (6 lb 7
oz) at birth and now weighs 2600 g (5 lb 12 oz). Physical examination shows an
enlarged clitoris and labial Iusion. A sibling born 5 years ago died at 1 week oI age. A
liIe-threatening complication oI this syndrome results Irom Iailure to produce which
oI the Iollowing?
O A) Aldosterone
O B) Cholesterol
O C) Estrogen
O D) Insulin
0 E) Testosterone


AAAAAAAAAAAAAAAAAAA

44. A 57-year-old hospitalized man undergoes right subclavian venous catheterization
Ior hyperalimentation. He is currently being treated Ior a small bowel Iistula. While
the results oI an x-ray Iilm oI the chest to check the catheter position are pending, the
patient suddenly becomes agitated. His blood pressure is 70150 mm Hg, and pulse is
1101min. Examination shows jugular venous distention. The lungs are clear to
auscultation. Breath sounds are equal bilaterally. The trachea is midline. An x-ray
Iilm oI the chest shows a catheter in the superior vena cava, transversing the right
ventricle and crossing to the leIt oI the midline. Which oI the Iollowing is the most
likely cause oI this patient's hypotension?
O A) Air embolism
O B) Pericardial tamponade
O C) Pulmonary artery perIoration
O D) Staphylococcal bacteremia
O E) Tension pneumothorax

BBBBBBBBBBBBBBBBBBBBBB

45. Twenty-Iour hours aIter hospitalization Ior Iixation oI a Iemoral Iracture, a 35-
year-old construction worker has nausea, agitation, and insomnia. His blood pressure
is 1501100 mm Hg. He is oriented but anxious. Which oI the Iollowing is the most
appropriate next step in diagnosis?
O A) Inquire about recent alcohol intake
O B) Examination oI a urine specimen Ior Iat bodies
O C) Leukocyte count with diIIerential
O D) ECG
0 E) Pulmonary arteriography

AAAAAAAAAAAAAAAAAAA

46. A previously healthy 27-year-old woman comes to the physician because oI a 3-
month history oI moderate abdominal pain that improves Ior a short time aIter she
eats. She has not had any rectal bleeding. Her temperature is 36.4C (97.5F), blood
pressure is 110170 mm Hg, and pulse is 80/min. Abdominal examination shows
midepigastric tenderness. Her hemoglobin level is 12 gIdL, and leukocyte count is
80001mm3. Serum Helicobacterpylori antibody assay is positive. Which oI the
Iollowing is the most appropriate next step in management?
O A) HIDA scan
O B) Amoxicillin, clarithromycin, and omeprazole therapy
O C) CiproIloxacin therapy
O D) Omeprazole, magnesium hydroxide, and metoclopramide therapy O E)
Ranitidine therapy


BBBBBBBBBBBBBBBBBBBB







Report Abuse

* Re:Ans key Ior NBME 3
#995128

coolamazon - 10/17/07 01:43


Here is NBME 3 sec 2:

here you go the section 2


1. For the past 10 years, a 28-year-old woman has Iollowed a strict diet that prohibits
dairy products, meat, Iish, and Iowl. On routine examination, her hematocrit is 30,
and mean corpuscular volume is 122 pm3. Which oI the Iollowing is the most
appropriate next step in diagnosis?

O A) Erythrocyte Iolate determination
O B) Serum iron and total iron-binding capacity determinations
O C) Serum thyroid hormone determination
O D) Serum urea nitrogen (BUN) determination
O E) Serum vitamin B12 (cyanocobalamin) determination

EEEEEEEEEEEEEEE


2. A 47-year-old man comes to the physician because oI a persistent nonproductive
cough Ior 3 months. He has not had Iever, rhinorrhea, or nasal congestion. He was
seen by a physician 3 months ago Ior the cough and treated with amoxicillin and an
over-the-counter cough medication. He has a 3-month history oI hypertension well
controlled with lisinopril. He is in no distress. He weighs 79 kg (175 lb) and is 178 cm
(70 in) tall. His temperature is 37.1C (98.8F), blood pressure is 130178 mm Hg,
pulse is 721min and regular, and respirations are 161min. Examination shows no
abnormalities. A complete blood count and x-ray Iilms oI the chest show no
abnormalities. Which oI the Iollowing is the most appropriate next step in
management?

O A) X-ray Iilms oI the sinuses
O B) Discontinue lisinopril therapy
O Q Add a beclomethasone inhaler to the medication regimen
O D) Add ciproIloxacin to the medication regimen
O E) Add hydrocodone to the medication regimen


BBBBBBBBBBBBBBBBBBBBB

3. A 12-year-old girl is brought to the physician by her mother because she is
concerned about her daughter's diIIiculty making Iriends and socializing. Her
development and adjustment had been normal until 6 months ago when she began to
reIuse to use restrooms at school or eat in the caIeteria. Her mother describes her as a
quiet, serious child who does not readily interact with other people. Her teachers
report that her concentration varies. at times, she appears to be daydreaming. On
examination, she is reserved but pleasant and appears to be oI normal intelligence.
Her speech is normal in rate and rhythm. She says that she is concerned that her voice
will Iail her iI she has to read aloud in class. Which oI the Iollowing is the most likely
diagnosis?

O A) Attention-deIicitthyperactivity disorder, inattentive type
O B) Autistic disorder
O C) Expressive language disorder
O D) Oppositional deIiant disorder
O E) Selective mutism
O F) Social phobia
O G) Age-appropriate behavior

FFFFFFFFFFFFFFFFFF



A picture here Ior q4

4. A 25-year-old woman is brought to the emergency department 45 minutes aIter
being hit in the right eye with a tennis ball. She has severe pain and decreased vision
in the injured eye. Visual acuity is 201400 on the right. A photograph oI the eye is
shown. Which oI the Iollowing is the most likely diagnosis?
O A) Acute angle-closure glaucoma
O B) Anterior iritis
O C) Cataract
O D) Hyphema
O E) Hypopyon

DDDDDDDDDDDDDDD

5. A 44-year-old woman is brought to the emergency department 40 minutes aIter
being stabbed in the leIt groin. Her blood pressure is 128/84 mm Hg, pulse is
100/min, and respirations are 16/min. Examination oI the wound shows a small
hematoma and no external bleeding. Popliteal and pedal pulses are palpable in the leIt
lower extremity below the injury. Lacerations oI the Iemoral artery and vein are Iound
and repaired. Two days postoperatively, she has progressively severe pain oI the leIt
lower extremity and swelling oI the leg Irom the knee down. Which oI the Iollowing
is the most likely cause oI her new symptoms?

O A) Arterial embolism
O B) Compartment syndrome
O C) Lymphatic disruption Irom the injury
O D) Thrombosis oI the Iemoral vein
0 E) Wound hemorrhage

BBBBBBBBBBBBBB


6. The crude mortality rate Ior coronary artery disease in Community A is twice the
crude mortality rate Ior coronary artery disease in Community B. The age-
standardized mortality rates Ior coronary artery disease in the two communities are
the same. These Iindings are most consistent with which oI the Iollowing inIerences?

A) Coronary artery disease mortality in the elderly is higher in Community A than
Community B
B) Coronary artery disease mortality in young adults is higher in Community A than
Community B
C) The population oI Community B is younger than the population oI Community A
D) The proportionate mortality Irom coronary artery disease is higher in Community
A than Community B
E) The two communities have similar age distributions

CCCCCCCCCCCCCC

7. An otherwise healthy 28-year-old man comes to the physician because oI a dry
hacking cough Ior 3 months. A routine examination 6 months ago showed no
abnormalities. His paternal grandIather had colon cancer. Examination today shows
normal Iindings. An x-ray Iilm oI the chest shows a large anterior mediastinal mass.
Which oI the Iollowing is the most likely origin oI this neoplasm?
O A) Colon
O B) Kidney
O C) Prostate
O D) Stomach
0 E) Testicle

EEEEEEEEEEEEEEEEEE

A testis tumor may become metastatic and maniIest with large retroperitoneal and/or
chest lesions, while the primary tumor is nonpalpable. Scrotal ultrasonography may
locate the primary tumor. Histopathology oI the primary testis oIten shows a Iocus oI
tumor surrounded by Iibrous scar, termed burned-out testis cancer.

8. A 50-year-old man has a cardiac arrest during sexual intercourse. He is promptly
resuscitated and recovers uneventIully Irom an acute anterior wall myocardial
inIarction. At discharge Irom the hospital, he and his wiIe inquire about the saIety oI
Iuture sexual intercourse. Which oI the Iollowing is the most appropriate advice?
O A) Psychiatric consultation is necessary
O B) Sexual intercourse should be perIormed while wearing an ambulatory ECG
monitor
O C) Sexual intercourse will be saIe when exercise tolerance improves
O D) They should wait 3 months beIore having sexual intercourse
O E) They should have sexual intercourse as soon as possible to overcome the Iear oI
another event



CCCCCCCCCCCCCCCCCC



9. A 68-year-old woman with terminal metastatic breast cancer is living at home with
her son. She has a living will requesting that she be allowed to "die in peace." She is
unresponsive to voice and has not had any Iood or liquids Ior 3 days. Her son
disagrees with her decision not to accept Iurther therapy, including chemotherapy,
antibiotics, hospitalization, and enteral or parenteral nutrition. Which oI the Iollowing
is the most appropriate next step in management?
p A) Abide by the decisions oI the next oI kin
p B) Obtain a court order to allow additional therapy to be given
O C) Start enteral Ieedings
p D) Admit her to the hospital
0 E) No Iurther intervention


EEEEEEEEEEEEEEEEEEE


10. A 57-year-old woman comes to the physician Ior a Iollow-up examination. She
has a 5-year history oI hypercalcemia, which was diagnosed with routine laboratory
studies, and her serum calcium levels have ranged Irom 10.8 mg/dL to 11.5 mg/dL.
She declined Iurther evaluation in the past because she "Ielt well." She takes no
medications. Her last menstrual period was 7 years ago. She maintains a weight oI 67
kg (148 lb) and is 170 cm (67 in) tall, BMI is 23 kg/m2. Her blood pressure is 126/80
mm Hg, and pulse is 66/min. Examination shows no abnormalities. Which oI the
Iollowing is the most appropriate next step to assess her risk Ior Iracture?

O A) 24-Hour urine collection Ior measurement oI collagen cross-links excretion
O B) Bone densitometry
O C) Bone-speciIic measurement oI serum alkaline phosphatase activity
O D) Posteroanterior and lateral x-ray Iilms oI the thoracic spine and hips
O E) Biopsy oI the iliac crest


BBBBBBBBBBBBBBBBBBBBBBB



11. A 30-year-old woman, gravida 4, para 3, is admitted to the hospital in labor at 38
weeks' gestation. The cervix is 4 cm dilated. Contractions occur every4 minutes. The
Iundal height is 40 cm. The membranes are intact. The Ietal lie cannot be determined
by abdominal examination, and no presenting part is palpable in the pelvis. Which oI
the Iollowing is the most appropriate next step in management?

O A) Ultrasonography
O B) Administration oI oxytocin
O C) Administration oI tocolytic drugs
O D) Amniotomy
O E) Cesarean delivery


AAAAAAAAAAAAAAAAA


12. Six days aIter undergoing a laparoscopic cholecystectomy Ior acute cholecystitis,
a 35-year-old woman comes to the physician because oI Iever and abdominal pain Ior
3 days. She is jaundiced. Her temperature is 38C (100.4F). Abdominal examination
shows distention and incisions that are healing normally. Leukocyte count is
12,000/mm3, and total serum bilirubin level is 7.9 mg/dL. Which oI the Iollowing is
the most likely cause oI the jaundice?

O A) Anesthetic-related hepatitis
O B) Common bile duct injury
O C) Fulminant hepatic Iailure
O D) Reaction to perioperative antibiotics
O E) Subhepatic abscess

BBBBBBBBBBBBB


13. A 9-year-old boy is brought to the physician because oI progressive weakness and
a purple-red discoloration over his cheeks and upper eyelids over the past 8 weeks.
His symptoms began shortly aIter a camping trip, and he now is unable to climb
stairs, walk long distances, comb his hair, or dress himselI. His mother says that she
was careIul to apply his sunscreen on the trip and can recall no tick bites or exposure
to poisonous plants. His only medication is a topical corticosteroid Ior several dry,
scaly patches oI the skin. He appears weak and lethargic. He is at the 75th percentile
Ior height and 25th percentile Ior weight, he has had no change in his weight since his
last examination 9 months ago. His temperature is 37.7C (99.8F), blood pressure is
110/68 mm Hg, pulse is 105/min, and respirations are 28/min. Examination oI the
skin shows a purple-red discoloration over the cheeks and eyelids, periorbital edema,
erythematous plaques and scales over the elbows and knees, and Ilat-topped red
papules over all knuckles. There is generalized weakness and atrophy oI the proximal
muscles. Which oI the Iollowing is the most likely diagnosis2

O A) Dermatomyositis
O B) Duchenne's muscular dystrophy
O C) Eczema
O D) Lyme disease
O E) Psoriasis
O F) Rocky Mountain spotted Iever
O G) Seborrhea
0 H) Systemic lupus erythematosus


AAAAAAAAAAAAAAAA

14. An 82-year-old man is brought to the emergency department because oI the
sudden onset oI conIusion 48 hours ago. His Iamily says that he previously had been
Iunctional and independent. His temperature is 37.5C (99.5F), blood pressure is
110/70 mm Hg, pulse is 90/min, and respirations are 12/min. The skin is warm and
clammy. Bowel sounds are hypoactive, and there is guarding and tenderness in the
right lower quadrant oI the abdomen. Examination oI the heart, lungs, and extremities
shows no abnormalities. He is drowsy but arousable and oriented to person but not
place or time. There are no Iocal deIicits. Which oI the Iollowing is the most likely
diagnosis?
O A) Appendicitis
O B) Colon cancer
O C) Meningitis
O D) Pneumonia
0 E) Urinary tract inIection

AAAAAAAAAAAAAAAA

15. Two hours aIter emergency cholecystectomy, a 48-year-old woman has an oxygen
saturation oI 84 and a PO2 oI 56 mm Hg on 2 LImin oI oxygen via nasal cannula.
Her blood pressure is 120180 mm Hg, and respirations are 16/min. Decreased breath
sounds are heard on the right, and there is decreased excursion on inspiration
bilaterally. There is minimal dullness over the right base. Examination shows no
jugular venous distention or pedal edema. An x-ray Iilm oI the chest shows increased
density over the right lower lung Iield, the tracheal, mediastinal, and cardiac
silhouettes are shiIted to the right. The right lung Iield appears considerably smaller
than the leIt lung Iield. Which oI the Iollowing is the most likely cause oI these
Iindings?
O A) Atelectasis
O B) Pleural eIIusion
O C) Pneumonia
O D) Pneumothorax
0 E) Pulmonary embolism

AAAAAAAAAAAA



16. A 47-year-old man comes to the physician because oI low-grade Iever and malaise
Ior 3 months. He was inIected with hepatitis B 25 years ago. He appears well
nourished but ill. His temperature is 37.7C (99.8F), blood pressure is 110170 mm
Hg, pulse is 801min, and respirations are 121min. Examination shows slight jaundice
and hepatomegaly. The spleen is not palpable, and there is no ascites or dependent
edema. Serum studies show:

Total bilirubin 3 mg1dL
Alkaline phosphatase 160 U!L
Aspartate aminotransIerase (AST, GOT) 230 U!L
Alanine aminotransIerase (ALT, GPT) 275 U!L

Serum a-Ietoprotein level is markedly increased. Ultrasonography oI the right upper
quadrant oI the abdomen shows a 5-cm, solid lesion in the right lobe oI the liver.
Biopsy is most likely to show which oI the Iollowing?
O A) Focal nodular hyperplasia
O B) Hepatic adenoma
O C) Hepatocellular carcinoma
O D) Metastatic adenocarcinoma
0 E) Regenerating liver nodule


CCCCCCCCCCCCCCCCCC



17. A previously healthy 30-year-old woman has had a painless lump in her neck Ior 2
days. Her mother was treated Ior a thyroid tumor at the age oI 35 years. Her 28-year-
old sister has an increased serum calcitonin level but no thyroid mass. Examination
shows a palpable thyroid nodule. Her serum calcitonin level is increased. Which oI
the Iollowing diagnoses should be excluded prior to surgical treatment oI the thyroid
gland?
O A) Adrenocortical carcinoma
O B) Lung carcinoma
O C) Ovarian carcinoma
O D) Parathyroid carcinoma
O E) Pheochromocytoma

EEEEEEEEEEEEEEEEEEEEE



18. A 67-year-old man comes to the physician because oI a 1-month history oI
shortness oI breath with exertion, easy Iatigability, and mild ankle swelling. Five
years ago, he underwent successIul chemotherapy with doxorubicin Ior lymphoma.
Moist crackles are heard throughout both lungs. An S3 is heard. The liver edge is
palpated 4 cm below the right costal margin. There is 2 pitting edema oI the pretibial
region, ankles, and Ieet. Which oI the Iollowing is the most likely underlying cause oI
this patient's symptoms?
O A) Aortic sclerosis
O B) Cardiotoxicity
O C) Cirrhosis
O D) Mediastinal obstruction
0 E) Pneumonitis

BBBBBBBBBBBBBBBBBBBB

The response options Ior the next two items are the same. You will be required to
select one answer Ior each item in the set.

For each patient with vaginal bleeding during pregnancy, select the most likely
diagnosis.

O A)Abortion oI a blighted twin
O B)Cervicitis
O C)Ectopic pregnancy
O D)Focal decidual necrosis
O E) Gestational trophoblastic disease
0 F) Incomplete abortion
0 G) Ovarian torsion
0 H) Ruptured ovarian cyst
0 I) Threatened abortion


19. A 23-year-old woman, gravida 2, para 0, has had vaginal spotting and abdominal
cramps Ior 2 days. Her last menstrual period was 8 weeks ago. A home pregnancy test
was positive 2 weeks ago. She underwent a salpingectomy4 years ago Iollowing an
ectopic pregnancy. Examination shows a closed cervix, an enlarged uterus, and no
adnexal masses. Transvaginal ultrasonography shows an empty uterus. Serum 3-hCG
level is 8000 mIU1mL.

CCCCCCCCCCCCCCCC

For each patient with vaginal bleeding during pregnancy, select the most likely
diagnosis.

C A) Abortion oI a blighted twin 0 F) Incomplete abortion
C B) Cervicitis ) Ovarian torsion
0 C) Ectopic pregnancy H) Ruptured ovarian cyst
0 D) Focal decidual necrosis I) Threatened abortion
0 E) Gestational trophoblastic disease


20. A 25-year-old woman has had painless vaginal postcoital bleeding Ior 2 hours.
Her last menstrual period was 10 weeks ago. A pregnancy test is positive. There is a
small amount oI dark blood in the vaginal canal. She has a reddened, Iriable cervical
os.


IIIIIIIIIIIIIIIIIIIIIIII



21. A previously healthy 62-year-old man comes to the physician because oI a 1-year
history oI numbness and weakness oI the right hand. He is a carpenter and has no
history oI injury to his hand. Examination shows wasting oI muscle mass in the Iirst
web space. Sensation to touch is decreased along the ring and little Iingers. There is
weakness oI abduction and adduction oI the Iingers. Which oI the Iollowing is the
most likely site oI nerve injury?
O A) Axillary nerve
O B) Long thoracic nerve
O C) Median nerve above the elbow
O D) Median nerve at the elbow
O E) Median nerve at the wrist
O F) Musculocutaneous nerve above the elbow O G) Musculocutaneous nerve at the
elbow
O H) Musculocutaneous nerve at the wrist
O I) Radial nerve above the elbow
O J) Radial nerve at the elbow
O K) Radial nerve at the wrist
O L) Suprascapular nerve
O M) Thoracodorsal nerve
0 N) Ulnar nerve at the elbow

NNNNNNNNNNNNNNNNNNNNN

22. A 45-year-old man comes to the physician Ior a Iollow-up visit. He has had
recurrent ulcer disease Ior 8 years, his symptoms are currently relieved with antacid
therapy. He had one episode oI bleeding 1 year ago that required a blood transIusion.
He appears pale. His hemoglobin level is 10.6 g/dL, mean corpuscular volume is 78
pm3, and reticulocyte count is 0.8. Which oI the Iollowing additional laboratory
Iindings is most likely?

Serum Iron Iron-binding Capacity (TIBC) Saturation oI Serum Ferritin
(pg/dL) (pg/dL) (N250-400) TIBC () (ng/mL)
O A) 30 120 25 50
O B) 40 360 11 10
O C) 60 180 33 80
O D) 80 200 40 120
0 E) 100 150 67 600

BBBBBBBBBBBBBBB




23. A 6-year-old girl is brought to the physician because oI a 1-day history oI
vomiting, headache, and weakness. One month ago, she underwent resection oI an
astrocytoma and placement oI a ventriculoatrial shunt Ior residual hydrocephalus, her
postoperative course had been uncomplicated. Currently, she is drowsy and irritable.
Her temperature is 37.5C (99.5F), blood pressure is 126/54 mm Hg, pulse is 82/min,
and respirations are 24/min. Funduscopic examination shows papilledema. ReIlexes
are brisk with hypertonia in the lower extremities.

Which oI the Iollowing is the most likely underlying mechanism Ior these symptoms?

O A) Carcinomatous inIiltration oI cerebrospinal Iluid
O B) MalIunction oI ventriculoatrial shunt
O C) Overproduction oI cerebrospinal Iluid Irom a choroid plexus papilloma
O D) Recurrence and extension oI the astrocytoma
O E) Thrombosis oI the superior sagittal sinus
O
Permalink Reply by usmle99 on March 13, 2009 at 7:28am
BBBBBBBBBBBB

The response options Ior the next three items are the same. You will be required to
select one answer Ior each item in the set.

For each patient with a respiratory problem, select the most appropriate next step in
management.

O A)Administration oI BCG vaccine
O B)Administration oI an inhaled bronchodilator
O C)Administration oI oxygen
O D)Arterial blood gas analysis
O E)Bronchoscopy
O F)Culture oI the pharynx Ior bacteria
O G)Direct laryngoscopy
O H) Gastric washings Ior acid-Iast bacteria
O I) Intravenous inIusion oI saline
O J) Isoniazid and riIampin therapy O K) Lumbar puncture
O L) Measurement oI serum aspirin level
O M) Subcutaneous administration oI epinephrine.
N) Viral culture oI respiratory secretions
00) X-ray Iilms oI the chest
O P) X-ray Iilms oI the neck


24. A previously healthy 8-year-old boy is brought to the emergency department
because oI swelling oI the lips and diIIiculty breathing Ior 20 minutes. The symptoms
began when he was helping his Iather clean the gutters on their house. He appears
anxious. His temperature is 37.2C (99F), pulse is 1201min, and respirations are
50/min. Pulse oximetry shows an oxygen saturation oI 96. His lips and eyes appear
puIIy. He has subcostal and intercostal retractions. Auscultation oI the chest shows
diIIuse bilateral wheezing.


MMMMMMMMMMMMMMMMMM



For each patient with a respiratory problem, select the most appropriate next step in
management.

O A)Administration oI BCG vaccine 0 I) Intravenous inIusion oI saline
O B)Administration oI an inhaled bronchodilator O J) Isoniazid and riIampin therapy
Oc)Administration oI oxygen O K) Lumbar puncture
O D)Arterial blood gas analysis O L) Measurement oI serum aspirin level
O E)Bronchoscopy O M) Subcutaneous administration oI epinephrine
O F)Culture oI the pharynx Ior bacteria O N) Viral culture oI respiratory secretions
O G)Direct laryngoscopy 00) X-ray Iilms oI the chest
O H)Gastric washings Ior acid-Iast bacteria O P) X-ray Iilms oI the neck


25. A 16-year-old girl is brought to the emergency department because oI heavy
breathing Ior 8 hours, and vomiting and ringing in the ears Ior 1 hour. She recently
broke up with her boyIriend and has been threatening to hurt herselI. She appears
somnolent but is arousable and answers questions appropriately. Her temperature is
37.5C (99.5F), pulse is 88/min, and respirations are 50/min. Pulse oximetry shows
an oxygen saturation oI 96. She has no retractions, nasal Ilaring, or cough. The
lungs are clear to auscultation.


L) Measurement oI serum aspirin level



For each patient with a respiratory problem, select the most appropriate next step in
management.

0 A)Administration oI BCG vaccine 0 I) Intravenous inIusion oI saline
0 B)Administration oI an inhaled bronchodilator 0 J) Isoniazid and riIampin therapy
0 C)Administration oI oxygen 0 K) Lumbar puncture
0 D)Arterial blood gas analysis O L) Measurement oI serum aspirin level
0 E)Bronchoscopy 0 M) Subcutaneous administration oI epinephrine
0 F)Culture oI the pharynx Ior bacteria 0 N) Viral culture oI respiratory secretions
O G)Direct laryngoscopy O) X-ray Iilms oI the chest
O H)Gastric washings Ior acid-Iast bacteria 0 P) X-ray Iilms oI the neck


26. A previously healthy 6-year-old boy is brought to the physician Ior a Iollow-up
examination aIter a PPD skin test produced an induration greater than 15 mm. He has
had no cough or Iever. He has a good appetite and is at the 50th percentile Ior height
and weight. He appears well. His temperature is 37.2C (99F), pulse is 88/min, and
respirations are 18/min. The lungs are clear to
auscultation.

OOOOOOOOOOOOOOOOOOOO



27. A 25-year-old nulligravid woman comes to the emergency department because oI
severe pain in the right lower quadrant oI the abdomen Ior 4 hours. She has had no
nausea, vomiting, Iever, or chills. Three weeks ago, a right adnexal mass was Iound
on routine examination. Her last menstrual period was 3 days ago. Her temperature is
37.2C (99F), blood pressure is 110/70 mm Hg, and pulse is 92/min. Pelvic
examination shows right adnexal tenderness. Hemoglobin level is 13 g/dL, and
leukocyte count is 9000/mm3. Pelvic ultrasonography shows a small amount oI Iree
Iluid in the cul-de-sac. Which oI the Iollowing is the most likely cause oI the pain?
O A) Appendicitis
O B) Endometriosis
O C) Ovarian hemorrhage
O D) Ruptured ovarian cyst
O E) Torsion oI the adnexa
0 F) Tubal obstruction

DDDDDDDDDDDDDDDDDDDDD

28. A 15-year-old boy comes to the physician because oI acne over his Iace Ior 1 year.
He preIers not to use any oral medications. Examination shows 10 to 15 pustules.
Which oI the Iollowing is the most eIIective treatment Ior this patient's symptoms?

p A) Avoidance oI chocolate, Iatty Ioods, and caIIeine
p B) Use oI benzoyl peroxide soap
O C) Vigorous scrubbing oI aIIected areas
p D) Application oI vitamin A and vitamin E to aIIected areas
p E) Ultraviolet light therapy

BBBBBBBBBBBBBBBBBBBBBBBBBB


29. A 3-year-old boy with acute lymphoblastic leukemia has had Iever Ior 3 days. He
completed his last course oI chemotherapy 6 days ago. He has had no malaise, rash, or
anorexia and has had no known contact with sick children in preschool. He appears
well. His temperature is 38.6C (101.5F), blood pressure is 75160 mm Hg, pulse is
100/min, and respirations are 22/min. Examination shows normal Iindings.
Laboratory studies show:
Hemoglobin 10.1 g/dL (N11.5-15.5)
Leukocyte count 2200/mm3
Segmented neutrophils 5
Bands 1
Lymphocytes 65
Atypical lymphocytes 11
Monocytes 18
Platelet count 35,000/mm3

Which oI the Iollowing is the most appropriate next step in management?

O A) Schedule a Iollow-up visit and discharge without medication
O B) Observe him in the oIIice Ior 3 hours, discharge without medication iI
examination remains unchanged
O C) Discharge him with oral broad-spectrum antibiotic therapy
O D) Admit him to the hospital Ior observation
O E) Admit him to the hospital Ior intravenous broad-spectrum antibiotic therapy

EEEEEEEEEEEEEEEEEEEEEE



30. A 72-year-old woman is brought to the emergency department 1 hour aIter the
sudden onset oI right Iacial droop and weakness oI the right arm and leg. She takes
captopril Ior hypertension and daily aspirin. Her blood pressure is 150/90 mm Hg,
pulse is 80/min, and respirations are 161min. Examination shows a leIt carotid bruit
and right central Iacial paralysis. There is moderate expressive aphasia. A CT scan oI
the head shows no abnormalities. Which oI the Iollowing is the most appropriate
initial pharmacotherapy?
O A) Oral ticlopidine
O B) Oral warIarin
O C) Sublingual niIedipine
O D) Intravenous nitroprusside
O E) Intravenous tissue plasminogen activator




EEEEEEEEEEEEEEEEEEEEEE/




31. A previously healthy 37-year-old woman comes to the physician because oI a 3-
month history oI episodes oI severe anxiety, shortness oI breath, palpitations, and
numbness in her hands and Ieet. Her vital signs are within normal limits. Physical
examination shows no abnormalities. Thyroid Iunction studies and an ECG show no
abnormalities. Which oI the Iollowing is the most appropriate pharmacotherapy?
A) Lithium carbonate
B) Methylphenidate
O C) Olanzapine
p D) Paroxetine
0 E) Valproic acid

DDDDDDDDDDDDDD



32. A 37-year-old nulligravid woman comes to the physician because she has not been
able to conceive Ior 2 years. Her 40-year-old husband has a child by a previous
marriage. Her last menstrual period was 6 weeks ago, menses have occurred at
increasingly inIrequent intervals over the past year. FiIteen years ago, she was treated
Ior one episode oI Chlamydia trachomatis inIection. She weighs 59 kg (130 lb) and is
170 cm (67 in) tall. Examination shows no abnormalities. Serum studies show:

R-hCG 5 mlUImL
Follicle-stimulating hormone 50 mlU1mL
Luteinizing hormone 45 mlUImL
Prolactin 13 ngImL
Thyroid-stimulating hormone 3 pU/mL

Which oI the Iollowing is the most likely cause oI this patient's inIertility?

O A) Hypothalamic amenorrhea
O B) Hypothyroidism
O C) Pituitary adenoma
O D) Polycystic ovarian disease
O E) Premature ovarian Iailure
0 F) Tubal Iactor


EEEEEEEEEEEEEEEEE

33. A 47-year-old woman comes to the physician because oI a 2-day history oI Iever
and joint pain. Six days ago, she completed a 10-day course oI dicloxacillin Ior
Iolliculitis. She appears comIortable. Her temperature is 38.6C (101.5E), pulse is
721min and regular, respirations are 161min, and blood pressure is 120176 mm Hg.
Examination shows an urticarial rash over the trunk and extremities. There is
moderate generalized lymphadenopathy and diIIuse joint tenderness. The remainder
oI the examination shows no abnormalities. Which oI the Iollowing is the most likely
cause oI these symptoms?
O A) Epstein-Barr virus inIection
O B) IgE-mediated allergic reaction
O C) Mycobacterium haemophilum inIection
O D) Serum sickness
0 E) Staphylococcal sepsis

DDDDDDDDDDDDDDDDDDDD


34. A 34-year-old man is brought to the emergency department by his Iamily because
oI a decreased need Ior sleep, increased goal-directed activity, increasingly irritable
and unstable mood, and uncharacteristic sexual promiscuity over the past week.
Yesterday, he quit his long-standing job at a hardware store and plans to move to
Washington, DC, to become the "Senator-in-ChieI." He says that this position is being
created especially Ior him by the president. He believes that he has developed
supersensitive hearing and is now able to hear conversations oI people hundreds oI
miles away. He has no history oI drug use, and there is no personal or Iamily history
oI psychiatric illness. On examination, he is extremely irritable and agitated,
threatening to "punch out anyone who tries to mess with me." Which oI the Iollowing
is the most likely diagnosis?
O A) Bipolar disorder
O B) Cyclothymic disorder
O C) Delusional disorder
O D) Schizophrenia
O E) SchizophreniIorm disorder

AAAAAAAAAAAAAAAAAAAAAAAA

.


35. A 17-year-old girl has had Ilu-like symptoms, low-grade Iever, and malaise Ior 3
days and mild jaundice Ior 2 days. Serum studies show an aspartate aminotransIerase
(AST, GOT) activity oI 670 UIL and an alanine aminotransIerase (ALT. GPT)
activity oI 860 U/L. Serum IgM antibody to hepatitis A is positive. Which oI the
Iollowing is most likely to minimize the risk Ior this disease in Iamily members?
O A) Acyclovir therapy Ior Iamily members
O B) Hepatitis B vaccination Ior Iamily members
O C) Immune globulin therapy Ior Iamily members
OD) InterIeron alIa-2b therapy Ior Iamily members
0 E) Use oI separate toilet Iacilities by the patient

CCCCCCCCCCCCCCCCC


36. One day aIter missing her dialysis treatment, a 27-year-old woman comes to the
physician because oI generalized weakness. Her blood pressure is 95172 mm Hg, and
pulse is 45/min and regular. Examination shows a well-Iunctioning arteriovenous
Iistula in the leIt upper extremity. Muscle strength is 315 in all extremities. An ECG
shows third-degree atrioventricular block. Laboratory studies are ordered. Which oI
the Iollowing is the most appropriate next step in management?
O A) Kayexalate-sorbitol enema
O B) Intravenous administration oI calcium gluconate
O C) Oral administration oI sodium polystyrene sulIonate (Kayexalate)
O D) Immediate placement oI a pacemaker
O E) No treatment pending laboratory results


DDDDDDDDDDDDDDDDDD

37. A previously healthy 2-year-old boy is brought to the physician because oI Iever
and abdominal pain Ior 24 hours. His developmental milestones are appropriate Ior
age. He is Iully alert and responsive. His temperature is 38.6C (101.5F), blood
pressure is 85/60 mm Hg, pulse is 1001min, and respirations are 20/min. Examination
shows suprapubic tenderness to deep palpation but no guarding or rebound. There are
no palpable abdominal masses or costovertebral angle tenderness. The penis is
circumcised, there is no urethral discharge. Urinalysis shows 20-30 leukocytes/hpI, 5-
6 erythrocytes/hpI, and nitrites. A urine culture grows 100,000 colonies/mL oI
Escherichia coli sensitive to all tested antibiotics. Amoxicillin therapy is initiated.
Five days later, he is aIebrile and asymptomatic. Ultrasonography oI the kidneys
shows no abnormalities. Which oI the Iollowing is the most appropriate next step in
management?

O A) Discontinue the antibiotic in 2 days and reexamine only iI symptoms recur
O B) Voiding cystourethrography
O C) Intravenous pyelography
O D) Cystoscopy
O E) No Iurther testing


BBBBBBBBBBBBBBBBBBBBBBB

38. A 62-year-old woman comes to the physician because oI increasingly severe low
back pain over the past month. Prolonged periods oI rest have not relieved her
symptoms. Sensorimotor examination oI the lower extremities shows no
abnormalities. Her hematocrit is 32, and serum calcium level is 12 mgIdL. An x-ray
Iilm oI the chest shows no abnormalities. An x-ray Iilm oI the spine is shown. Which
oI the Iollowing is the most likely diagnosis?
O A) Herniated nucleus pulposus
O B) Mechanical low back pain
O C) Metastatic carcinoma
O D) Osteopetrosis
0 E) Spinal stenosis


ccccccccccccccccc


39. A 43-year-old man comes to the physician Ior evaluation and management oI
cardiac risk Iactors 8 weeks aIter sustaining a myocardial inIarction. He takes aspirin
and metoprolol daily, and he does not smoke cigarettes. His Iather and brother both
had myocardial inIarctions beIore the age oI 50 years, their serum cholesterol levels
are unknown. There is no Iamily history oI diabetes mellitus. He weighs 86 kg (190
lb) and is 180 cm (71 in) tall. His blood pressure is 130170 mm Hg, pulse is 68/min,
and respirations are 14/min. Two years ago, his serum cholesterol level was 245
mg/dL. Fasting serum glucose level is 88 mg/dL. Which oI the Iollowing is the most
appropriate next step to evaluate his cardiac risk Iactors?


O A) Random measurements oI serum cholesterol level
O B) Measurement oI Iasting serum cholesterol level only
O C) Fasting serum lipid studies only
O D) Oral glucose tolerance test and Iasting serum lipid studies
O E) Oral glucose tolerance test and measurement oI Iasting serum cholesterol level

CCCCCCCCCCCCCCCCCCCC



40. A 47-year-old man comes to the physician because oI a 3-week history oI
increasing Iacial swelling and a 1-week history oI morning headaches and mild
shortness oI breath. He had previously Ielt well. He completed a course oI
chemotherapy 4 months ago Ior small cell carcinoma oI the lung. His temperature is
37.2C (98.9F), blood pressure is 142180 mm Hg with an 8-mm Hg paradoxical
pulse, pulse is 84/min, and respirations are 18/min. Examination shows signiIicant
diIIuse Iacial and periorbital edema. The optic discs are sharp, and ocular movements
are intact. The lungs are hyperresonant bilaterally with a moderately prolonged
expiratory phase. Mild rhonchi are heard on inspiration and expiration. An x-ray Iilm
oI the chest shows a 10-cm mass in the right upper lobe and apex. Which
oI the Iollowing is the most likely explanation Ior these Iindings?
O A) Chemotherapy-induced bone marrow toxicity
O B) Chemotherapy-related cardiac toxicity
O C) Hypercoagulable state secondary to malignancy
O D) Interstitial metastatic pulmonary disease
O E) Intracranial metastases
O F) Lymphatic obstruction
O G) Malignant pericarditis
O H) Paraneoplastic syndrome
O I) Pleural metastases
O J) SoIt tissue metastases
0 K) Vascular obstruction


KKKKKKKKKKKKKKKKK


41. A 13-month-old girl is brought Ior a well-child examination. There is no history oI
prenatal or perinatal problems. Her diet consists oI breast milk, juice, legumes,
cooked vegetables, and Iruit. She has had three ear inIections treated successIully with
amoxicillin and two episodes oI diarrhea since birth. She will pull to a stand and stand
alone Ior 1 minute but has not begun to walk. She is able to pick up raisins with her
thumb and IoreIinger but is unable to Ieed herselI with a spoon. Which oI the
Iollowing is the most appropriate
assessment oI Iine and gross motor development?

`` Fine Motor Gross Motor
Development Development
O A) Normal normal
O B) Normal delayed
O C) Delayed normal
O D) Delayed delayed

AAAAAAAAAAAAAAAAAAA

42. An 18-month-old girl is brought to the physician because oI intermittent shortness
oI breath over the past 6 months. Her mother notes that her daughter oIten squats to
relieve her symptoms. Examination shows cyanosis and clubbing oI the Iingers. A
prominent right ventricular impulse with a grade 3/6, systolic ejection murmur is
heard best in the third leIt intercostal space. An ECG shows right ventricular
hypertrophy. An x-ray Iilm oI the chest shows a small-sized heart and decreased
pulmonary blood Ilow. Which oI the Iollowing is the most likely diagnosis?
O A) Atrial septal deIect (ostium primum type)
O B) Atrial septal deIect (ostium secundum type)
O C) Atrioventricular canal
O D) Coarctation oI the aorta
O E) Hypoplastic leIt heart syndrome
O F) Patent ductus arteriosus
O G) Tetralogy oI Fallot
O H) Transposition oI the great arteries
O I) Tricuspid atresia
0 J) Ventricular septal deIect

GGGGGGGGGGGG


43. A 5-day-old girl has been Ieeding poorly since birth. She weighed 2900 g (6 lb 7
oz) at birth and now weighs 2600 g (5 lb 12 oz). Physical examination shows an
enlarged clitoris and labial Iusion. A sibling born 5 years ago died at 1 week oI age. A
liIe-threatening complication oI this syndrome results Irom Iailure to produce which
oI the Iollowing?
O A) Aldosterone
O B) Cholesterol
O C) Estrogen
O D) Insulin
0 E) Testosterone


AAAAAAAAAAAAAAAAAAA

44. A 57-year-old hospitalized man undergoes right subclavian venous catheterization
Ior hyperalimentation. He is currently being treated Ior a small bowel Iistula. While
the results oI an x-ray Iilm oI the chest to check the catheter position are pending, the
patient suddenly becomes agitated. His blood pressure is 70150 mm Hg, and pulse is
1101min. Examination shows jugular venous distention. The lungs are clear to
auscultation. Breath sounds are equal bilaterally. The trachea is midline. An x-ray
Iilm oI the chest shows a catheter in the superior vena cava, transversing the right
ventricle and crossing to the leIt oI the midline. Which oI the Iollowing is the most
likely cause oI this patient's hypotension?
O A) Air embolism
O B) Pericardial tamponade
O C) Pulmonary artery perIoration
O D) Staphylococcal bacteremia
O E) Tension pneumothorax

BBBBBBBBBBBBBBBBBBBBBB

45. Twenty-Iour hours aIter hospitalization Ior Iixation oI a Iemoral Iracture, a 35-
year-old construction worker has nausea, agitation, and insomnia. His blood pressure
is 1501100 mm Hg. He is oriented but anxious. Which oI the Iollowing is the most
appropriate next step in diagnosis?
O A) Inquire about recent alcohol intake
O B) Examination oI a urine specimen Ior Iat bodies
O C) Leukocyte count with diIIerential
O D) ECG
0 E) Pulmonary arteriography

AAAAAAAAAAAAAAAAAAA

46. A previously healthy 27-year-old woman comes to the physician because oI a 3-
month history oI moderate abdominal pain that improves Ior a short time aIter she
eats. She has not had any rectal bleeding. Her temperature is 36.4C (97.5F), blood
pressure is 110170 mm Hg, and pulse is 80/min. Abdominal examination shows
midepigastric tenderness. Her hemoglobin level is 12 gIdL, and leukocyte count is
80001mm3. Serum Helicobacterpylori antibody assay is positive. Which oI the
Iollowing is the most appropriate next step in management?
O A) HIDA scan
O B) Amoxicillin, clarithromycin, and omeprazole therapy
O C) CiproIloxacin therapy
O D) Omeprazole, magnesium hydroxide, and metoclopramide therapy O E)
Ranitidine therapy


BBBBBBBBBBBBBBBBBBBB




Report Abuse

* Re:Ans key Ior NBME 3
#995132

coolamazon - 10/17/07 02:02


nbme 3,BLOCK 3:

BLOCK 3
1. A 32-year-old man comes to the physician Ior a second opinion regarding an
enlarged cervical lymph node that he noted 6 weeks ago. He has not had pain or
tenderness but has been unable to return to work despite normal Iindings on a biopsy
oI the node 4 weeks ago. He had a mild upper respiratory tract inIection 1 month ago.
His maternal uncle recently died oI lymphoma. He states that cancer runs in his
Iamily, and that Ior the past several years he has thought that he will have some kind
oI cancer eventually. Which oI the Iollowing is the most likely diagnosis?
A) Asthma
B) Conversion disorder
C) Factitious disorder
D) Hypochondriasis
E) Malingering
F) Pulmonary embolus
G) Somatization disorder
Answer: D
2. 32 y/o woman given captopril Ior sev HTN now has stridor 14 hrs later.shes
anxious bp 140 /85 pulse 140 RR 32 .exam shows sweeling oI lips n tongue.diIIuse
stridorous wheezes heard on ausc diminished air movemnt.
most app nxt step in Mx
measure serum captopril
CXR
observe only
measure serum IgE
tracheal intubation
Answer: E
3. Pt with GBS, what to do at delivery? IV penicillin
4. A 22-year-old woman comes to the physician because oI diIIuse constant
headaches and vision problems Ior 3 months; the headaches are worse in the morning
when she awakens Irom sleep. She also has had brieI episodes oI loss oI vision in
both eyes. She has had an 18-kg (40-lb) weight gain over the past year. She now
weighs 100 kg (220 lb) and is 163 cm (64 in) tall. Visual acuity is 20/30 bilaterally.
Visual Iields are Iull, but the blind spots are enlarged bilaterally. Funduscopic
examination shows marked blurring oI the optic disc margins bilaterally. The
remainder oI the neurologic examination shows no abnormalities. Which oI the
Iollowing is the most likely diagnosis?
A ) Amaurosis Iugax
B ) Central retinal vein occlusion
C ) Glaucoma
D ) Macular degeneration
E ) Migraine
F ) Nutritional optic neuropathy
G ) Optic neuritis
H ) Pituitary adenoma
I ) Pseudotumor cerebri
J ) Temporal arteritis
Answer: I
5. a healthy 60 yo Iemale came Ior routine examination. she has no h/o illness or
surgeries since last year.she takes no medications. she attained menopause 6 yrs ago.
she wighs 60 kg and is 160 cm tall. pelvic exam showed atrophic external genitalia
and small midpositioned uterus. the leIt ovary is 3x3cm,rt ovary is not palpable.what
is the next best step in the managment oI this management?
A)re examination in 1 month
B) re examination in 1 year
C) obtain patients medical records
D)measure serum progesterone
E)pelvic ultrasonography.
Answer: C
6. A 47-year-old woman comes to the emergency department because oI severe
abdominal pain Ior 3 hours. The pain began aIter a 2-week drinking binge. She has a
15-year history oI alcoholism. She has no history oI jaundice or hepatitis. Current
medications include multivitamins and iron. Her temperature is 38C (100.4F), blood
pressure is 110/80 mm Hg, and pulse is 110/min. Examination shows mild jaundice
and diIIuse spider angiomata over the trunk and abdomen. The liver is tender to
palpation. A complete blood count shows mild anemia with normal red cell indices.
Ultrasonography oI the abdomen shows normal-sized hepatic ducts. Which oI the
Iollowing is the most likely set oI laboratory Iindings?
Total Indirect Alkaline Alanine
Bilirubin Bilirubin Phosphatase AminotransIerase Reticulocyte
(mg/dL) (mg/dL) (U/L) ALT, GPT) (U/L) Count ()
A)2 0.9 80 30 1.2
B)3 2.8 70 30 1.0
C)3 2.8 80 20 3.0
D)4 2.0 800 200 1.5
E)4 1.0 150 400 1.0
Answer: B
7. 5 years aIter being shot in the thing, a 21yo comes with a buzzing sensation
adjacent to the scar. A the time oI the initial wound, he was discharged aIter 6 hours
oI observation and would cleaning. A loud murmur is hear on auscultation, there is a
thrhill. He has dilated varicose vening with incompetent valvles in the lower legs. Dx?
Arterial spasm, AV Iistula, DVT, Occlusion oI the superIicial Iemoral artery,
Pseudoaneurysm
Answer: B
8. 5 days aIter sustaining a 6cm laceration through the skin and subcutaneous tissue oI
the leIt upper extremity with a clean kniIe, a 52-year-old man has increasing
tenderness in the area oI the laceration. Treatment at the time oI injury included
cleansing and dressing oI the wound. The wound is now erythematous, and yellow
pus is expressed when pressure is applied. Which oI the Iollowing is the most likely
mechanism Ior the accumulation oI pus?
A. Chemotaxis
B. Dysplasia
C. Hyperoxia
D. Metaplasia
E. Vasoconstriction
Answer: A
9. 40 yr male with c/o jaw stiIInes and muscle spasm brought to ER. H/o heroin
abuse. sexually active with multiple partners uses condon inconsistently. temp 38. 3
C. he is irritable. examination shows Iacial and paraspinal muscle spasm and rigidity
and stimulation cause paroxysmal spasm. bp 150/ 96. pulse 120. mobility oI jaw is
decreased and neurological examination shows hyper reIlexia. which oI the Iollowing
have most likely prevented this condition..
1. use oI condom
2. tetanus toxoid vaccination
3. thiamine ( vit B 1)
4.prednisone therapy
5. riIampin therapy
6. botulism antitoxin therapy
Answer: B/2
10. Question about a guy whos wiIe dies and 6 weeks later hes still crying and not
sleeping, but no suicidal ideations Normal Bereavement
11. A 12-year-old girl with chronic renal Iailure has had persistent epistaris tor 15
days. Laboratory studes show
Hemoglobin 7 2g/dL
Platelet count VSMGImm1
Bleeding time 12mm
Prothrombin time 12 sec
Partial thromboplastin Dme (activated) 30 sec
Serum urea nitrogen (BUN) 125mc/dL
Which oI the Iollowing is the most likely cause ol the epistaxis?
A) Acquired platelet dysIunction
B) Circulating immune complexes
C) Erythropoietin deIiciency
D) Factor III deIiciency
E) Hypocalcemia
Answer: B
12. question said Iixed, split S2 ASD
13. 19yo has Iever, sore throat, HA, cervical lymphadenopathy Ior 5 days. Pharynx is
red and swollen with enlarged tonsils and exudates. Tender and palpable axillary and
inguinal lymph nodes as well. Spleen tip is palpable. Leukocyte is 14000 (60
atypical, 15 mono). Pathogen? Adeno, CMV, EBV, Group A Strep, Toxo
Answer: C
14. An 80-year-old man has had poor balance Ior 6 months. He has a history oI
hypertension treated with hydrochlorothiazide. His blood pressure is 136186 mm Hg.
Neurologic examination shows mild tremor oI the hands when his arms are
outstretched and decreased vibratory sensation at the knees. Deep tendon reIlexes oI
the quadriceps and gastrocnemius-soleus muscles are hyperactive. Babinski's sign is
present bilaterally. He is unable to stand with his eyes closed. Which oI the Iollowing
is most consistent with normal age-related changes?
A) Babinski's sign
B) Hyperactive deep tendon reIlexes oI the gastrocnemius-soleus muscles
C) Inability to stand with the eyes closed
D) Reduced vibratory sensation at the knees
E) Tremor oI the outstretched hands
Answer: D
15. Tx Ior bronchospastic episodes occurring at night in someone who already is on
B2 agonist Ior daily cough and wheezing? Inhaled steroid, inhaled ipratropium, oral
erythro, oral Iurosemide and oral theophylline
Answer: A
16. 12 days aIter cerebral inIarct 70 y/o man has Iever and cough .initial Sx included
inability to move Rt arm & leg , swallow, speak ,respond to Q's. he is wearing Ialse
teeth .temp 101.8 bp 135/85 pulse 94 rr 28. exam shows moderate weakness oI Iacial
muscles and Rt extremity.gag reIelx is absent .breath sounds dec...dullness to perc
over Rt lung base posteriorly.CXR shows inIiltrate in post basal segment oI rt lung
.which oI the Iolloiwng is most likely to Prevent recurrence oI this pts lung condition?
a removal oI his Ialse teeth
b suppression oI gastric acid production
c chronic Ab prophylaxis
d metoclopramide to inc GIT motility
e insertion oI Ieeding jejunostomy tube
Answer: A
17. Question said VC is decreased, FEV1 is decreased and ratio oI FEV1 to VC is
increased, all answer were obstructive, only one was restrictive: Pulm Fibrosis
18. a 2 y/o girl Iound Iloating Iacedown in a sweeming pool.on rescue, she
immediately coughs and breath spontaneously. She is conscious, and
oriented..without treatment which oI the Iollowing is most likely outcome oI the
child?
a)complete recovery
b)development oI ARDS
c) hemolysis
d) pulmonary HT
e) severe neurological deIicit
Answer: A
19. A previously healthy 6-month-old boy is brought to the physician because oI a 12-
hour history oI vomiting and diarrhea. He vomits aIter all Ieedings, the vomitus does
not contain blood or bile. His mother says that he has had Iewer wet diapers than
usual during this period. He appears dehydrated and is crying without tears. He is at
the 50th percentile Ior length and 30th percentile Ior weight. He appears lethargic. His
temperature is 38C (100.4F), pulse is 180/min, and blood pressure is 60/40 mm Hg.
Examination shows sunken eyes, dry mucous membranes, and a sunken anterior
Iontanel. Arterial blood gas analysis on room air shows:
pH 7.2
PCO2 38 mm Hg
PO2 90 mm Hg
Which oI the Iollowing is the most likely explanation Ior this patient's arterial blood
gas Iindings?
A) Excessive metabolic acid Iormation
B) Impaired ventilation
C) Increased chloride loss
D) Increased CO2 concentration in the extracellular Iluid
E) Increased metabolic acid produced by the gastrointestinal tract
Answer: C
20. 24hrs aIter splenectomy Ior blunt truama sustained in MVA, previously healthy
25yr old man has oliguria and pain at the incision site. intra operative complication
was 3o min hypotension and total blood loss 2.5L required 4 units oI Packed RBC.
Now temp 100.4F, Bp 120/80 mmhg, PR 100, rep 14, CVP 8 cm H2O. Lungs clear,
breath sound bilaterally.
Abd exam shows no distension, bowel sounds absent, a Ioley catheter is in place and
over the past 3hrs urine output has been 20 ml/hr. Lab show Hct 28, BUN 30 mg/dl,
Cr 2.5 mg/dl, electrolytes normal what's the likely explanation
a. ATN
b Foley catheter malIunction
c Hypovolemia
d. TransIusion reaction
e. ureteral injury
Answer: A
21. guy with 30 stenosis oI internal carotid, next step: Aspirin therapy, Heparin
therapy, NiIedipine therapy, placement oI carotid endovascular stent, carotid
endarterectomy
Answer: A
22. during a routine exam 20 y/o man is Iound to have Iirm bilaterally enlarged
thyroid gland and lymphadenopathy .his Iather died oI tghyroid Ca at 40. which
hormone is most likely associated with this pts Iindings
ACTH
ADH
aldosterone
calcitonin
growth H
Bhcg
insulin
PTH
TSH
T4
Answer: D
23. A previously healthy 24-year-old woman, gravida 3, para 3, is brought to the
emergency department because oI deep, sharp, intermittently severe pain in the leIt
lower quadrant oI the abdomen Ior 2 hours. Her last menstrual period was 3 weeks
ago. She takes no medications. She is in obvious distress and is lying on her leIt side
with her lower extremities drawn up against her abdomen. Her temperature is 37C
(98.6F), blood pressure is 140/70 mm Hg, and pulse is 125/min. Abdominal
examination shows rigidity and tenderness. Pelvic examination shows a 12-cm mass
in the leIt lower quadrant oI the abdomen. Which oI the Iollowing is the most
appropriate next step in management?
A ) Ultrasound-guided aspiration
B ) Dilatation and curettage
C ) Exploratory laparotomy
D ) Hysterectomy
Answer: C
24. One week aIter cholecystectomy Ior acute cholecystitis, a 57-year-old woman
comes to the physician because oI a 1-day history oI abdominal cramps and watery,
green, Ioul-smelling diarrhea. She Iinished a 7-day course oI ceIoxitin 3 days ago. She
appears dehydrated. Her temperature is 39.8C (103.6F), pulse is 115/min, and blood
pressure is 105/70 mm Hg. Abdominal examination shows moderate distention and
diIIuse tenderness; there is no guarding. Test oI the stool Ior occult blood is positive;
the stool contains leukocytes. Her hemoglobin level is 10.8 g/dL, and leukocyte count
is 39,000/mm3. The patient is admitted to the hospital, and administration oI
intravenous Iluids is begun. Which oI the Iollowing is the most appropriate next step
in pharmacotherapy?
A ) Intravenous amphotericin
B ) Intravenous ampicillin, clindamycin, and gentamicin
C ) Intravenous corticosteroids
D ) Oral diphenoxylate and atropine
E ) Oral metronidazole
Answer: E
25. An aIebrile 2-year-old boy has had right ear pain Ior 3 days. He has been
swimming every day Ior the past week. Purulent Iluid is draining Irom the right
external ear canal, and manipulation oI the pinna is painIul. No abnormalities are
noted on visualization oI tympanic membranes. Which oI the Iollowing is the most
likely causal organism?
A) Haemophilus inIluenzae type b
B) Moraxella catarrhalis
C) Pasteurella multocida
D) Pseudomonas aeruginosa
E) Streptococcus pneumoniae
Answer: D
26. 42yo with HTN and 1 hour onset oI chest discomIort, HA, irritability and
conIusion. His BP is 220/148, () papilledema. ECG shows ischemic changes with
LVH. CT scan is negative. Best tx: Atenolol, Captopril, Diltiazem, Furosemide,
Guanethidine, Hydralazine, Losartan, Methyldopa, Nitroprusside, Prazosin, Thiazide
diuretic.
Answer: C
27. 72 y/o woman C/C urinary incontinence 6 months with no senstion to urinate prior
to episodes.the episodes r less Irquent iI she schedules her trips to the bathroom and
restricts liquid intake prior to beddtime.she has 20 Yr Hx DM 2 is on meIormin and
glyburide.has mild retinopathy .pelvic exam nL vaginal mucosa.sesation to pinprick
decreased in glove stocking dist.U/A 1 protein. Which is the most likely cause oI pts
urinary symptoms.
Iunctional incontinence
normal aging
overIlow oI urine Irom large residual volumes
hypersensitivity oI detrusor muscle
urethral atrophy with loss oI uretherovesical angle
Answer: B
28. 57 yr m, 10 yr type 2 DM routine check. He Ieels well. Current medication
enalapril, glyburide. 168 cm tall 185 lb weight; BMI 30. Pulse 60/min, BP 100/70
mmhg. Funduscope soIt and hard exudates. Lab:
HB A1c 12
BUN 23
Cr 1.4
Urine protein 1
Which oI the Iollowing is the most appropriate addition pharmacotherapy?
A. Atenolol
B. Captopril
C. Hydrochlorothiazide
D. MetIormin
E. Verapamil
Answer: D
29. 65 y/o woman
C/C palpitations heat intolerance wt loss 7 lb -Ior 6 months
neck mass -10 yrs
131 i scan shows enlarged thyroid with mutiple areas oI inc and dec uptake
most likely DX
graves
multinod goitre
thyroiditid
toxic adenoma
T3 thyrotoxicosis
Answer: B
30. 18 y/o Iemale knee pain since 5 year.. pain is worse with prolonged sitting and
going up and donw stairs. on examination she has 10 degree hyperextense oI both
elbow.. she become apprehensive when lateral directed pressure is applied.. x ray is
normal
what is the diagnosis??
a) anterior cruciate ligament reconstruction
b) arthroscopic partial meniscectomy
c) quadriceps strengthening excercises
d) sympathetic blockage
e) total knee replacement
I) upper tibial osteotomy
Answer: C
31. A 70-year-old man comes to the physician because oI knee pain that has been
present Ior 10 years. He is unable to comIortably walk Iurther than one block and has
diIIiculty sleeping because oI the pain. He has pain on both medial and lateral sides oI
the knee. Range oI motion is Irom 15 to 100 degrees, there is a bowleg deIormity
when he stands same choices as # 30??
Answer: E
32. A 35-year-old man with a 10-year history oI persistent hallucinations and
delusions comes Ior a Iollow-up examination. He has been treated with haloperidol,
chlorpromazine, and Iluphenazine with minimal relieI oI symptoms. He is currently
taking haloperidol. On examination, he walks slowly with no arm swing and has no
Iacial movements. Which oI the Iollowing is the most likely cause oI his motor
behavior?
A ) Excess limbic -aminobutyric acid activity
B ) Increased serum prolactin level
C ) Nigrostriatal dopamine blockade
D ) Noradrenergic depletion in the Irontal lobe
E ) Ventral tegmental dopamine blockade
Answer: C
33. a study is conducted to investigate prevalence oI past inIection with genital herpes
among students attending a large coeducational midwestern Uni.aIter obtaining
inIormed consent the participants undergo antibody testing Ior HSV2.which oI the
Iollowing is the most imp Iactor in determining validity oI a positive test
the no oI sexual partners oI the student
proportion oI students inIected with HSV
proportion oI studenst with negative tests
proportion oI students with positive tests
the total no oI students screened
Answer: E
O
Permalink Reply by usmle99 on March 13, 2009 at 7:29am
34. Tx Ior GAD Buspiron
35. some nasty vaginal anal warty looking picture it said what is the strongest risk
Iactor crohns, Iamily ho psoriasis, multiple sexual partners, treatment with
metronidazole, treatment with sulIasalazine
Answer: C
36. An 82-year-old woman comes to the physician because oI a 2-month history oI
progressive shortness oI breath with exertion and a 3-week history oI right upper
quadrant abdominal discomIort. She has chronic obstructive pulmonary disease,
hypertension, and rheumatoid arthritis. Medications include methotrexate,
hydrochlorothiazide, naproxen, albuterol metered-dose inhaler, and aspirin.
Examination shows pallor. Breath sounds are decreased. The liver is enlarged, Iirm,
and mildly tender. Test oI the stool Ior occult blood is positive. Her hematocrit is
27, and mean corpuscular volume is 76 /m3. A CT scan oI the abdomen shows
multiple hypodense lesions in the liver. Which oI the Iollowing is the most likely
diagnosis?
A ) Cholangiocarcinoma
B ) Hepatocellular carcinoma
C ) Lymphoma
D ) Metastatic colon cancer
E ) Metastatic lung cancer
F ) Metastatic pancreatic cancer
Answer: D
37. An 80 year old man is brought to ED on an 80 degree day 2 hrs aIter running a
syncopal episode while playing golI. He plaus golI twice weekly. An ECG obtained at
his last examination 6 months ago showed occassional premature ventricular
contractions. On arrival his temp is 38C(100.4F), Q3. BP is 150/90mmHg, pulse is
80/min and regular, respirations are 22/min. Cardiac examination shows grade 3/6
late-peaking systolic ejection murmur. No bruits are heard over the carotid arteries,
but the upstrokes are delayed. Which oI the Iollowing is the most likely cause oI the
syncope.
a. Aortic stenosis
b. Heat Stroke
c. Hypertensive cardiomyopathy
d. Platelet emboli
e. PVCs
Answer: A
38. A 38-year-old woman comes to the physician because oI a low-grade Iever and
generalized rash Ior 4 days. She is currently receiving ceIazolin therapy Ior chronic
osteomyelitis. Her temperature is 38.2C (100.8F), blood pressure is 1501108 mm
Hg, and pulse is 1001min. There is a Iaint diIIuse maculopapular rash. Examination oI
the back shows no costovertebral angle tenderness. Cardiac and pulmonary
examinations show no abnormalities. Laboratory studies show:
Leukocyte count 10,8001mm3
Segmented neutrophils 60
Bands 8
Eosinophils 4
Lymphocytes 20
Monocytes 8
Serum
Urea nitrogen (BUN) 20 mg/dL
Creatinine 1.6 mg/dL
Urine
WBC 121hpI
RBC 81hpI
RBC casts none
WBC casts rare
Eosinophils are Iound in the urine sediment. Which oI the Iollowing is the most likely
explanation Ior these Iindings?
A) Acute tubular necrosis
B) Fibromuscular dysplasia
C) Interstitial nephropathy
D) Polyarteritis nodosa
E) Pyelonephritis
F) Wegener's granulomatosis
Answer: C
39. 43 y old man with a 8 year history oI inter. diIIiculty oI swallowing solids and
liquids has had increasingly severe exacerberation since 6 months due to stress. he has
nocturnal regurgitation and cough. esophageal motility show Iailure oI relaxation oI
lower esophageal sphincter.what is the diagnosis
1) achalasia
2) sqamous cell ca
3) adenocarcinoma
4) reIlux esophagitis
5) diIIuse esophageal spasm
6) viral esophagitis
7) candidial esophagitis
8) scleroderma
Answer: A
40. A 45-year-old woman has had generalized weakness Ior 1 year, increasing
diIIiculty walking up stairs over the past 6 months, and diIIiculty swallowing Ior 1
month. Examination shows weakness oI the proximal muscles oI the extremities. Ear,
nose, and throat examinations and manometry show decreased contractions oI the
pharynx with decreased upper esophageal tone. Same answers as question 39?
Answer: 8
41. A 6-month-old girl is brought to the physician because oI Iever, cough, and coryza
Ior 1 day. Her pulse is 100/min, and respirations are 50/min. Her cough is harsh and
sounds like a dog's bark. There is inspiratory stridor and intercostal retractions. Which
oI the Iollowing is the most likely diagnosis?
A ) Asthma
B ) Bacterial tracheitis
C ) Bronchiolitis
D ) Foreign body in the small airways
E ) Foreign body in the trachea
F ) Laryngotracheobronchitis
G ) Pneumonia
H ) Pneumothorax
I ) Pulmonary edema
Answer: F
42. On the 5th day oI a 7 day cruise in the western caribbean, 37 yr old women
develops headache, Iever, chills, abddiscomIort, ans watery diarrhea over 12 hr
period. Many other passengers and crew haave had similar syptoms.
PE: tept 98.6 F abd exam shows diIIuse tenderness without rebound, bowel sounds
are hyperactive. Leucocytes 11,000, Gm stain oI stool shows small # oI neutrophils, a
stool culture grows Salmonella enteritidis, 2 days aIter Rx with bismuth susalicylates
and oral rehydration, her syptoms subsides.
what's appropritae immediate measure to prevent Iurther spread oI pathogens?
a. cancellation oI shore leave Ior all crew members
b. elimination oI seaIood and shellIish Irom the ship menu
c. exclusive use oI pasteurized eggs
d. hyperchlorination oI the ship's drinking water
e. reaasignment oI all the Iood handling personnel to other duties
I. Rx oI all aIIected persons with doxycycline
Answer: C
43. Over the past 2 weeks, a 60-year-old man has had shortness oI breath on exertion.
He also has paroxysmal nocturnal dyspnea with two-pillow orthopnea. He has taken
aspirin daily since a myocardial inIarction 3 years ago. He has a history oI atrial
Iibrillation well controlled with digoxin and type 2 diabetes mellitus treated with diet.
His blood pressure is 136188 mm Hg, pulse is 98/min and irregular, and respirations
are 20/min. Jugular-venous pressure is increased. Breath sounds are decreased over
the right lung base.. there is dullness to percussion. Cardiac examination shows an S.
gallop. There is 2 edema oI the lower extremities. Pulse oximetry shows an oxygen
saturation oI 90. Which oI the Iollowing is the most appropriate next step in
diagnosis?
A) X-ray Iilm oI the chest
B) Ambulatory ECG monitoring
C) Thallium stress test
D) Echocardiography
E) Ventilation-perIusion lung scans
Answer: A
44. A 3-month-old boy is brought to the physician in January because oI diIIiculty
breathing, clear nasal discharge, and cough Ior 24 hours. His temperature is 37.6C
(99.6F), blood pressure is 88/54 mm Hg, pulse is 168/min, and respirations are
60/min. Bilateral wheezing, prolonged expiration, and a grade 2/6 systolic murmur
along the leIt sternal border are heard. The liver is palpated 3 cm below the right
costal margin. An x-ray Iilm oI the chest shows bilateral hyperinIlation and no
cardiomegaly. Which oI the Iollowing is the most likely diagnosis?
A) Adenovirus pneumonia
B) Congestive heart Iailure
C) InIluenza A virus pneumonia
D) Respiratory syncytial viral bronchiolitis
E) Staphylococcal pneumonia
F) Status asthmaticus
Answer: D
45. Old guy with periIollicular inIlammation, purpura and conIused Scurvy
46. A 55-year-old man with alcoholic cirrhosis is hospitalized Ior 2 weeks in April Ior
treatment oI gastrointestinal bleeding. His last immunization Ior diphtheria-tetanus
(Td) was 6 years ago. Which oI the Iollowing is the most appropriate management
prior to discharge?
A) Administration oI immune globulin
B) Inactivated poliovirus vaccine
C) InIluenza virus vaccine
D) Pneumococcal vaccine
E) Td toxoids
Answer: C

Report Abuse



* Re:All questions to nbme 3 - plz check my answers!
#907497
drjiggy - 09/03/07 15:16

BLOCK 4:
1. Six days aIter undergoing a laparoscopic cholecystectomy Ior acute cholecystitis, a
35-year-old woman comes to the physician because oI Iever and abdominal pain Ior 3
days. She is jaundiced. Her temperature is 38C (100.4F). Abdominal examination
shows distention and incisions that are healing normally. Leukocyte count is
12,000/mm3, and total serum bilirubin level is 7.9 mg/dL. Which oI the Iollowing is
the most likely cause oI the jaundice?
A) Anesthetic-related hepatitis
B) Common bile duct injury
C) Fulminant hepatic Iailure
D) Reaction to perioperative antibiotics
E) Subhepatic abscess
Answer: B
2. 47yo comes because oI increasing Iacial swelling 1 week oI morning HA and mild
SOB. He previously Ielt well. He Iinished Chemotx Ior small cell ca oI lung 4 months
ago. Temp is 98.9. There is diIIuse Iacial and periorbital edema. Dx SVC syndrome
3. The crude mortality rate Ior coronary artery disease in Community A is twice the
crude mortality rate Ior coronary artery disease in Community B. The age-
standardized mortality rates Ior coronary artery disease in the two communities are
the same. These Iindings are most consistent with which oI the Iollowing inIerences?
A ) CAD mortality in the elderly is higher in Community A than Community B
B ) CAD mortality in young adults is higher in Community A than Community B C )
The population oI Community B is younger than the population oI Community A
D) The proportionate mortality Irom CAD is higher in Com A than Community B
E ) The two communities have similar age distribution
Answer: C
4. 44 y/o woman brought to emergency 40 minutes aIter being stabbled in lIt groin.bp
128/84 pulse 100 RR 16 .exam shows small hematoma and no external
bleeding.popliteal and pedal pulses are palpablein lIt lower ext below injury
.laceartions oI Iem art n vein r Iound and repaired.2 days post op she has progresively
sev pain oI lIt lower ext and swelling oI leg Irom knee downward..wats most likely
cause oI her new Sx?
lymphatic disruption Irom injury
thrombosis oI Iem vein
wound hemorrage
compartment syndrome
arterial embolism
Answer: D
5. Crazy guy with crazy thoughts Ior one week SchizophreniIorm disorder
6. A 23-year-old woman, gravida 2, para 0, has had vaginal spotting and abdominal
cramps Ior 2 days. Her last menstrual period was 8 weeks ago. A home pregnancy test
was positive 2 weeks ago. She underwent a salpingectomy 4 years ago Iollowing an
ectopic pregnancy. Examination shows a closed cervix, an enlarged uterus, and no
adnexal masses. Transvaginal ultrasonography shows an empty uterus. Serum -hCG
level is 8000 mIU/mL.
A) Abortion oI a blighted twin
B) Cervicitis
C) Ectopic pregnancy
D) Focal decidual necrosis
E) Gestational trophoblastic disease
F) Incomplete abortion
G) Ovarian torsion
H) Ruptured ovarian cyst
I) Threatened abortion
Answer: C
7. A 25-year-old woman has had painless vaginal postcoital bleeding Ior 2 hours. Her
last menstrual period was 10 weeks ago. A pregnancy test is positive. There is a small
amount oI dark blood in the vaginal canal. She has a reddened, Iriable cervical os.
Same answers as question 6.
Answer: I
8. A 72-year-old woman is brought to the emergency department 1 hour aIter the
sudden onset oI right Iacial droop and weakness oI the right arm and leg. She takes
captopril Ior hypertension and daily aspirin. Her blood pressure is 150/90 mm Hg,
pulse is 80/min, and respirations are 16/min. Examination shows a leIt carotid bruit
and right central Iacial paralysis. There is moderate expressive aphasia. A CT scan oI
the head shows no abnormalities. Which oI the Iollowing is the most appropriate
initial pharmacotherapy?
A ) Oral ticlopidine
B ) Oral warIarin
C ) Sublingual niIedipine
D ) Intravenous nitroprusside
E ) Intravenous tissue plasminogen activator
Answer: E
9. A 50-year-old man has a cardiac arrest during sexual intercourse. He is promptly
resuscitated and recovers uneventIully Irom an acute anterior wall myocardial
inIarction. At discharge Irom the hospital, he and his wiIe inquire about the saIety oI
Iuture sexual intercourse. Which oI the Iollowing is the most appropriate advice?
A ) Psychiatric consultation is necessary
B ) Sexual intercourse should be perIormed while wearing an ambulatory ECG
monitor
C ) Sexual intercourse will be saIe when exercise tolerance improves
D ) They should wait 3 months beIore having sexual intercourse
E ) They should have sexual intercourse as soon as possible to overcome the Iear oI
another event
Answer: E (ReIer to Kaplan, says sex can start right aIter discharge)
10. 28 y/o man has dry hacking cough Ior 3 months.his paternal grandpa had colon Ca
exam NL ..CXR large anterior mediastinal mass.wats most likely origin oI this
neoplasm?
colon
kidney
prostate
stomach
testicle
Answer: E
11. Easy question on Tetralogy oI Iallot. Question said the kid squats and is cyanotic c
RVH.
12. Guy with cough, already on amoxicillin add steroid
13. Question oI a guy with recurrent ulcers, MCV is 78, wants to know what serum
iron, TIBC, saturation oI TIBC and Ierritin will be like (Low, High, High, Low)
14. A 3-year old boy with acute lymphoblastic leukemia has had Iever Ior 3 days. He
completed his last course oI chemotherapy 6 days ago. He has had no malaise, rash, or
anorexia and has had no known contact with sick children in preschool. He appears
well. His T is 38.6 degree, BP is 75/60mm Hg, pulse is 100/min, and respiration are
22/min. Examination shows normal Iindings. Laboratory studied show: Hb 10.1
(n11.5-15.5)
Leukocyte count 2200/mm3, segmented neutrophils 5, Bands 1, lymphocytes
65, atypical lymphocytes 11, monocytes 18, platelet count 35000/mm3. Which
oI the Iollowing is the most appropriate next step?
A. Schedule a Iollow-up visit and discharge without medication
B. Observe him in the oIIice Ior 3 hours, discharge without medication iI examination
remains unchanged
C. Discharge him with oral broad-spectrum antibiotic therapy
D. Admit him to the hospital Ior observation
E. Admit him to the hospital Ior intravenous broad-spectrum antibiotic therapy
Answer: E
15. a 15 y/o boy comes to the physician because oI acne over his Iace Io 1 year. He
preIers not to use any oral medication. Examination shows 10 to 15 pustules. which oI
the Iollowing is the most eIIective treatment Ior this patient`s symptoms
a) Avoidance oI chocolate, Iatty Ioods, and caIIeine
b) Use oI benzoyl peroxide soap
c) Vigorous scrubing oI aIIected areas
d) Application oI vitamin A and E oI aIIected areas
e) Ultraviolet light therapy
Answer: B
16. Vegetarian, MCV is 122 B12
17. 24 hrs aIter the Iixtation oI Iemur Iracture, 35 yr old construction worker develops
insomnia, nausea ,agitation. his bp is 150/100, oriented but anxious.which oI the
Iollowing is the most appropriate next step in diagnosis?
a. inquire about recent alcohol intake
b. ecg
c. pulm.arteriography
d. examination oI urine specimen Ior Iat globule
e.leukocye count wit diIIerential.
Answer: A
18. a 12 y/o grl is brought to th e physician by her mother because she is concerned
about her doughters diIIiculty making Iriends and socialising. her development and
adjustment has been normal until 6 months ago, whe she began to reIuse to use
restrooms at school or eat in the caIeteria. her mother describes her as a quiet serious
child who does not readily interact with other people. her teachers report that her
concentration varies, at times she appears to be daydreaming. on examination, she is
reserved but pleasant and appears to be oI normal inteligence. her speech is normal in
rate and rythm. she says that se is concerned that her voice will Iail her iI she has to
read aloud in class. which oI the Iollowing is the most likely diagnosis?
a) adhd, innatentive type.
b) autistic disorder
c) expressive language dissorder
d) oppositional deIiant disorder
e) selective mutism
I) social phobia
g) age appropriate behavior
Answer: F
19. Loss oI sensation oI last two Iingers Ulnar nerve at the elbow
20. a 68 y/o woman with terminal metastatic breast cancer is living with her son. She
has a living will requesting that she be allowed to die in peace. she is unresponsive to
voice and has not had any Iood Ior 3 days. Her son disagrees her desicion not to
accept Iurther therapy, including, chemotherapy, antibiotics, hospitalization and
enteral or parenteral nutrition. which oI the Iollowing is the most appropriate next step
in management?
a) abide to the decisions oI the next oI kin.
b) obtain a court order to allow additional therapy to be given
c) start enteral Ieedings
d) admit her to the hospital
e) no Iurther intervention
Answer: E
21. a 17 y/o girl has had Ilu-like symptoms, low grade Iever and malaise Ior 3 days
and mild jaundice Ior 2 days. serum studies show: AST: 670, ALT: 860,. SERUM
IgM Ior hepatitis A is positive. which oI the Iollowing is most likely to minimize the
risk Ior this diseace in Iamily members?
a) acyclovir therapy Ior Iamily members.
b) Hepatits B vaccination Ior Iamily members
c) Immune globuline therapy Ior Iamily members
d) interIeron alIa 2b Ior Iamily members
e) use oI separate toilet Iacilities by the patient
Answer: C
22. A 43-year-old man comes to the physician Ior evaluation and management oI
cardiac risk Iactors 8 weeks aIter sustaining a myocardial inIarction. He takes aspirin
and metoprolol daily, and he does not smoke cigarettes. His Iather and brother both
had myocardial inIarctions beIore the age oI 50 years; their serum cholesterol levels
are unknown. There is no Iamily history oI diabetes mellitus. He weighs 86 kg (190
lb) and is 180 cm (71 in) tall. His blood pressure is 130/70 mm Hg, pulse is 68/min,
and respirations are 14/min. Two years ago, his serum cholesterol level was 245
mg/dL. Fasting serum glucose level is 88 mg/dL. Which oI the Iollowing is the most
appropriate next step to evaluate his cardiac risk Iactors?
A ) Random measurements oI serum cholesterol level
B ) Measurement oI Iasting serum cholesterol level only
C ) Fasting serum lipid studies only
D ) Oral glucose tolerance test and Iasting serum lipid studies
E ) Oral glucose tolerance test and measurement oI Iasting serum cholesterol level
Answer: B
23. Guy with HBV Ior 25 yearrs, Increased AFP Dx? HCC
24. a 5 y/o girl has been Ieeding poorly since birth. she weghted 2900 gr at birth and
now weights 2600 gr. physical exam shows an enlarged clitoris and labial Iusion. A
sibling born 5 years ago died al 5 years oI age. A liIe threatening complication oI this
syndrome results Irom Iailure to produce which oI the Iollowing?
a) aldosterone
b) cholesterol
c) estrogen
d) insulin
e) testosterone
Answer: A
25. a previously healthy 3 y/o boy presents with Iever abd pain Ior 24 hrs.milestones
app Ior age ..Iully alert responsive. 101.5 temp ... 85/60 bp ...100 pulse RR 20 .exam
shows suprapubic tenderness but no gaurding or rebound.no and masses or costv
angel tendernes.no urethral discharge,
U/A shows 20- 30 leuk 5,6 eryth culture grows 100,000 colonies e coli all sensitive to
ab's..amox is initiated ..5 days later hes aIebrile asx U/S kidneys NL ..which oI teh
Iollowing is most app nxt stp Mx?
discontinue Ab's in 2 days and reexamine only is Sx recur
VCUG
IVP
cystoscopy
no Iurther testing
Answer: B
26. Two hours aIter emergency cholecystectomy, a 48-year-old woman has an oxygen
saturation oI 84 and a PO2 oI 56 mm Hg on 2 L/min oI oxygen via nasal cannula.
Her blood pressure is 120/80 mm Hg, and respirations are 16/min. Decreased breath
sounds are heard on the right, and there is decreased excursion on inspiration
bilaterally. There is minimal dullness over the right base. Examination shows no
jugular venous distention or pedal edema. An x-ray Iilm oI the chest shows increased
density over the right lower lung Iield; the tracheal, mediastinal, and cardiac
silhouettes are shiIted to the right. The right lung Iield appears considerably smaller
than the leIt lung Iield. Which oI the Iollowing is the most likely cause oI these
Iindings?
A ) Atelectasis
B ) Pleural eIIusion
C ) Pneumonia
D ) Pneumothorax
E ) Pulmonary embolism
Answer: A
27. Picture oI a hyphema
O A)Administration oI BCG vaccine
O B)Administration oI an inhaled bronchodilator
O C)Administration oI oxygen
O D)Arterial blood gas analysis
O E)Bronchoscopy
O F)Culture oI the pharynx Ior bacteria
O G)Direct laryngoscopy
O H) Gastric washings Ior acid-Iast bacteria
O I) Intravenous inIusion oI saline
O J) Isoniazid and riIampin therapy
O K) Lumbar puncture
O L) Measurement oI serum aspirin level
O M) Subcutaneous administration oI epinephrine.
N) Viral culture oI respiratory secretions
00) X-ray Iilms oI the chest
O P) X-ray Iilms oI the neck


28. A previously healthy 8-year-old boy is brought to the emergency department
because oI swelling oI the lips and diIIiculty breathing Ior 20 minutes. The symptoms
began when he was helping his Iather clean the gutters on their house. He appears
anxious. His temperature is 37.2C (99F), pulse is 120/min, and respirations are
50/min. Pulse oximetry shows an oxygen saturation oI 96. His lips and eyes appear
puIIy. He has subcostal and intercostal retractions. Auscultation oI the chest shows
diIIuse bilateral wheezing.
Answer: M

29. A 16-year-old girl is brought to the emergency department because oI heavy
breathing Ior 8 hours, and vomiting and ringing in the ears Ior 1 hour. She recently
broke up with her boyIriend and has been threatening to hurt herselI. She appears
somnolent but is arousable and answers questions appropriately. Her temperature is
37.5C (99.5F), pulse is 88/min, and respirations are 50/min. Pulse oximetry shows
an oxygen saturation oI 96. She has no retractions, nasal Ilaring, or cough. The
lungs are clear to auscultation
Answer: L

30. Guys PPD came back over 15mm. Next step - CXR
28. Question on dermatomyosits
29. Easy Q Tx Ior H.pylori Amox, Clarithromycin, Omeprazole
30. A guy taking doxorubicin gets S3 and crackles Cardiotoxicity
31. Picture oI spine with degenerative changes and pt has high calcium. Dx?
Herniated nucleus polposus, mechanical low back pain, met carcinoma, osteopetrosis,
spinal stenosis
32. A 37-year-old nulligravid woman comes to the physician because she has not been
able to conceive Ior 2 years. Her 40-year-old husband has a child by a previous
marriage. Her last menstrual period was 6 weeks ago; menses have occurred at
increasingly inIrequent intervals over the past year. FiIteen years ago, she was treated
Ior one episode oI Chlamydia trachomatis inIection. She weighs 59 kg (130 lb) and is
170 cm (67 in) tall. Examination shows no abnormalities. Serum studies show:
-hCG5 mIU/mL
Follicle-stimulating hormone 50 mIU/mL
Luteinizing hormone 45 mIU/mL
Prolactin 13 ng/mL
Thyroid-stimulating hormone 3 U/mL
Which oI the Iollowing is the most likely cause oI this patient's inIertility?
A) Hypothalamic amenorrhea
B) Hypothyroidism
C) Pituitary adenoma
D) Polycystic ovarian disease
E) Premature ovarian Iailure
F) Tubal Iactor
Answer: E
33. An 82-year-old man is brought to the emergency department because oI the
sudden onset oI conIusion 48 hours ago. His Iamily says that he previously had been
Iunctional and independent. His temperature is 37.5C (99.5F), blood pressure is
110/70 mm Hg, pulse is 90/min, and respirations are 12/min. The skin is warm and
clammy. Bowel sounds are hypoactive, and there is guarding and tenderness in the
right lower quadrant oI the abdomen. Examination oI the heart, lungs, and extremities
shows no abnormalities. He is drowsy but arousable and oriented to person but not
place or time. There are no Iocal deIicits. Which oI the Iollowing is the most likely
diagnosis?
A ) Appendicitis
B ) Colon cancer
C ) Meningitis
D ) Pneumonia
E ) Urinary tract inIection
Answer: B (probable A though)

34. A 6-year-old girl is brought to the physician because oI a 1-day history oI
vomiting, headache, and weakness. One month ago, she underwent resection oI an
astrocytoma and placement oI a ventriculoatrial shunt Ior residual hydrocephalus; Her
postoperative course had been uncomplicated. Currently, she is drowsy and irritable.
Her T 37.5 degree, BP 126/54 mmHg, pulse is 82/min, and respirations are 24/min.
Funduscopic examination shows papilledema. ReIlexes are brisk with hypertonia in
the lower extremities. Which oI the Iollowing is the most likely underlyiong
mechanism Ior these symptoms?
A. Carcinomatous inIiltration oI cerebrospinal Iluid
B. MalIunction oI ventriculoatrial shunt
C. Overproduction oI cerebrospinal Iluid Irom a choroid plexus papilloma
D. Recurrence and extension oI the astrocytoma
E. Thrombosis oI the superior sagittal sinus
Answer: B

35. A 30 year old woman, g4p3, is admitted to the hospital in labor at 38 weeks
gestation. The cervix is 4 cm dilated. Contractions occur every 4 minutes. The Iundal
height is 40 cm, the membranes are intact. The Ietal lie cannot be determined by abd
examination, and no presenting part is palpable in the pelvis. Which oI the Iollowing
is the most appropriate next step in management?
A. Ultrasonography
B. Administration oI oxytocin
C. Administration oI tocolytic drugs
D. Amniotomy
E. Cesarean delivery
Answer: A
35. See above
36. A previously healthy 30-year-old woman has had a painless lump in her neck Ior 2
days. Her mother was treated Ior a thyroid tumor at the age oI 35 years. Her 28-year-
old sister has an increased serum calcitonin level but no thyroid mass. Examination
shows a palpable thyroid nodule. Her serum calcitonin level is increased. Which oI
the Iollowing diagnoses should be excluded prior to surgical treatment oI the thyroid
gland?
A) Adrenocortical carcinoma
B) Lung carcinoma
C) Ovarian carcinoma
D) Parathyroid carcinoma
E) Pheochromocytoma
Answer: E
37. A 57-year-old woman comes to the physician Ior a Iollow-up examination. She
has a 5-year history oI hypercalcemia, which was diagnosed with routine laboratory
studies, and her serum calcium levels have ranged Irom 10.8 mg/dL to 11.5 mg/dL.
She declined Iurther evaluation in the past because she "Ielt well." She takes no
medications. Her last menstrual period was 7 years ago. She maintains a weight oI 67
kg (148 lb) and is 170 cm (67 in) tall, BMI is 23 kg/m2. Her blood pressure is 126/80
mm Hg, and pulse is 66/min. Examination shows no abnormalities. Which oI the
Iollowing is the most appropriate next step to assess her risk Ior Iracture?
A) 24-Hour urine collection Ior measurement oI collagen cross-links excretion
B) Bone densitometry
C) Bone-speciIic measurement oI serum alkaline phosphatase activity
D) Posteroanterior and lateral x-ray Iilms oI the thoracic spine and hips
E) Biopsy oI the iliac crest
Answer: B
38. 13-month-old girl is brought Ior a well-child examination. There is no history oI
prenatal or perinatal problems. Her diet consists oI breast milk, juice, legumes,
cooked vegetables, and Iruit. She has had three ear inIections treated successIully with
amoxicillin and two episodes oI diarrhea since birth. She will pull to a stand and stand
alone Ior 1 minute but has not begun to walk. She is able to pick up raisins with her
thumb and IoreIinger but is unable to Ieed herselI with a spoon. Which oI the
Iollowing is the most appropriate assessment oI Iine and gross motor development?
Fine motor development Gross motor development
A Normal Normal
B Normal Delayed
C Delayed Normal
D Delayed Delayed
Answer: A
39. a previously healthy 37 yo woman comes to the physician beccause oIa 3 mth hx
oI episodes oI severe anxiety, SOB, palpitaltions and numbness in her hands and Ieet.
her vital signs are within normal limits, PE shows no abnormalities, thyroid Iunction
studies and an ECG show no abn. which oI the Iollowing is the most appropriate
pharmacology?
lithium
methyphenidate
olanzapine
paroxetine
valproic acid
Answer: D
40. Tx Ior 3rd degree AV block Immediate placement oI a pacemaker
41. A 57-year-old hospitalized man undergoes right subclavian venous catheterization
Ior hyperalimentation. He is currently being treated Ior a small bowel Iistula. While
the results oI an x-ray Iilm oI the chest to check the catheter position are pending, the
patient suddenly becomes agitated. His blood pressure is 70/50 mm Hg, and pulse is
110/min. Examination shows jugular venous distention. The lungs are clear to
auscultation. Breath sounds are equal bilaterally. The trachea is midline. An x-ray
Iilm oI the chest shows a catheter in the superior vena cava, transversing the right
ventricle and crossing to the leIt oI the midline. Which oI the Iollowing is the most
likely cause oI this patient's hypotension?
A ) Air embolism
B ) Pericardial tamponade
C ) Pulmonary artery perIoration
D ) Staphylococcal bacteremia
E ) Tension pneumothorax
Answer: E
42. 25year old nulligravid woman comes to the ER because oI severe pain in the RLQ
oI the abdomen Ior 4 hours. She has had no nausea, vomiting, Iever or chills. Three
weeks ago, a right adnexal mass was Iound on routine exam. Her LMP was 3 days
ago. Her temp is 99, BP is 110/70 pulse is 92. Pelvic exam shows right adnexal
tenderness. Hb is 13, WBC is 9000. Pelvic U/S shows small amount oI Iree Iluid in
the cul-de-sac. What is the cause oI her pain? Appendicitis, Endometriosis, Ovarian
hemorrhage, Ruptured ovarian cyst, Torsion oI the adnexa, Tubal obstruction
Answer: D
43. A 47-year-old woman comes to the physician because oI a 2-day history oI Iever
and joint pain. Six days ago, she completed a 10-day course oI dicloxacillin Ior
Iolliculitis. She appears comIortable. Her temperature is 38.6C (101.5F), pulse is
72/min and regular, respirations are 16/min, and blood pressure is 120/76 mm Hg.
Examination shows an urticarial rash over the trunk and extremities. There is
moderate generalized lymphadenopathy and diIIuse joint tenderness. The remainder
oI the examination shows no abnormalities. Which oI the Iollowing is the most likely
cause oI these symptoms?
A ) Epstein-Barr virus inIection
B ) IgE-mediated allergic reaction
C ) Mycobacterium haemophilum inIection
D ) Serum sickness
E ) Staphylococcal sepsis
Answer: D

Report Abuse

* Re:Ans key Ior NBME 3
#995133

coolamazon - 10/17/07 02:04
O
Permalink Reply by usmle99 on March 13, 2009 at 7:29am
bme3,BLOCK 4:
1. Six days aIter undergoing a laparoscopic cholecystectomy Ior acute cholecystitis, a
35-year-old woman comes to the physician because oI Iever and abdominal pain Ior 3
days. She is jaundiced. Her temperature is 38C (100.4F). Abdominal examination
shows distention and incisions that are healing normally. Leukocyte count is
12,000/mm3, and total serum bilirubin level is 7.9 mg/dL. Which oI the Iollowing is
the most likely cause oI the jaundice?
A) Anesthetic-related hepatitis
B) Common bile duct injury
C) Fulminant hepatic Iailure
D) Reaction to perioperative antibiotics
E) Subhepatic abscess
Answer: B
2. 47yo comes because oI increasing Iacial swelling 1 week oI morning HA and mild
SOB. He previously Ielt well. He Iinished Chemotx Ior small cell ca oI lung 4 months
ago. Temp is 98.9. There is diIIuse Iacial and periorbital edema. Dx SVC syndrome
3. The crude mortality rate Ior coronary artery disease in Community A is twice the
crude mortality rate Ior coronary artery disease in Community B. The age-
standardized mortality rates Ior coronary artery disease in the two communities are
the same. These Iindings are most consistent with which oI the Iollowing inIerences?
A ) CAD mortality in the elderly is higher in Community A than Community B
B ) CAD mortality in young adults is higher in Community A than Community B C )
The population oI Community B is younger than the population oI Community A
D) The proportionate mortality Irom CAD is higher in Com A than Community B
E ) The two communities have similar age distribution
Answer: C
4. 44 y/o woman brought to emergency 40 minutes aIter being stabbled in lIt groin.bp
128/84 pulse 100 RR 16 .exam shows small hematoma and no external
bleeding.popliteal and pedal pulses are palpablein lIt lower ext below injury
.laceartions oI Iem art n vein r Iound and repaired.2 days post op she has progresively
sev pain oI lIt lower ext and swelling oI leg Irom knee downward..wats most likely
cause oI her new Sx?
lymphatic disruption Irom injury
thrombosis oI Iem vein
wound hemorrage
compartment syndrome
arterial embolism
Answer: D
5. Crazy guy with crazy thoughts Ior one week SchizophreniIorm disorder
6. A 23-year-old woman, gravida 2, para 0, has had vaginal spotting and abdominal
cramps Ior 2 days. Her last menstrual period was 8 weeks ago. A home pregnancy test
was positive 2 weeks ago. She underwent a salpingectomy 4 years ago Iollowing an
ectopic pregnancy. Examination shows a closed cervix, an enlarged uterus, and no
adnexal masses. Transvaginal ultrasonography shows an empty uterus. Serum -hCG
level is 8000 mIU/mL.
A) Abortion oI a blighted twin
B) Cervicitis
C) Ectopic pregnancy
D) Focal decidual necrosis
E) Gestational trophoblastic disease
F) Incomplete abortion
G) Ovarian torsion
H) Ruptured ovarian cyst
I) Threatened abortion
Answer: C
7. A 25-year-old woman has had painless vaginal postcoital bleeding Ior 2 hours. Her
last menstrual period was 10 weeks ago. A pregnancy test is positive. There is a small
amount oI dark blood in the vaginal canal. She has a reddened, Iriable cervical os.
Same answers as question 6.
Answer: I
8. A 72-year-old woman is brought to the emergency department 1 hour aIter the
sudden onset oI right Iacial droop and weakness oI the right arm and leg. She takes
captopril Ior hypertension and daily aspirin. Her blood pressure is 150/90 mm Hg,
pulse is 80/min, and respirations are 16/min. Examination shows a leIt carotid bruit
and right central Iacial paralysis. There is moderate expressive aphasia. A CT scan oI
the head shows no abnormalities. Which oI the Iollowing is the most appropriate
initial pharmacotherapy?
A ) Oral ticlopidine
B ) Oral warIarin
C ) Sublingual niIedipine
D ) Intravenous nitroprusside
E ) Intravenous tissue plasminogen activator

9. A 50-year-old man has a cardiac arrest during sexual intercourse. He is promptly
resuscitated and recovers uneventIully Irom an acute anterior wall myocardial
inIarction. At discharge Irom the hospital, he and his wiIe inquire about the saIety oI
Iuture sexual intercourse. Which oI the Iollowing is the most appropriate advice?
A ) Psychiatric consultation is necessary
B ) Sexual intercourse should be perIormed while wearing an ambulatory ECG
monitor
C ) Sexual intercourse will be saIe when exercise tolerance improves
D ) They should wait 3 months beIore having sexual intercourse
E ) They should have sexual intercourse as soon as possible to overcome the Iear oI
another event
Answer: E (ReIer to Kaplan, says sex can start right aIter discharge)
10. 28 y/o man has dry hacking cough Ior 3 months.his paternal grandpa had colon Ca
exam NL ..CXR large anterior mediastinal mass.wats most likely origin oI this
neoplasm?
colon
kidney
prostate
stomach
testicle

11. Easy question on Tetralogy oI Iallot. Question said the kid squats and is cyanotic c
RVH.
12. Guy with cough, already on amoxicillin add steroid
13. Question oI a guy with recurrent ulcers, MCV is 78, wants to know what serum
iron, TIBC, saturation oI TIBC and Ierritin will be like (Low, High, High, Low)
14. A 3-year old boy with acute lymphoblastic leukemia has had Iever Ior 3 days. He
completed his last course oI chemotherapy 6 days ago. He has had no malaise, rash, or
anorexia and has had no known contact with sick children in preschool. He appears
well. His T is 38.6 degree, BP is 75/60mm Hg, pulse is 100/min, and respiration are
22/min. Examination shows normal Iindings. Laboratory studied show: Hb 10.1
(n11.5-15.5)
Leukocyte count 2200/mm3, segmented neutrophils 5, Bands 1, lymphocytes
65, atypical lymphocytes 11, monocytes 18, platelet count 35000/mm3. Which
oI the Iollowing is the most appropriate next step?
A. Schedule a Iollow-up visit and discharge without medication
B. Observe him in the oIIice Ior 3 hours, discharge without medication iI examination
remains unchanged
C. Discharge him with oral broad-spectrum antibiotic therapy
D. Admit him to the hospital Ior observation
E. Admit him to the hospital Ior intravenous broad-spectrum antibiotic therapy

15. a 15 y/o boy comes to the physician because oI acne over his Iace Io 1 year. He
preIers not to use any oral medication. Examination shows 10 to 15 pustules. which oI
the Iollowing is the most eIIective treatment Ior this patient`s symptoms
a) Avoidance oI chocolate, Iatty Ioods, and caIIeine
b) Use oI benzoyl peroxide soap
c) Vigorous scrubing oI aIIected areas
d) Application oI vitamin A and E oI aIIected areas
e) Ultraviolet light therapy

16. Vegetarian, MCV is 122 B12
17. 24 hrs aIter the Iixtation oI Iemur Iracture, 35 yr old construction worker develops
insomnia, nausea ,agitation. his bp is 150/100, oriented but anxious.which oI the
Iollowing is the most appropriate next step in diagnosis?
a. inquire about recent alcohol intake
b. ecg
c. pulm.arteriography
d. examination oI urine specimen Ior Iat globule
e.leukocye count wit diIIerential.

18. a 12 y/o grl is brought to th e physician by her mother because she is concerned
about her doughters diIIiculty making Iriends and socialising. her development and
adjustment has been normal until 6 months ago, whe she began to reIuse to use
restrooms at school or eat in the caIeteria. her mother describes her as a quiet serious
child who does not readily interact with other people. her teachers report that her
concentration varies, at times she appears to be daydreaming. on examination, she is
reserved but pleasant and appears to be oI normal inteligence. her speech is normal in
rate and rythm. she says that se is concerned that her voice will Iail her iI she has to
read aloud in class. which oI the Iollowing is the most likely diagnosis?
a) adhd, innatentive type.
b) autistic disorder
c) expressive language dissorder
d) oppositional deIiant disorder
e) selective mutism
I) social phobia
g) age appropriate behavior

19. Loss oI sensation oI last two Iingers Ulnar nerve at the elbow
20. a 68 y/o woman with terminal metastatic breast cancer is living with her son. She
has a living will requesting that she be allowed to die in peace. she is unresponsive to
voice and has not had any Iood Ior 3 days. Her son disagrees her desicion not to
accept Iurther therapy, including, chemotherapy, antibiotics, hospitalization and
enteral or parenteral nutrition. which oI the Iollowing is the most appropriate next step
in management?
a) abide to the decisions oI the next oI kin.
b) obtain a court order to allow additional therapy to be given
c) start enteral Ieedings
d) admit her to the hospital
e) no Iurther intervention

21. a 17 y/o girl has had Ilu-like symptoms, low grade Iever and malaise Ior 3 days
and mild jaundice Ior 2 days. serum studies show: AST: 670, ALT: 860,. SERUM
IgM Ior hepatitis A is positive. which oI the Iollowing is most likely to minimize the
risk Ior this diseace in Iamily members?
a) acyclovir therapy Ior Iamily members.
b) Hepatits B vaccination Ior Iamily members
c) Immune globuline therapy Ior Iamily members
d) interIeron alIa 2b Ior Iamily members
e) use oI separate toilet Iacilities by the patient

22. A 43-year-old man comes to the physician Ior evaluation and management oI
cardiac risk Iactors 8 weeks aIter sustaining a myocardial inIarction. He takes aspirin
and metoprolol daily, and he does not smoke cigarettes. His Iather and brother both
had myocardial inIarctions beIore the age oI 50 years; their serum cholesterol levels
are unknown. There is no Iamily history oI diabetes mellitus. He weighs 86 kg (190
lb) and is 180 cm (71 in) tall. His blood pressure is 130/70 mm Hg, pulse is 68/min,
and respirations are 14/min. Two years ago, his serum cholesterol level was 245
mg/dL. Fasting serum glucose level is 88 mg/dL. Which oI the Iollowing is the most
appropriate next step to evaluate his cardiac risk Iactors?
A ) Random measurements oI serum cholesterol level
B ) Measurement oI Iasting serum cholesterol level only
C ) Fasting serum lipid studies only
D ) Oral glucose tolerance test and Iasting serum lipid studies
E ) Oral glucose tolerance test and measurement oI Iasting serum cholesterol level

23. Guy with HBV Ior 25 yearrs, Increased AFP Dx? HCC
24. a 5 y/o girl has been Ieeding poorly since birth. she weghted 2900 gr at birth and
now weights 2600 gr. physical exam shows an enlarged clitoris and labial Iusion. A
sibling born 5 years ago died al 5 years oI age. A liIe threatening complication oI this
syndrome results Irom Iailure to produce which oI the Iollowing?
a) aldosterone
b) cholesterol
c) estrogen
d) insulin
e) testosterone

25. a previously healthy 3 y/o boy presents with Iever abd pain Ior 24 hrs.milestones
app Ior age ..Iully alert responsive. 101.5 temp ... 85/60 bp ...100 pulse RR 20 .exam
shows suprapubic tenderness but no gaurding or rebound.no and masses or costv
angel tendernes.no urethral discharge,
U/A shows 20- 30 leuk 5,6 eryth culture grows 100,000 colonies e coli all sensitive to
ab's..amox is initiated ..5 days later hes aIebrile asx U/S kidneys NL ..which oI teh
Iollowing is most app nxt stp Mx?
discontinue Ab's in 2 days and reexamine only is Sx recur
VCUG
IVP
cystoscopy
no Iurther testing

26. Two hours aIter emergency cholecystectomy, a 48-year-old woman has an oxygen
saturation oI 84 and a PO2 oI 56 mm Hg on 2 L/min oI oxygen via nasal cannula.
Her blood pressure is 120/80 mm Hg, and respirations are 16/min. Decreased breath
sounds are heard on the right, and there is decreased excursion on inspiration
bilaterally. There is minimal dullness over the right base. Examination shows no
jugular venous distention or pedal edema. An x-ray Iilm oI the chest shows increased
density over the right lower lung Iield; the tracheal, mediastinal, and cardiac
silhouettes are shiIted to the right. The right lung Iield appears considerably smaller
than the leIt lung Iield. Which oI the Iollowing is the most likely cause oI these
Iindings?
A ) Atelectasis
B ) Pleural eIIusion
C ) Pneumonia
D ) Pneumothorax
E ) Pulmonary embolism

27. Picture oI a hyphema
O A)Administration oI BCG vaccine
O B)Administration oI an inhaled bronchodilator
O C)Administration oI oxygen
O D)Arterial blood gas analysis
O E)Bronchoscopy
O F)Culture oI the pharynx Ior bacteria
O G)Direct laryngoscopy
O H) Gastric washings Ior acid-Iast bacteria
O I) Intravenous inIusion oI saline
O J) Isoniazid and riIampin therapy
O K) Lumbar puncture
O L) Measurement oI serum aspirin level
O M) Subcutaneous administration oI epinephrine.
N) Viral culture oI respiratory secretions
00) X-ray Iilms oI the chest
O P) X-ray Iilms oI the neck


28. A previously healthy 8-year-old boy is brought to the emergency department
because oI swelling oI the lips and diIIiculty breathing Ior 20 minutes. The symptoms
began when he was helping his Iather clean the gutters on their house. He appears
anxious. His temperature is 37.2C (99F), pulse is 120/min, and respirations are
50/min. Pulse oximetry shows an oxygen saturation oI 96. His lips and eyes appear
puIIy. He has subcostal and intercostal retractions. Auscultation oI the chest shows
diIIuse bilateral wheezing.


29. A 16-year-old girl is brought to the emergency department because oI heavy
breathing Ior 8 hours, and vomiting and ringing in the ears Ior 1 hour. She recently
broke up with her boyIriend and has been threatening to hurt herselI. She appears
somnolent but is arousable and answers questions appropriately. Her temperature is
37.5C (99.5F), pulse is 88/min, and respirations are 50/min. Pulse oximetry shows
an oxygen saturation oI 96. She has no retractions, nasal Ilaring, or cough. The
lungs are clear to auscultation


30. Guys PPD came back over 15mm. Next step - CXR
28. Question on dermatomyosits
29. Easy Q Tx Ior H.pylori Amox, Clarithromycin, Omeprazole
30. A guy taking doxorubicin gets S3 and crackles Cardiotoxicity
31. Picture oI spine with degenerative changes and pt has high calcium. Dx?
Herniated nucleus polposus, mechanical low back pain, met carcinoma, osteopetrosis,
spinal stenosis
32. A 37-year-old nulligravid woman comes to the physician because she has not been
able to conceive Ior 2 years. Her 40-year-old husband has a child by a previous
marriage. Her last menstrual period was 6 weeks ago; menses have occurred at
increasingly inIrequent intervals over the past year. FiIteen years ago, she was treated
Ior one episode oI Chlamydia trachomatis inIection. She weighs 59 kg (130 lb) and is
170 cm (67 in) tall. Examination shows no abnormalities. Serum studies show:
-hCG5 mIU/mL
Follicle-stimulating hormone 50 mIU/mL
Luteinizing hormone 45 mIU/mL
Prolactin 13 ng/mL
Thyroid-stimulating hormone 3 U/mL
Which oI the Iollowing is the most likely cause oI this patient's inIertility?
A) Hypothalamic amenorrhea
B) Hypothyroidism
C) Pituitary adenoma
D) Polycystic ovarian disease
E) Premature ovarian Iailure
F) Tubal Iactor

33. An 82-year-old man is brought to the emergency department because oI the
sudden onset oI conIusion 48 hours ago. His Iamily says that he previously had been
Iunctional and independent. His temperature is 37.5C (99.5F), blood pressure is
110/70 mm Hg, pulse is 90/min, and respirations are 12/min. The skin is warm and
clammy. Bowel sounds are hypoactive, and there is guarding and tenderness in the
right lower quadrant oI the abdomen. Examination oI the heart, lungs, and extremities
shows no abnormalities. He is drowsy but arousable and oriented to person but not
place or time. There are no Iocal deIicits. Which oI the Iollowing is the most likely
diagnosis?
A ) Appendicitis
B ) Colon cancer
C ) Meningitis
D ) Pneumonia
E ) Urinary tract inIection


34. A 6-year-old girl is brought to the physician because oI a 1-day history oI
vomiting, headache, and weakness. One month ago, she underwent resection oI an
astrocytoma and placement oI a ventriculoatrial shunt Ior residual hydrocephalus; Her
postoperative course had been uncomplicated. Currently, she is drowsy and irritable.
Her T 37.5 degree, BP 126/54 mmHg, pulse is 82/min, and respirations are 24/min.
Funduscopic examination shows papilledema. ReIlexes are brisk with hypertonia in
the lower extremities. Which oI the Iollowing is the most likely underlyiong
mechanism Ior these symptoms?
A. Carcinomatous inIiltration oI cerebrospinal Iluid
B. MalIunction oI ventriculoatrial shunt
C. Overproduction oI cerebrospinal Iluid Irom a choroid plexus papilloma
D. Recurrence and extension oI the astrocytoma
E. Thrombosis oI the superior sagittal sinus


35. A 30 year old woman, g4p3, is admitted to the hospital in labor at 38 weeks
gestation. The cervix is 4 cm dilated. Contractions occur every 4 minutes. The Iundal
height is 40 cm, the membranes are intact. The Ietal lie cannot be determined by abd
examination, and no presenting part is palpable in the pelvis. Which oI the Iollowing
is the most appropriate next step in management?
A. Ultrasonography
B. Administration oI oxytocin
C. Administration oI tocolytic drugs
D. Amniotomy
E. Cesarean delivery

35. See above
36. A previously healthy 30-year-old woman has had a painless lump in her neck Ior 2
days. Her mother was treated Ior a thyroid tumor at the age oI 35 years. Her 28-year-
old sister has an increased serum calcitonin level but no thyroid mass. Examination
shows a palpable thyroid nodule. Her serum calcitonin level is increased. Which oI
the Iollowing diagnoses should be excluded prior to surgical treatment oI the thyroid
gland?
A) Adrenocortical carcinoma
B) Lung carcinoma
C) Ovarian carcinoma
D) Parathyroid carcinoma
E) Pheochromocytoma

37. A 57-year-old woman comes to the physician Ior a Iollow-up examination. She
has a 5-year history oI hypercalcemia, which was diagnosed with routine laboratory
studies, and her serum calcium levels have ranged Irom 10.8 mg/dL to 11.5 mg/dL.
She declined Iurther evaluation in the past because she "Ielt well." She takes no
medications. Her last menstrual period was 7 years ago. She maintains a weight oI 67
kg (148 lb) and is 170 cm (67 in) tall, BMI is 23 kg/m2. Her blood pressure is 126/80
mm Hg, and pulse is 66/min. Examination shows no abnormalities. Which oI the
Iollowing is the most appropriate next step to assess her risk Ior Iracture?
A) 24-Hour urine collection Ior measurement oI collagen cross-links excretion
B) Bone densitometry
C) Bone-speciIic measurement oI serum alkaline phosphatase activity
D) Posteroanterior and lateral x-ray Iilms oI the thoracic spine and hips
E) Biopsy oI the iliac crest

38. 13-month-old girl is brought Ior a well-child examination. There is no history oI
prenatal or perinatal problems. Her diet consists oI breast milk, juice, legumes,
cooked vegetables, and Iruit. She has had three ear inIections treated successIully with
amoxicillin and two episodes oI diarrhea since birth. She will pull to a stand and stand
alone Ior 1 minute but has not begun to walk. She is able to pick up raisins with her
thumb and IoreIinger but is unable to Ieed herselI with a spoon. Which oI the
Iollowing is the most appropriate assessment oI Iine and gross motor development?
Fine motor development Gross motor development
A Normal Normal
B Normal Delayed
C Delayed Normal
D Delayed Delayed

39. a previously healthy 37 yo woman comes to the physician beccause oIa 3 mth hx
oI episodes oI severe anxiety, SOB, palpitaltions and numbness in her hands and Ieet.
her vital signs are within normal limits, PE shows no abnormalities, thyroid Iunction
studies and an ECG show no abn. which oI the Iollowing is the most appropriate
pharmacology?
lithium
methyphenidate
olanzapine
paroxetine
valproic acid

40. Tx Ior 3rd degree AV block Immediate placement oI a pacemaker
41. A 57-year-old hospitalized man undergoes right subclavian venous catheterization
Ior hyperalimentation. He is currently being treated Ior a small bowel Iistula. While
the results oI an x-ray Iilm oI the chest to check the catheter position are pending, the
patient suddenly becomes agitated. His blood pressure is 70/50 mm Hg, and pulse is
110/min. Examination shows jugular venous distention. The lungs are clear to
auscultation. Breath sounds are equal bilaterally. The trachea is midline. An x-ray
Iilm oI the chest shows a catheter in the superior vena cava, transversing the right
ventricle and crossing to the leIt oI the midline. Which oI the Iollowing is the most
likely cause oI this patient's hypotension?
A ) Air embolism
B ) Pericardial tamponade
C ) Pulmonary artery perIoration
D ) Staphylococcal bacteremia
E ) Tension pneumothorax

42. 25year old nulligravid woman comes to the ER because oI severe pain in the RLQ
oI the abdomen Ior 4 hours. She has had no nausea, vomiting, Iever or chills. Three
weeks ago, a right adnexal mass was Iound on routine exam. Her LMP was 3 days
ago. Her temp is 99, BP is 110/70 pulse is 92. Pelvic exam shows right adnexal
tenderness. Hb is 13, WBC is 9000. Pelvic U/S shows small amount oI Iree Iluid in
the cul-de-sac. What is the cause oI her pain? Appendicitis, Endometriosis, Ovarian
hemorrhage, Ruptured ovarian cyst, Torsion oI the adnexa, Tubal obstruction

43. A 47-year-old woman comes to the physician because oI a 2-day history oI Iever
and joint pain. Six days ago, she completed a 10-day course oI dicloxacillin Ior
Iolliculitis. She appears comIortable. Her temperature is 38.6C (101.5F), pulse is
72/min and regular, respirations are 16/min, and blood pressure is 120/76 mm Hg.
Examination shows an urticarial rash over the trunk and extremities. There is
moderate generalized lymphadenopathy and diIIuse joint tenderness. The remainder
oI the examination shows no abnormalities. Which oI the Iollowing is the most likely
cause oI these symptoms?
A ) Epstein-Barr virus inIection
B ) IgE-mediated allergic reaction
C ) Mycobacterium haemophilum inIection
D ) Serum sickness
E ) Staphylococcal sepsis

You might also like